YCT Capacitance NEET JEE Questions Practice

You might also like

Download as pdf or txt
Download as pdf or txt
You are on page 1of 152

05.

Capacitance
Let C1 and C2 be the capacitance of the system for
(a) Capacitance 1 2d
x = d and x = , respectively. If C1 = 2µF
3 3
1. The distance between two plates of a capacitor the value of C2 is ____µF
is d and its capacitance is C1, when air is the
medium between the plates. If a metal sheet of JEE Main-06.04.2023, Shift-I
2d Ans. (3) : Given, plate Area = A
thickness and of same area as plate is Plate separation = d, Dielectric constant (K) = 4
3
introduced between the plates, the capacitance C1 = 2µF
When, dielectric of thickness,
C
of the capacitor becomes C2. The ratio 2 is: 1
C1 x = d are inserted between plates.
3
(a) 2 : 1 (b) 4 : 1
εo A εo A
(c) 3 : 1 (d) 1 : 1 C1 = =
JEE Main-10.04.2023, Shift-II x d d /3
d−x+ d− +
Ans. (c) : Given, K 3 K
Case 1 – When air is the medium between the plates. εo A εo A
C1 = =
εA 2d d / 3 2d d
Capacitance (C1) = o ......(i) + +
d 3 4 3 12
2d 12 ε A
Case 2 – When, t = thickness sheet introduce C1 = ⋅ o
3 9 d
between plates 2d
When dielectric of thickness x = are inserted
εo 3
Capacitance (C2) = between plates
t
d−t+ εo εo A
K C2 = =
εo A x 2d 2d / 3
d−x+ d− +
C2 = K 3 K
2d t
d− + εo εo A
3 ∞ C2 = =
εA d 2d / 3 d d
C2 = o + +
d 3 4 3 6
εA
3 C2 = 2 ⋅ o
εA d
C2 = 3 o Ratio of C1 and C2,
d
εA
Ratio of capacitance C1 and C2 2 o
C2 d = 2× 9 = 3
εA =
3 o C1 12 ε A 12 2
C2 3 ⋅ o
= d = 9 d
C1 ε A 1
o 3 3
d Therefore, C2 = C1 = × 2 = 3µF
2 2
2. A parallel plate capacitor with plate area A and 3. In the given circuit,
plate separation d is filled with a dielectric C1 = 2 µF, C2 = 0.2 µF, C3 = 2 µF, C4 = 4 µF, C5
material of dielectric constant K=4. The thickness = 2 µF , C6 = 2 µF, the charge stored on
of the dielectric material is x, where x < d, capacitor C4 is …..µC.

JEE Main-11.04.2023, Shift-II


Objective Physics Volume-III 809 YCT
Ans. (4):  3
Electric field at distance  rp = 
 π
λ σ
EP = −
2πε o rm 2ε o
λ  λ 
EP =  − σ
From question 2εo λ × 3
Equivalent capacitance of C3, C4 and C5  
 λ 
1 1 1 1 1 1 1 1 λ 1  λ − 3σ 
= + + = + + 
EP = − σ =
Ceq 1 C3 C4 C5 2 4 2 2εo  3  2εo  3 
4
Ceq1 =  4
5 Electric field at distance  rQ = 
 π
Equivalent capacitance of C2, C3, C4, C5
 λ σ 
Ceq2 = C2 + Ceq = 0.2 +
4
=1 EQ =  − 
5  2πε o rQ 2ε o 
Equivalent capacitance of total circuit λ σ
1 1 1 1 EQ = −
= + + 4 2εo
2πεo
Ceq C1 Ceq 2 C6 π
1 1 1 1 1 λ 
= + + EQ = − σ
Ceq 2 1 2 2εo  4 
1 1+ 2 +1 1  λ − 4σ 
= EQ =
Ceq 2 2εo  4 
Ratio of EP and EQ
1 4
= 1  λ − 3σ 
Ceq 2  
E P 2εo  3 
1 =
Ceq = = 0.5µF EQ 1  λ − 4σ 
2  
Q = 0.5 × 10 = 5 µC 2 εo  4 
Charge on C4 capacitance, E P 4  λ − 3σ 
=  
Q × 0.8 5 × 0.8 E Q 3  λ − 4σ 
Q′ = =
0.8 + 0.2 0.8 + 0.2 λ = 2σ
Q′ = 4µC E P 4  2σ − 3σ 
=
E Q 3  2σ − 4σ 
4. A thin infinite sheet charge and an infinite line
charge of respective charge densities +σ and +λ
are placed parallel at 5 m distance from each E P 4  −σ 
=
other points. 'P' and 'Q' are at
3
m and
4
m E Q 3  −2σ 
π π
perpendicular distance from line charge EP 4 4
= =
towards sheet charge, respectively. 'EP' and EQ 6 a
'EQ' are the magnitudes of resultant electric a=6
field intensities at point 'P' and 'Q', The value of a is 6
E 4
respectively. If P = for 2|σ| = |λ|. Then the 5. As shown in the figure, two parallel plate
EQ a capacitors having equal plate area of 200 cm2
value of a is ______. are joined in such a way that a ≠ b. The
equivalent capacitance of the combination is x
JEE Main-13.04.2023, Shift-I ε0 F. The value of x is ______.
Ans. (6) : Charge density of charge sheet = +σ
Charge density of infinite line = +λ

JEE Main-06.04.2023, Shift-II

Objective Physics Volume-III 810 YCT


Ans. (5) : Given that, Reason R: Capacitance of metallic spheres
Area (A) = 200 cm2 = 200 × 10–4 m2 depend on the radii of spheres.
d = 5 mm = 5 × 10–3 m In the light of the above statements, choose the
correct answer from the options given below.
c = 1mm = 1 × 10–3 m (a) Both A and R are true and R is correct
We know that, explanation of A
ε0 A (b) Both A and R are true but R is not the correct
Capacitance of capacitor, C =
( − c)
d explanation of A
(c) A is true but R is false
εo × 200 × 10−4 (d) A is false but R is true
C =
5 × 10−3 − 1×10−3 JEE Main-01.02.2023, Shift-II
ε × 200 × 10−4 Ans. (d) : In case of metallic sphere, either solid or
C= 0 hollow, the charge will reside on the surface of the
4 ×10−3 sphere. Since both spheres have same surface area, so
C = εo × 5 they can hold equal maximum charge.
∴ x=5 The capacitance of sphere depends on its radius because
The situation is equivalent to a conducting slab placed both spheres have equal charges.
between the plates. Hence, Assertion is false and Reason is true.
6. A parallel plate capacitor with air between the 8. A big drop of radius R is formed by 729 small
plate has a capacitance of 15pF. The separation drops of water of radius r, then the radius of
between the plate becomes twice and the space each small drop will be:
between them is filled with a medium of R R
(a) (b)
dielectric constant 3.5. Then the capacitance 9 900
x R R
becomes pF. The value of x is ____ (c) (d)
4 1800 9000
JEE Main-24.01.2023, Shift-II AIIMS-1997
Ans. (105) : Capacitance between the parallel plate Ans. (a) : Number of drops (n) = 729
capacitor when filled with air, C = 15 pF Radius of big drop = R
But we know that, Radius of each small drop = r
Equating volume in big drop and small drop,
εA
C = 0 = 15 pF .....(i) 4 3 4
d πR = n πr 3
3 3
If the distance between the plates becomes twice, then R3 = 729r3
new distance d′ = 2d
R3
Dielectric constant of the substance filled in between r=
the plates, k′ = 3.5 (729)1/ 3
Hence, the capacitance of the capacitor becomes, R
r=
k ′ε A 9
C′ = 0
d′ 9. The earth is assumed to be a charged
3.5ε0 A conducting sphere having volume ‘V’ and
C′ = .....(ii) surface area ‘A’. The capacitance of the earth
2d
in free space is
On solving equation (i) & (ii), we get–
(ε0 = permittivity of free space)
3.5
C′ = C V V
2 (a) 4πε0 (b) 8πε0
A A
3.5
C′ = × 15 (c) 2πε0
V
(d) 12πε0
V
2 A A
105
C′ = pF MHT-CET 2020
4 4
Therefore, the value of x is 105. Ans. (d) : Volume of earth (V) = πR 3
3
7. Given below are two statements: One is Area of earth (A) = 4πR2
labelled as Assertion A and the other is labelled
Ratio of volume of earth to the area of earth,
as Reason R.
4 3
Assertion A: Two metallic spheres are charged πR
V 3
to the same potential. One of them is hollow =
and another is solid, and both have the same A 4πR 2
radii. Solid sphere will have lower charge than V R 3V
= ⇒R =
the hollow one. A 3 A
Objective Physics Volume-III 811 YCT
Where, R → radius of earth Ans. (a): Capacitor of a conductor is ratio of charge (Q)
Q Capacitance of earth in free space, to it by rise in its potential (V).
C = 4πε0R Q kε A
3V C= = 0
= 4πε0 × V d
A
∴ When a dielectric slab is introduced between the
V
C =12πεo × plates of a capacitor connected to battery, Thus capacity
A
of capacitor will increase i.e., charge on capacitor
10. A capacitor of unknown capacitance C is
increases and voltage across capacitor decreases.
connected across a battery of V volt. The
charge stored in it becomes Q coulomb. When 13. A capacitor of capacitance 10µF is charged to
potential across the capacitor is reduced by V′ potential 50 V with a battery. The battery is
volt, the charge stored in it becomes Q′ now disconnected and an additional charge
coulomb. The capacitance C is 200 µC is given to the positive plate of the
Q – Q′ V′ capacitor. The potential difference across the
(a) (b)
V ′ Q – Q′ capacitor will be
Q + Q′ Q – Q′ (a) 50 V (b) 80 V
(c) (d) (c) 100 V (d) 60 V
V′ V′
MHT-CET 2020 Manipal UGET-2015
Ans. (a) : We know that – Ans. (d) : Given, Capacitor of capacitance (C) = 10µF
Q = CV ....(i) Charged to potential (V) = 50 V
According to the question,
Q' = C(V−V') = CV − CV'
Q' = Q − CV' (from (i))
∴ CV' = Q − Q'
Q − Q'
C=
V'
Charge (Q) = CV
11. Assume each oil drop consists of a capacitance
of C. If combine n drops to form a bigger drop, = 10 × 50 = 500 µC
then the capacitance of bigger drop C' would Q Additional charge to the positive plate (Q') = 200 µC
be
2n1/ 3 5n1/ 3
(a) C ' = C (b) C ' = C
3 4
n1/ 3
(c) C ' = C (d) C ' = C.n1/ 3
5
TS-EAMCET-10.09.2020, Shift-1 Total charge on positive plate = 700µC
Ans. (d) : Radius of oil drop = r Total charge in negative plate = −500 µC
Capacitance of an oil drop (C) = 4πε0r Q Net electric field at point P is zero
Q n × volume of each small drop = volume of large Ep = 0
drop
4 4
n × πr 3 = πR 3
3 3
R = n1/ 3 .r
So, capacitance of large drop,
C' = 4πεo R = 4πεo n1/ 3 r
C' = n1/ 3C
12. When a dielectric slab is introduced between
the plates of a capacitor connected to a battery, 700 − Q Q 500 − Q Q
then ∴ + + =
2Aεo 2Aεo 2Aεo 2Aεo
(a) charge on capacitor increases
(b) potential difference across the capacitor Q = 600 µC
increases ∴ Potential difference between the plates,
(c) energy stored increases Q 600
(d) capacity remains the same ∆V = = = 60V
C 10
Manipal UGET -2020
Objective Physics Volume-III 812 YCT
14. The capacity of a parallel plate capacitor Kεo A
increases with the C=
(a) Decreases of its area d
Where, A = Area of the parallel plate
(b) Increase of its distance
(c) Increases of its area d = Distance between parallel plate
(d) None of these εo = Permittivity of free space (8.85 × 10−12
F/m)
Karnataka CET-2015
CG PET -2018 K = Dielectric constant
Hence, the capacitance of a parallel plate condenser
MHT-CET-2020
Ans. (c) : We know that, does not depend upon metal of plates.
17. A 500 µF capacitor is charged at a steady rate
In a parallel plate capacitor, the capacity of capacitor,
εA of 100 µC/s. The potential difference across the
C=K o capacitor will be 10 V after an interval of
d
Where, A = Area of the plate (a) 5 s (b) 25 s
d = Distance between two plate (c) 20 s (d) 50 s
1 CG PET- 2008
Then, C ∝ A & C ∝ Ans. (d) : Given,
d
Capacitor (C) = 500 µF
So, the capacity of a parallel plate capacitor increases
Potential difference (V) = 10 V
with the increase its area and the decrease its distance.
15. Q charge is given to two capacitors C1 and C2 Charged at a steady rate (current I) = 100µC/s
which are in parallel. The charge distribution I = 100 × 10−6 C/s
among them is Q Q = CV
(a) C1 :C2 (b) C2 :C1
Q = 500 µF × 10 V = 5 × 10−3 C
(c) C1 C2 :1 (d) 1 :C1C2
Now, we know-
CG PET- 2012
Q = It
Ans. (a) : When C1 and C2 are connected in parallel
Q
they have the same potential difference. t=
I
5 × 10−3
t=
100 × 10−6
t = 50 s
18. Capacity of parallel plate capacitor is 10 µF,
when gap between plates is 8 cm, then what will
Then, V1 = V2 = V be its capacity if gap is reduced to 4 cm?
We know, Q = C.V (a) 10 µF (b) 40 µF
Q1 C1V C1 (c) 20 µF (d) 30 µF
So, = =
Q2 C2 V C2 CG PET- 2007
Q1 : Q2 = C1 : C2 Ans. (c) : Given,
16. The capacitance of a parallel- plate condenser Capacitor (C1) = 10µF
does not depend upon Gap between plates(d1) = 8cm
(a) Area of the plates Gap is reduce (d2) = 4cm
(b) Medium between the plates Kε A
For parallel plate capacitor, C = o
(c) Distance between the plates d
(d) Metal of the plates Q A1 = A2
CG PET- 2010 C1d1 C2 d 2
=
Ans. (d) : From question, εo εo
For parallel plate condenser,
Cd
C2 = 1 1
d2
10 × 8
C2 = = 20µF
4
19. Two metal spheres of capacitance C1 and C2
carry some charges. They are put in contact
and then separated. The final charges Q1 and
Q2 on them will satisfy

Objective Physics Volume-III 813 YCT


Q1 C1 Q1 C1 Ans. (b):
(a) < (b) =
Q2 C2 Q2 C2
Q1 C1 Q1 C2
(c) > (d) =
Q2 C2 Q 2 C1 Where, e → Charge on the particle
CG PET- 2005 Ee → Electric force working on particle
Particle is suspended in an equilibrium,
Ans. (b) : Potential difference V across capacitor, F = Ee = mg
Q
V= ∴ E=
V
C d
Where, Q = Charge on the capacitor
800
C = Capacitance of the capacitor = = 4 × 104 V/m
Therefore, after putting them in contact and separating 0.02
them, if the final charge are given by Q1 & Q2 ∴ Ee = mg
V1 = V2 4 ×104 ne = 1.96 × 10–15 × 9.8
1.96 × 10−15 × 9.8
Q1 Q 2
=
Q
⇒ 1 = 1
C ne = [∴ n→ Number of electrons]
C1 C 2 Q2 C2 4 × 104
1.96 × 10−15 × 9.8
Note:- When two conductors are placed in contact with n=
each other they acquire same potential. 4 × 104 × e
20. A parallel plate capacitor with air between the 1.96 × 10−15 × 9.8
n= = 2.9  3
plates has a capacitance of 9 pF. The 4 × 104 × 1.6 ×10−19
separation between the plates is d. The space n=3
between the plates is now filled with two The charge on the particle must be 3e.
dielectrics constant K1 = 3 and thickness d/3 22. Spherical capacitors has outer sphere of radius
while the other one has dielectric constant K2 = 5 cm and inner sphere of radius 2 cm. when the
6 and thickness 2d/3. Capacitance of the inner sphere is earthed, its capacity is C1 and
capacitor is now when the outer sphere is earthed its capacity is
(a) 1.8 pF (b) 45 pF C
(c) 40.5 pF (d) 20.25 pF C2. Then 1 is
C2
BITSAT-2017
5 2 7 3
Ans. (c) : We know, (a) (b) (c) (d)
For parallel plate capacitors, 2 5 3 7
AP EAMCET (23.04.2019) Shift-I
Ans. (a) : Given that,
Radius of inner sphere, R1 =2 cm
Radius of outer sphere, R2 = 5 cm
When outer shell is earthed
R1 R 2
C2 = 4 π ε o
C = εo
A
= 9 pF ....(i) ( 2 − R1 )
R
d When inner shell is earthed
The equivalent capacitance, C1 = C2 + 4 π ε o R2
K1 ε o A K 2 ε o A
×  R R 
Ceq = d / 3 2d / 3 Then, C1 = 4 π ε o  1 2 + R 2 
K1εo A K 2 εo A  R 2 − R1 
+
d /3 2d / 3  2×5 
= 4 π εo  + 5
3K1K 2 ε A 3× 3× 6 5 − 2 
= × o = ×9
2 K1 + K 2 d 2×3+ 6 25
Ceq = 40.5 pF C1 = 4 π ε o × Farad
3
21. Two metal plates having potential difference of R1 R 2
800 V are 2 cm apart. It is found that a particle And, C2 = 4π εo
of mass 1.96 × 10–15 kg remains suspended in
( 2 − R1 )
R
the region between the plates. The charge on 2× 5 10
= 4 π εo = 4 π εo Farad
the particle must be (e = elementary charge) (5 − 2) 3
(a) 2 e (b) 3 e
(c) 6 e (d) 8 e C1 25 3
Hence, = =5 2
BITSAT-2017 C2 10 3

Objective Physics Volume-III 814 YCT


23. 27 small drops each having charge q and radius 25. The capacitance of a spherical conductor with
r coalesce to form big drop. How many times radius 1 m is
charge and capacitance will become? (a) 9×109 F (b) 1µ F
(a) 3, 27 (b) 27, 3 (c) 1.1 × 10–10F (d) 1×10–6F
(c) 27, 27 (d) 3, 3 JCECE-2014
JCECE-2007 Ans. (c) : Given,
UPSEE - 2006 Radius of spherical conductor (r) = 1m
Ans. (b) : Given, We know the capacitance of spherical conductor -
Total number of drop = 27 C = 4πεor
Radius of big drop = R r
Radius of small drop = r C=
1
In coalescing into a single drop, charge remains
conserved charge on big drop = 27 × Charge of small 4πε0
drop 1
Since, = 9 ×109 Nm 2 C2
q' = 27q 4πε0
Since, Volume is also remain same. 1
4 3 4 So, C= {r = 1m}
πR = 27 × πr 3 9 ×109
3 3 C = 0.11 ×10 −9 F
R3 = (3r)3
R = 3r C = 1.1×10–10F
So, capacitance of small drop = 4πεor 26. In the figure, charge and the potential
And, capacitance of bigger drop difference across the 4µF capacitor will be
C' = 4πεoR nearly
= 4π εo(3r)
= 12πεor = 3 × (4πεor)
Hence, C' = 3C
24. Figure below shows four plates each of area A
and separated from one another by a distance d.
What is the capacitance between P and Q?
(a) 600 µC, 150 V (b) 300 µ C, 75 V
(c) 800 µ C, 200 V (d) 580 µ C, 145 V
JCECE-2011
Ans. (d) : Given diagram,
εA 2ε 0 A
(a) 0 (b)
d d
3ε0 A 4ε 0 A
(c) (d)
d d
(e) Zero
WBJEE-2013
BCECE-2013
GUJCET-2019
Kerala CEE 2005
AP EAMCET-06.09.2021 (Shift-II)
Ans. (b) : From question,
The given circuit is equivalent to a parallel combination
two identical capacitors.
Hence, equivalent capacitance between P and Q is. From figure,
All capacitor in series,
1 1 1 1 31
= + + =
Ceq 8 20 12 120
120
Two capacitors C in parallel order are formed between Ceq = µF
the plates. 31
CPQ = C1 + C2 Total charge of (8µF) = Q = CV
ε A ε A 2ε A 120
CPQ = o + o = o Q= × 300 = 1161 µC
d d d 31
Objective Physics Volume-III 815 YCT
Q 1161 30. A capacitor of capacitance 5µF is connected as
Then, charge of (4µF) = = µC shown in the figure. The internal resistance of
2 2
the cell is 0.5Ω. The amount of charge on the
= 580µC capacitor plate is–
Q
Hence, Potential difference, (V) =
C
580
= = 145V
4
27. Instantaneous displacement current of 1.0 A in
the space between the parallel plates of 1 µF (a) zero (b) 5µC
capacitor can be established by changing (c) 10µC (d) 25µC
potential difference of BCECE-2017
(a) 10–6 V/s (b) 106 V/s Ans. (c) :
–8
(c) 10 V/s (d) 108 V/s
JCECE-2011
Ans. (b) : Given that,
Current, I = 1A ,
Capacitance, C = 1µF
q it Internal resistance = 0.5Ω
We know, V = =
C C Capacitor of capacitance = 5µF
dV i 1 Net resistance of the circuit, R = 1+1+0.5 = 2.5Ω
Then, = = = 106 V / s
dt C 1× 10−6 Current flow from the cell (I) =
V 2.5
= = 1A
dV R 2.5
Hence, = 106 V / s Potential difference between two parallel plate;
dt
V = E − Ir = 2.5−1×0.5 = 2V
28. A capacitor of 10µF is connected to a 10 V cell. So, charge on the capacitor plates,
The maximum charge on the capacitor will be Q = CV = 5×2 = 10µC
(a) 1µC (b) 10 µC 31. The 90 pF capacitor is connected to a 12V
(c) 100 µC (d) 1000 µC battery. How many electrons are transferred
COMEDK 2012 from one plate to another?
Ans. (c) : Given that, (a) 1.1× 109 (b) 6.7 × 109
Capacitance, C = 10µF (c) 4 × 1019 (d) 5 × 1019
Potential, V = 10 Volt BCECE-2011
We know, Q = CV Ans. (b) : Given that,
Q = 10 × 10 Capacitance, C = 90pF
Potential of battery, V = 12V
Q = 100µC
Charge (Q) = CV
29. Two capacitors C1 and C2 are charged to 120 V = 90×10–12 × 12 = 108×10–11C
and 200 V respectively. When they are Conversion from charge to electrons,
connected in parellel, it is found that potenial Q = ne
on each one of them is zero. Therefore, Q
(a) 5 C1 = 3 C2 (b) 3 C1 = 5 C2 n=
e
(c)3 C1+ 5 C2= 0 (d)9 C1= 4 C2
108 ×10−11
COMEDK 2020 =
1.6 × 10−19
Ans. (b) : Given that, = 6.75×109
Q = CV
32. A 6 × 10 −4 F parallel plate air capacitor is
Two capacitors C1 & C2
connected to a 500V battery. When air is
Q1 = 120V , Q2 = 200V replaced by another dielectric material, 7.5×10–
Potential can be made zero only when they are 4
C charge flows into the capacitor. The value of
connected in parallel, the dielectric constant of the material is–
120C1 = 200C2 (a) 1.5 (b) 2.0
6C1 = 10C2 (c) 1.0025 (d) 3.5
3C1 = 5C2 BCECE-2011
Objective Physics Volume-III 816 YCT
Ans. (c): Capacitance (C) = 6×10–4 F 34. In a parallel plate air capacitor the distance
(V) = 500V between plates is reduced to one forth and the
Charge, (Q) = VC space between them is filled with a dielectric
= 500×6×10–4 medium of constant 2. If the initial capacity of
= 3000 ×10–4 C the capacitor is 4µF, then its new capacity is
Air is replaced by another dielectric material, (a) 8 µF (b) 18 µF
Q' = 7.5×10–4C (c) 44 µF (d) 32 µF
Total charge stored, MHT-CET 2019
Qnet = 3000×10–4 + 7.5×10–4
= 3007.5 ×10–4 C Ans. (d) : Given that,
Capacitance of parallel plate, Capacitance, C = 4×10–6 F
Dielectric constant, K = 2
Q 3007.5 × 10−4
C' = net = = 6.015 × 10−4 F Distance, d1 = d/4
V 500
For capacitance of parallel plate,
Q C' = KC, where, K = dielectric constant
A
∴ 6.015 × 10−4 = K × 6 × 10–4 C = εo ....(i)
d
K = 1.0025
Where, εo = permittivity of the medium
33. The numerical value of charge on either plate
of capacitor C shown in figure– A = Area of plates
d = distance between the plates
When the distance between plates is reduced and a
dielectric slab is introduced, then capacitance,
εA
C' = K o ....(ii)
d1
Where, K = dielectric constant of medium
CER1
(a) CE (b) From equation (ii), we get-
R1 + r
( Kεo A ) = 4K  εo A 
CER 2 CER1 C' =  
(c) (d) d1  d 
R2 + r R2 + r
C' = 4KC
BCECE-2010
= 4 × 2 × 4 × 10–6
Ans. (b) :
= 32 µF
35. If the distance between the plates of a parallel
plate capacitor of capacity 10 µF is doubled,
then new capacity will be
(a) 5 µF (b) 20 µF
(c) 10 µF (d) 15 µF
MHT-CET 2011
Ans. (a) : Given that,
There will be no current in the branch containing Capacitance, C = 10µF
1
capacitor (as the resistance offered by the capacitor in
D.C. circuit in infinite). Therefore current drown from Initial distance, d1 = d
the battery Final distance, d 2 = 2d
E For parallel plate capacitor
I=
r + R1 C = εo A .....(i)
Potential difference across the terminals of the battery d
1
Er C∝
V = E−Ir = E− d
r + R1
C1 d 2
ER1 =
= C2 d1
r + R1
C d 10 × d
CER1 C2 = 1 1 =
∴ Charge on capacitor (Q) = CV = d2 2d
r + R1
C2 = 5µF

Objective Physics Volume-III 817 YCT


36. A parallel plate condenser with oil (dielectric C = 100 pF = 100 × 10−12 F, εo = 8.85 × 10−12 C2/Nm2
constant 2) between the plates has capacitance A = πr2 = 3.14 × (10−2)2 (r = 1cm)
C. If oil is removed, then capacitance of
capacitor becomes t = 10−3 m, d = t, K = 4
(a) 2C (b) 2 C 8.85 × 10 −12 × 3.14 ×10−4
C1 =
C C  1
(c) (d) 10−3 − 10−3 1 − 
2 2  4
MHT-CET-2000, 2007 27.79 × 10−16
=
Ans. (d) : For capacitance of a parallel plate:- 10−3 − 0.75 ×10−3
A 27.79 ×10−13
C = K εo ....(i) =
d 0.25
When dielectric (oil) is removed,
27.79 × 10−13
= =111.16 ×10 −13 F (for 1 set)
Capacitance (C') = εo A .....(ii) 0.25
d
Comparing equation (i) & (ii), we get-
C Required C = 100 × 10−12 F
= C'
K C 100 × 10−12
n= = =10
C C C1 111.16 ×10 −13
C = KC' ⇒ C' = =
K 2 39. A potential difference of 300 V is applied to a
37. If the distance between the plates of parallel combination of 2.0 µF and 8.0 µF capacitors
plate capacitor is halved and the dielectric connected in series. The charge on the 2.0µF
constant is doubled, then its capacity will
capacitor is
(a) increase by 16 times (b) increase by 4 times
(c) increase by 2 times (d) remain the same (a) 2.4×10−4C (b) 4.8×10−4C
−4
MHT-CET 2003 (c) 7.2×10 C (d) 9.6×10−4C
Ans. (b) : Given that, VITEEE-2006
Initial distance, d1 = d Ans. (b) : Potential difference (V) = 300V
Final distance, d2 = d/2 C1=2µF C2=8µF
First dielectric constant, K1 = K
Second dielectric constant, K2 =2K
First capacitance, C1= C
For parallel plate, V = 300V
1 1 1 1 1 5
C = K εo A In series, = + = + =
d C C1 C 2 2 8 8
K 8 8
C∝ C= µF = × 10−6 F
d 5 5
C1 d 2 K1 Charge (Q) = VC
= ×
C2 d1 K 2 8
= 300× ×10−6
C d K 1 5
= × =
C2 2d 2K 4 = 4.8×10–4C
C2 = 4C 40. Four metal plates are arranged as shown in the
38. A parallel plate capacitor of capacitance 100 figure. Capacitance between X and Y (A→
pF is to be constructed by using paper sheets of Area of each plate, d → distance between the
1mm thickness as dielectric. If the dielectric plates) is :
constant of paper is 4, the number of circular
metal foils of diameter 2 cm each required for
the purpose is
(a) 40 (b) 20
(c) 30 (d) 10
VITEEE-2007 3 ε0 A 2ε 0 A
(a) (b)
Ans. (d) : The capacitance (C) of a parallel plate 2 d d
capacitor with dielectric is given by 2 ε0 A 3ε 0 A
(c) (d)
εo A 3 d d
C=
 1 UP CPMT-2012
d − t 1 − 
 K Karnataka CET-2016
Objective Physics Volume-III 818 YCT
Ans. (c): 42. In a parallel plate capacitor of capacitance C, a
metal sheet is inserted between the plates,
parallel to them. The thickness of the sheet is
half of the separation between the plates. The
capacitance now becomes :
(a) C/4 (b) C/2
(c) 2C (d) 4C
Capacitance of each capacitor, Karnataka CET-2001
εo A Ans. (c) : We know that,
C=
d εA
Q C= o
d
With dielectric k is filled partially between plates,
εo A
C1 =
d−t+ t
k
If a dielectric slab of dielectric constant k and thickness
Capacitor across V and Y are in parallel, t is inserted in the air gap of a capacitor of plate
C1 = C + C = 2C separation d and plate area A, its capacitance becomes.
Q Capacitor C and C1 will be in series, For metal sheet,
d
1 1 1 1 1 3 t = ,k =∞
= + = + = 2
Ceq C1 C 2C C 2C
εA
2C ∴ C2 = o
Ceq = d−t
3 εA  d
= o Q t = 2 
2 εo A d
Ceq = d−
3 d 2
41. A capacitor and an inductance coil are εA
C2 = 2 o
connected in separate AC circuits with a bulb d
glowing in both the circuits. The bulb glows C2 = 2C
more brightly when: 43. A capacitor of capacitance C1 is charged to a
(a) an iron rod is introduced into the inductance potential V and then connected in parallel to an
coil uncharged capacitor of capacitance C2. The
final potential difference across each capacitor
(b) the number of turns in the inductance coil is will be
increased C1V C2 V
(c) separation between the plates of the capacitor (a) (b)
C1 + C 2 C1 + C 2
is increased
C C
(d) a dielectric is introduced into the gap between (c) 1 + 2 (d) 1 − 2
the plates of the capacitor C1 C1
Karnataka CET-2010 J&K CET- 2011
Ans. (d) : Ans. (a) : The common potential difference across two
capacitors connected in parallel,
C V + C 2 V2
Veq = 1 1
C1 + C 2
Here, potential of charged capacitor,
Potential of uncharged capacitor (V2) = 0
C1V
∴ Veq =
C1 + C 2
Resistance of bulb = R 44. n identical capacitors each of capacitance C
V when connected in parallel give the effective
Q Current (i) =
R 2 + C2 capacitance 90 µF and when connected in series
give 2.5 µF. Then the values of n and C
If a dielectric is introduced into the gap between the respectively are
plates, of capacitor, its capacitance will increase and (a) 6 and 15 µF (b) 5 and 18 µF
hence impedance of the circuit will decrease.
(c) 15 and 6 µF (d) 18 and 5 µF
Thus, current and hence brightness of the bulb increase J&K CET- 2010
Objective Physics Volume-III 819 YCT
Ans. (a) :Given that, Ans. (b): Two sheets of metal 7 m × 15 m,
When connected in parallel Separated by a layer of air (d) = 0.5 cm
( Ceq ) = 90 µF
1
= 0.5 × 10–2 m
Q Capacity of a parallel plate capacitor,
‘n’ identical capacitance in parallel, A
( Ceq ) = C1 + C2 + ........Cn .....(i) C = εo
d
( C ) = nC ⇒ 90 = nC
eq =
8.85×10−12 ×(7 ×15)
90 0.5×10−2
n= ....(ii) = 185.85 × 10–9 F
C C = 0.185 × 10–6 F
When ‘n’ identical capacitor connected in series,
C ≃ 0.2 µF
1 n
= 47. The insulated plates of a charged parallel plate
Ceq C capacitor (with small separation between the
C plates) are approaching each other due to
Ceq = electrostatic attraction.
n
Assuming no other force to be operative and no
C
2.5 µF = radiation taking place, which of the following
n graphs approximately shows the variation with
C = 2.5 × n time (t) of the potential difference (V) between
90 the plates?
C = 2.5 × [from equation (ii)]
C
2
C = 225 (a) (b)
C = 15 µF
Putting C = 15 µF in equation (i) we get –
n × 15 = 90
n=6
45. If eight similar charge drops combine to form a (c) (d)
bigger drop, then the ratio of capacitance of
bigger drop to that of smaller drop will be
(a) 2 : 1 (b) 8 : 1
(c) 4 : 1 (d) 16 : 1 WB JEE 2018
J&K CET- 2005 Ans. (a) : As separation between the plates are
Ans. (a) : Let ‘R’ be the radius of big drop and ‘r’ be decreasing as they approach each other,
the radius of small drop.
Q Volume of bigger drop = 8 × volume of small drop
4 3 4
πR = 8 × πr 3
3 3
R = 2r
Capacitance of bigger drop.
C = 4πε0R
Capacitance of small drop. V = E.d
C' = 4πε0r Electric field remains constant between the plates,
C ' 4πε 0 r r V∝d
∴ = = Q2
C 4πε 0 R R Force on each plate (F) = = ma
2A ∈0
C' r 1
= = Q2
C 2r 2 Acceleration (a) =
C : C' = 2 : 1 2A ∈0 (m)
46. A parallel plate capacitor is constructed of two 1
sheets of metal 7 ṁ by 15 m, and separated by a Where, d = d 0 − at 2 (Q Plates are moving)
2
layer of air 0.5 cm. The capacitance of this
 1 
system is ∴ V = E d o − at 2 
(a) 0.1 µF (b) 0.2 µF  2 
(c) 2 µF (d) 1.2 µF This is a equation of a parabola with downward
J&K CET- 1997 concavity.

Objective Physics Volume-III 820 YCT


48. Consider the circuit given here. The potential Ans. (c) :
difference VBC between the points B and C is

Equivalent capacitance of the series combinations,


(a) 1 V (b) 0.5 V 1 1 1 3
(c) 0 V (d) –1 V = + =
Ceq 5 10 10
WB JEE 2017
10
Ans. (b) : Ceq = µF
3

Hence, the energy stored,


1
Current flowing in the circuit, E = Ceq V 2
V 3 3 2
I= = = 1 10 2
R 1 + R 2 + R 3 (1 + 2 + 3) 6×103 E = × ×10−6 ×(300)
2 3
I = 0.5 × 10–3 A E = 0.15 J
Potential drop across the arm AD,
50. 64 identical spheres of charges q and
VAD = IR = 0.5 × 10–3 × 3 × 103 = 1.5 V capacitance C each are combined to form a
1 1 1 3 large sphere. The charge and capacitance of the
= + =
Ceq 1 2 2 large sphere is
(a) 64 q, C (b) 16q, 4C
2
Ceq = µF (c) 64q, 4C (d) 16q, 64C
3 WB JEE 2008
Charge (Q) = VC Ans. (c) : Volume of large sphere = volume of each
2 small sphere
= 1.5× = 1 µC
3 4 3 4 
πR = n  πr 3 
Apply KVL (from B to C) 3 3 
1 R3 = 64 r3
VB – I×2×103 + = VC
2 R = 4r
1 Q Qtotal = 64 q
VB – 0.5×10 –3 ×2×103 + = VC {∴ I = 0.5 × 10–3A}
2 C = 4πεor
1 Now, C' = 4πεoR
VB – VC = 1 – = 0.5V Putting the value of R = 4r
2
Hence, Potential difference VBC between the points B C' = (4πεo) 4r
and C is 0.5V. C' = 4C
49. A 5 µF capacitor is connected in series with a 51. The capacity an isolated conducting sphere of
radius R is proportional to
10 µF capacitor. When a 300 V potential
difference is applied across this combination, 1
(a) R2 (b)
the total energy stored in the capacitors is R2
(a) 15 J (b) 1.5 J 1
(c) (d) R
(c) 0.15 J (d) 0.10 J R
WB JEE 2015 UP CPMT-2005
Objective Physics Volume-III 821 YCT
Ans. (d): Capacitance of the conductor, 54. A bulb and capacitor are connected in series to
Q a source of alternating current. If frequency is
C= increased, while keeping the voltage of the
V
source constant, then
Potential on sphere of radius, R
(a) Bulb will give more intense light
[Where q is the charge and V is the potential]
(b) Bulb will give less intense light
1 Q
V= (c) Bulb will give light of same intensity as
4πε o R before
4πε o R (d) Bulb light will fluctuate
∴ C=Q×
Q TS EAMCET 30.07.2022, Shift-I
C = 4πεoR Ans. (a) : When a bulb and a capacitor are connected in
C∝R series to an AC source, then on increasing the
52. A spherical capacitor consists of two concentric frequency, the current in the circuit is increased,
spherical conductors. Find the capacitance of because the impedance of the circuit is decreased. So,
the spherical capacitor if the outer radius is 2R the bulb will give more intense light.
and the inner radius is R. 55. Sixty four water drops having equal charges
(a) 4πεoR (b) 8πεoR combine to form one bigger drop. The
8πεo 4πε o capacitance of bigger drop as compared to that
(c) (d)
R R of smaller drop will be
TS EAMCET 04.08.2021, Shift-I (a) 4 times (b) 8 times
Ans. (b) : Capacitance of spherical capacitor, (c) 16 times (d) 64 times
 ab  SRMJEEE - 2016
C = 4πεo  
 b−a  Ans. (a) : Volume of bigger drop = Volume of each
 R.2R  2R 2 small drop
∴ C = 4πεo   = 4 πε o 4 3 4 
 2R − R  R πR = 64 ×  πr 3 
C = 8πεoR 3 3 
53. Consider a spherical drop of mercury of radius R3 = 64 r3
R with capacitance C = 4π∈0R. If two such R=4r
droplets combine to form a larger one, what Now, C = 4πεor
would be its capacitance in terms of C? ∴ C' = 4πεοR
(a) 31/ 3 C (b) 32 / 3 C C' = 4πεo4R
(c) 2 2 / 3 C (d) 21/ 3 C C' = 4C
TS EAMCET 04.08.2021, Shift-II 56. A parallel plate capacitor has a plate area of 50
Ans. (d) : cm2 and plate separation of 1.0 cm. A potential
difference of 200 volt is applied across the
plates with air as the dielectric between plates.
The battery is then disconnected and a piece of
Bakelite of dielectric constant 4.8 inserted
For spherical drop of mercury, which fills the complete volume between the
Radius = R plates. The capacitance before and after
Capacitance (C) = 4πεoR inserting Bakelite are respectively.
When two droplets combine, (a) 44 pF; 211.2 pF (b) 4.4 pF; 211.2pF
r = smaller radius (c) 4.4 pF; 21.12pF (d) 21.12 pF; 44pF
Q Volume of bigger drop = volume of each small drop MP PET -2013
4 4 Ans. (c) :
π ( R' ) = 2 × πr 3
3

3 3
1
R' = 2 3 r
∴ Capacitance of bigger drop,
C' = 4πεoR'
1
C' = 4πεo × 2 3 r
1
C' = 2 3 C {Q C = 4πε 0 r} Area of plate A = 50 cm2
A = 50 × 10–4 m2
Objective Physics Volume-III 822 YCT
Distance between the plates, Ans. (a): We know that,
d = 1 cm = 0.01 m εA
C= 0
Dielectric constant for air (K) = 1 d
KAε o 1× 50 × 10 −4 × 8.854 ×10 −12 ε  3A 
C= = ∴ C1 = 0  
d 0.01 d 4 
C = 4.4 × 10–12 = 4.4 pF 3ε A
Dielectric constant for Bakelite, K = 4.8 C1 = 0
4d
4.8 × 50 × 10 −4 × 8.854 × 10−12 When dielectric constant introduce between plates,
C=
0.01 ε KA
C2 = 0  
C =21.12 × 10–12 = 21.12 pF d 4
57. In given circuit when switch S has been closed ε AK
then charge on capacitor A & B respectively C2 = 0
4d
∴ C' = C1 + C2
3ε A ε AK
C' = 0 + 0
4d 4d
ε0 A  3 K 
C' = +
d  4 4 
(a) 3q, 6q (b) 6q, 3q C
(c) 4.5q, 4.5q (d) 5q, 4q C' = ( K + 3)
4
MP PET-2008
59. Taking the Earth to be a spherical conductor of
Ans. (b) : Capacitor A and B are connected in parallel diameter 12.8 × 103 km. Its capacity will be
so, potential difference between them will be equal (a) 711µF (b) 611µF
So, VA = VB …..(i) (c) 811µF (d) 511µF
And QA + QB = 9q JIPMER-2004
When switch is closed Ans. (a) : Given,
QA' + QB' = 9q …..(ii) Diameter of the Earth, D = 12.8 × 103 km
Q VA = VB 12.8
QA ' Q B ' So, radius, R = × 103 km
∴ = 2
CA CB R = 6.4 × 103 km = 6.4 × 106 m
QA ' Q B ' We know that,
= [Q CA = 6C, CB = 3C] Capacity, C = 4πε0 R
6C 3C
1
QA' = 2QB' …..(iii) C= × 6.4 ×106
From equation (ii) and (iii) 9 × 109
QA' = 6q, QB' = 3q C = 711 × 10–6 F
C = 711 µ F
58. A parallel plate capacitor with air as the
dielectric has capacitance C.A slab of dielectric 60. In a parallel plate capacitor, if 1012 electrons
pass from one plate to another, a potential
constant K and having the same thickness as
difference of 10 V is developed across the
the separation between the plates is introduced plates. The capacitance of the capacitor is
so as to fill one-fourth of the capacitor as (a) 0.16×10–8F (b) 1.6×10–8F
shown in the figure. The new capacitance will (c) 16×10 F –8
(d) 0.8×10–8F
be AP EAMCET-07.07.2022, Shift-I
Ans. (b) : Given that,
Charge, Q = ne = 1 × 1012 × 1.6 × 10–19
Q = 1.6 × 10–7 C
Potential difference, V = 10 V
C C We know that,
(a) ( K + 3) (b) ( K + 2) Q
4 4 ∴ C=
C KC V
(c) ( K + 1) (d) 1.6 × 10 −7
4 4 C= = 1.6 ×10 −8 F
JIPMER-2012 10

Objective Physics Volume-III 823 YCT


61. In the given figure, the capacitors C1, C3, C4, C5 Ans. (b) : Given,
have a capacitance 4 µF each. If the capacitor Thickness, t = d/3
C2 has a capacitance 10 µF, then effective Capacity, C' = 4/3 C
capacitance between A and B will be We know that,
Aε o
Cair =
d
Aε o 4
C' = = C
t 3
(d − t ) +
K
Aε o 4 Aε o
(a) 2 µF (b) 6 µF =
t 3 d
(c) 4 µF (d) 8 µF (d − t ) +
Karnataka CET - 2004,2018 K
JIPMER-2016 4t
∴ K=
AIIMS-2002 4t − d
AIIPMT-1999 d
APEAMCET-08.07.2022 (Shift-II) 4×
K= 3
Ans. (c) : The given network can be replaced, 4×d
−d
C 4 C1 3
=
C 3 C5 4d
No charge flow through C2. So, it is not considered –
K= 3
Hence, capacitors C1 and C5 are in series. d
3
4d
K=
d
K=4
63. A capacitor of capacitance 4 µF is charged to a
potential of 100 V. It is then disconnected from
the battery and connected in parallel with
1 1 1 another capacitor C2. If their common
= + potential is 40 volts, then the value of C2 is
C' C1 C5
_____
C1C5 (a) 2 µF (b) 3 µF
C' =
C1 + C5 (c) 5 µF (d) 6 µF
4× 4 AP EAMCET-03.09.2021, Shift-I
C' =
4+4 Ans. (d) : Given that, C = 4µF , V = 100 V, V ' = 40V
1
C' = 2 µF Charge on first capacitor,
Similarly C4 and C3 are in series. Q = C 1V
Therefore, Q = 4 × 10–6×100
CC 4× 4 Q = 400×10–6
C '' = 3 4 = = 2µF
C3 + C 4 4 + 4 Q
∴ C1 + C2 =
Also, C' and C" are in parallel configuration. V'
Hence, effective capacitance between A and B is, 400 ×10 −6
CAB = 2+2 = 4 µF 4µF + C 2 =
40
62. Capacity of a parallel plate capacitor becomes
C2 = (10 × 10 ) – (4 × 10–6)
–6
[Qµ = 10–6]
4 −6
times its original value when a dielectric slab C2 = 6 × 10 F
3 C2 = 6 µF
d
of thickness t= is inserted between the plates 64. If the circumference of a sphere is 3 m, the
3 capacitance of the sphere in water is _____
of the capacitor, of separation ‘d’. Dielectric (dielectric constant of water = 80)
constant of the slab is _______. (a) 4250 pF (b) 2760 pF
(a) 8 (b) 4 (c) 2780 pF (d) 424 pF
(c) 6 (d) 2 AP EAMCET-25.08.2021, Shift-II
AP EAMCET-07.09.2021, Shift-I
Objective Physics Volume-III 824 YCT
Ans. (a): Given, k = 80 ER1
V=
L = 2πr = 3m r + R1
3 Thus, charge on capacitor (Q) = CV
r=
2π ER 1
The capacitance of the sphere in water is, Q = C×
r + R1
C = 4πε0kr
CER 1
Q=
3 R1 + r
C = 4π × 8.85 × 10–12 × 80 ×
2π 67. A parallel plate capacitor is charged by
C = 4248 × 10–12 F  4250 pF connecting a 2V battery across it. It is then
C = 4250 pF disconnected from the battery and a glass slab
65. A parallel plate capacitor has a capacity C. If a is introduced between plates. Which of the
thin metal plate (M) joins the two coatings A following pairs of quantities decrease?
and B of the capacitor, its new capacitance is (a) Energy stored and capacitance
(b) Capacitance and charge
(c) Charge and potential difference
(d) Potential difference and energy stored
Karnataka CET-2022
Ans. (d) : When battery is disconnected, charge remains
constant. On introducing glass slab, capacity increases.
C Potential difference and energy stored decreases.
(a) 2C (b)
2 68. Two tiny spheres carrying charges 1.8 µC and
(c) 0 (d) ∞ 2.8 µC are located at 40 cm apart. The
AP EAMCET-05.10.2021, Shift-I potential at the mid-point of the line joining the
Ans. (d) : The thin metal plate join two parallel plate two charges is
capacitor of capacitance C. (a) 4.3 × 104V (b) 3.6 × 105V
4
Since, thin metal plate (M) joins the two coatings A and (c) 3.8 × 10 V (d) 2.1 × 105 V
B of the capacitor, charges gets neutralized and both the Karnataka CET-2022
plates acquire same potential. Therefore, potential Ans. (d) : Given,
difference V = 0
Charges, q1 = 1.8 µC = 1.8 × 10–6 C
Q Q
So, C = = = ∞ q2 = 2.8 µC = 2.8 × 10–6 C
V 0 Distance between two sphere, d = 40 cm = 0.4 m
Thus, capacitance becomes infinite and can hold any
0.4
amount of charge. For mid-point, r1 = r2 = = 0.2 m
2
66. In the given circuit, the charge on the capacitor
is Potential (V) = V1 + V2
1  q1 q 2 
V = +
4 πε0  r1 r2 
1 1.8×10−6 2.8×10−6 
V= +
4πε 0  0.2 0.2 
1  (1.8 + 2.8)×10 
−6

V=
(a) CE (b)
CER 1 4πε 0  0.2 

R1 + r
9 ×109 ×(1.8 + 2.8)×10−6
CER 2 CER1 V = = 2.1 × 105 V
(c) (d) 0.2
R1 + r R2 + r
69. If combining two charged bodies, the current
AP EAMCET-25.04.2017, Shift-II does not flow then
Ans. (b) : Current through the battery, (a) charge is equal on both
E (b) capacitance is equal on both
i=
+
r R1 (c) potential is equal on both
Potential difference across R1, (d) resistance is equal on both
V = iR1 BITSAT-2009

Objective Physics Volume-III 825 YCT


Ans. (c): The current means the charge will flow from 1  Q1 Q 2 
one region to other region due to the potential VB =  + 
difference between two regions so if the potential is 4πε o  R 1 R 2 
equal on both, the current will not flow. Potential difference, VB − VA
70. 2µF capacitor is connected with 50V supply & 1  Q1 Q 2  1  Q1 + Q 2 
3µF capacitor is connected with 100V supply. VB − VA =  + −  
4πε o  R1 R 2  4πε o  R 2 
Now after removing battery if two plates of
same type of charges are placed to form new 1  1 1 
capacitor then potential difference is _____V. VB − VA = Q1  − 
4πε o  R 1 R 2 
(a) 80 (b) 333
(c) 200 (d) 75 Therefore Assertion and reason are correct and the
reason is a correct explanation of the assertion.
GUJCET 2020
72. Two spherical conductors A and B of radii a
Ans. (a) : Given, C1 = 2 µF, V1 = 50 V, C2 = 3 µF, V2 =
and b (b > a) are placed concentrically in air.
100 V.
The two are connected by a copper wire as
Potential difference of new capacitor is, shown in figure. Then the equivalent
Q + Q2 capacitance of the system is
VC = 1
C1 + C 2
C1V1 + C 2 V2
VC =
C1 + C 2

VC =
( 2 × 50 ) + ( 3 ×100 )
2+3
400
VC =
5
VC = 80 volts ab
71. Assertion : Two concentric charged shells are (a) 4πε 0 (b) 4πε0 (a + b)
b−a
given. The potential difference between the shells
(c) 4πε0b (d) 4πε0a
depends on charge of inner shell.
AIIMS-2017
Ans. (c) : All the charge given to inner sphere will pass
on to the outer one, so capacitance that of outer one is
4πεob
73. If the separation between the plates of a
capacitor is 5 mm. then the area of the plate of
a 3F parallel plate capacitor is :
Reason : Potential due to charge of outer shell (a) 4.259 × 109m2 (b) 1.964 × 109m2
9 2
remains same at every point inside the sphere. (c) 12.81 × 10 m (d) 1.694 × 109m2
(a) If both Assertion and Reason are correct and AIIMS-1998
the Reason is a correct explanation of the Ans. (d) : Given, The separation between the plates,
Assertion d = 5 mm = 5 × 10−3 m, C = 3F
(b) If both Assertion and Reason are correct, but The Expression of parallel plate capacitor is given by ,
Reason is not a correct explanation of the Aεo
Assertion C=
d
(c) If Assertion is correct but Reason is incorrect
So, area of plate,
(d) If both the Assertion and Reason are
incorrect. Cd 3 × 5 × 10−3 2
A= = m
AIIMS-2010 ε o 8.85 × 10 −12
Ans. (a) : A = 1.694 × 10−3 × 1012
A = 1.694 × 109 m2
74. An air parallel plate capacitor has capacity C.
The capacity and distance between plates are
doubled when immersed in a liquid then
dielectric constant of the liquid is :
(a) 1 (b) 2
1 Q1 + Q 2
VA = (c) 3 (d) 4
4πε o R 2 BCECE-2004
Objective Physics Volume-III 826 YCT
Ans. (d) : Given, C' = 2C 77. A pair of parallel metal plates are kept with a
Capacity of air parallel plate capacitor, separation d. One plate is at a potential + V
Aε and the other is at ground potential. A narrow
C= o beam of electrons enters the space between the
d
plates with a velocity v0 and in a direction
When immersed in liquid, then distance is doubled
parallel to the plates. What will be the angle of
K A εo
C' = the beam with the plates after it travels an axial
2d distance L?
or,
 eVL   eVL 
K (a) tan–1   (b) tan–1  2 
2C = C  mdv 0   mdv 0 
2
K=4  eVL   eVL 
(c) sin–1   (d) cos–1  2 
75. The capacitance of a spherical condenser is
 mdv 0   mdv 0 
1µF. If the spacing between the two spheres is 1
mm, the radius of the outer sphere is WB JEE 2020
(a) 30 cm (b) 6 m Ans. (b) : Distance between the parallel plates = d
(c) 5 cm (d) 3 m The axial distance of beam form the centre of parallel
AP EAMCET (18.09.2020) Shift-II plates = L
Ans. (d) : Given, C = 1 µF = 1 × 10−6 F and R2 − R1 = 1 Time (t) =
distance
=
L
mm = 1 × 10−3 m, R2 = ? Velocity v0
Capacity of spherical capacitor bounded by two sphere
is given by
4πε 0 R 1R 2
C= ........ (i)
(R 2 − R 1 )
4πε o R 1R 2
1×10 −6 =
1 ×10 −3
1
R 1R 2 = × 10 −9
4πε o
1 Component of velocity,
As we know, R1 ≈ R2 and = 9×109
4πε o  qE 
Vy =  t
So, R 22 = 9×109×10–9 m
R 22 = 9 e V L  V 
= × × Q E = , q = e 
R2 = 3 m m d vo  d 
76. A parallel plate capacitor of capacitance 5 eVL
Vy =
micro farad is charged to 120 V and then mdv 0
connected to another uncharged capacitor. If
the potential falls to 40 V, the capacitance of tan θ = Vy = eVL = eVL (where, Vx = vo)
the second capacitor is Vx mdv o .Vx mdv o2
(a) 5 micro farad (b) 10 micro farad
 eVL 
(c) 15 micro farad (d) 20 micro farad θ = tan-1  2 
J&K-CET-2013  mdv o 
Ans. (c) : Given, C1 = 5µF = 5 × 10−6 F, V = 120 V 78. A parallel plate capacitor in series with a
Q Q = CV resistance of 100 Ω, an inductor of 20 mH and
−6
Q = 5 × 10 × 120 an AC voltage source of variable frequency
After disconnection charge will remains same 1250
shows resonance at a frequency of Hz. If
Let capacitance on second capacitor is C2. π
5 × 10−6 × 120 = 40 × C2 this capacitor is charged by a DC voltage
5×120 ×10−6 source to a voltage 25 V, what amount of
C2 = charge will be stored in each plate of the
40 capacitor?
−6
C2 = 15 × 10 = 15 µF (a) 0.2 µC (b) 2 mC
or (c) 0.2 mC (d) 0.2 C
C2 = 15 micro farad.
WB JEE 2019
Objective Physics Volume-III 827 YCT
Ans. (c): Given, L = 20 mH = 20 × 10−3H, V = 25 V (Cp is equivalent capacitance of capacitor in Parallel)
1250 From (i) & (ii)
f = Hz C1 C2 = 3 ×16 = 48 …….(iii)
π From (ii) & (iii)
1
We know that, ω= C1 +
48
= 16
LC C1
1 C12 – 16 C1 + 48 = 0
2πf =
LC C12 – 12 C1– 4C1 + 48 =0
Squaring both side, C1 (C1 – 12) – 4 (C1 – 12) = 0
1 (C1 – 12) (C1 – 4) = 0
4π 2 f 2 = C1 = 12 µF, 4 µF
LC
From (iii)
1
C= 12 × C2 = 48
4π f L
2 2
48
1 C2 =
C= 12
2
 1250  −3
=4µF
4π 2 ×   × 20 ×10 C1 = 12µF C2 = 4 µF
 π 
C = 8 × 10–6 F 81. The distance between the two plates of a
parallel plate capacitor is doubled and the area
∴ For a DC Source of 25 V of each plate is halved. If C is initial
Stored charge, q = CV = 8× 10–6×25 capacitance, its final capacitance is equal to:
= 0.2 mC (a) 2 C (b) C/2
79. Four capacitors of equal capacitance have a net (c) 4C (d) C/4
capacitance C1 when connected in series and a Ans. (d) : Capacitance,
net capacitance C2 when connected in parallel.
εA
The ratio of C1/C2 is C= 0
1 1 d
(a) (b) Area (a) = A/2
4 16 Diameter (d) = 2d
1 1 εA
(c) (d) Then, C ' = 0
8 12 2 × 2d
WB JEE-2007 εA C
Ans. (b) : Let capacitance of each capacitor is c C' = 0 =
4d 4
In series, net capacitance, C
'
C =
C
C2 = 4
4 82. A slab of dielectric constant K has the same
In Parallel, net capacitance cross-sectional area as the plates of a parallel
C2 = 4C 3
C plate capacitor and thickness d, where d is
The ratio of 1 is, 4
C2 the separation of the plates. The capacitance of
C1 C 1 the capacitor when the slab is inserted between
= = the plates will be:
C 2 4 × 4C 16 (Given C0 = capacitance of capacitor with air as
C1 1 medium between plates.)
=
C 2 16 4KC0 3KC 0
(a) (b)
80. The effective capacitances of two capacitors are 3+ K 3+ K
3µF and 16µF, when they are connected in 3+ K K
(c) (d)
series and parallel respectively. The 4KC0 4+K
capacitance of two capacitors are: JEE Main-28.07.2022, Shift-II
(a) 10 µF, 6 µF (b) 8 µF, 8 µF Ans. (a) : Given that,
(c) 12 µF, 4 µF (d) 1.2 µF, 1.8 µF
3
Ans. (c) : Given, Thickness of slab (t) = d
4
C1C 2 The capacitance of a parallel plate capacitor in the
Cs = = 3µF .....(i)
C1 + C 2 absence of the dielectric is-
(Cs is equivalent capacitance of capacitor in series) εA
C0 = 0
Cp = C1 + C 2 = 16 µF …..(ii) d
Objective Physics Volume-III 828 YCT
If a dielectric slab is partially filled between the plates Ans. (a):The capacitance of the air filled parallel plate
of capacitor, then capacitor is,
ε0 A εA
C= C= 0 .....(i)
 t d
d − t + K  New capacitance is given by,
ε0 A ε0 A
C= C' =
 3   3d   1
d − t 1 − 
d − d +    K
 4   4K 
d
ε0 A After inserting metal sheet of thickness ,
C= 2
d 3d
+ d
4 4K t= , K=∞ (for metal)
2
4Kε 0 A
C= or,
1
=0
d(K + 3) K
C 4Kε 0 A d ε A 2ε A
∴ = × C' = 0 = 0 .....(ii)
C0 d(K + 3) ε 0 A d d
d−
4KC0 2
C= Hence, from equation (i) & (ii), we get-
(3 + K )
2ε 0 A
83. A force of 10 N acts on a charged particle C' 2
placed between two plates of a charged = d =
C ε0 A 1
capacitor. If one plate of capacitor is removed,
then the force acting on that particle will be d
(a) 5 N (b) 10 N =2:1
(c) 20 N (d) Zero 85. A composite parallel plate capacitor is made up
JEE Main-27.06.2022, Shift-I of two different dielectric materials with
different thickness (t1 and t2) as shown in
Ans. (a) : Given, figure. The two different dielectric material are
F = 10N separated by a conducting foil F. The voltage of
We know that, F= qE the conducting foil is _______V.
 Q  qQ
q =
 Aε 0  Aε 0
qQ
10 = ….(i)
Aε 0
Now when one plate of capacitor is removed JEE Main-26.07.2022, Shift-I
Q Ans. (60) : Capacitance of each capacitor
E′ = A3ε 0
2Aε o C1 = = 6Aε 0
1
 Q  1 qQ
F = qE′ = q  = 2
 2Aε 0  2 Aε 0 C 2 = A4ε 0 = 4Aε 0
1 Equivalent capacitance,
F = × 10 [(From (i)]
2 C1C 2 24
F = 5N Ceq = = Aε 0
C1 + C 2 10
84. Two metallic plates form a parallel plate qent = Ceq (∆V) = 240 Aε0
capacitor. The distance between the plate is 'd'.
240Aε 0
d ∆V 2 = = 60 V
A metal sheet of thickness and of area equal 4Aε 0
2
to area of each plate is introduced between the (∆ V2 is potential drop cross C2)
plates. What will be the ratio of the new Vfoil = 60 V
capacitance to the original capacitance of the 86. A parallel plate air capacitor has a capacitance
capacitor? of 8pF. If the separation between the plates is
(a) 2:1 (b) 1:2 reduced by half and the space is filled up with a
(c) 1:4 (d) 4:1 material of dielectric constant 6, then its
JEE Main-25.06.2022, Shift-II capacitance will be.
Objective Physics Volume-III 829 YCT
(a) 69pF (b) 86pF Then, the capacitance of the dielectric filled capacitor
(c) 68 pF (d) 96pF will be C2 = K C
Assam CEE-2021 Q + Q2
Now common potential is V2 = 1
Ans. (d) : Given capacitance of capacitor, C1 + C 2
εA Here,
C= 0 C = 8pF
d Q1 = C1V1 charge for air filled capacitor is
εA charged.
8= 0 Q2 = 0, for uncharge capacitor
d
C1V1
ε KA ε 0 × 6 × A 12Aε 0 V2 =
Now, C ' = 0 = = C1 + C 2
d' d/2 d
C′ = 12×8×10–12 = 96 pF CV1
So, V2 =
C′ = 96 pF C + KC
87. Three infinite long plane sheets of uniform V
K +1 = 1
charge densities σ1 = –5σ, σ 2 = +2σ and V2
σ1 = 3σ are placed parallel to each other as V1 − V2
K=
shown in the figure. The electric field at point P V2
will be 89. A parallel plate capacitor consists of circular
plates each of radius 10 cm separated by 2 mm.
If the charging current be 0.2 A. What is the
rate of variation of potential?
(a) 1.44 × 109 V.s–1 (b) 2.88 × 109 V.s–1
9 –1
(c) 0.72 × 10 V.s (d) 0.36 × 109 V.s–1
AP EAMCET-25.09.2020, Shift-I
Ans. (a) : Given,
r = 10 cm = 10 × 10−2 m
3σ d = 2 mm = 2×10–3 m
(a) Zero (b) − ĵ Capacitance blow the two plates is given by,
ε0
εA
5σ 7σ C= 0 , A = πr 2
(c) − ĵ (d) − ĵ d
ε0 2ε 0
ε πr 2 8.85 × 10 × π × (10 ×10 )
−12 −2 2
Assam CEE-2016
C= 0 =
Ans. (a) : Electric field at point P = d 2 ×10 −3
σ σ σ  8.85 ×10 −12 × 3.14 ×100 ×10 −4
=  1 + 2 + 3 × −k̂ ( ) =
 2ε 0 2ε 0 2ε 0  2 × 10−3
 −5σ 2σ 3σ  C = 13.8 945× 10–11F
=  + + ( )
× −k̂ Charge of each plate q = CV
 2ε 0 2ε 0 2ε 0  Differentiate on both side w. r. t. time, we get
= 0 dq dV
=C
88. An uncharged parallel plate capacitor filled dt dt
with an oil is connected in parallel with an dq
identical but air filled capacitor charged to a Q = current(I)
dt
potential V1. If the common potential is V2, the
dV I 0.2
dielectric constant of the oil is ∴ = =
dt C 13.8945 × 10 −11
V – V1 V – V2
(a) 2 (b) 1 = 0.0143 × 1011
V1 V2
= 1.43 × 109 V/s
V2 – V1 V2 + V1 90. If a capacitor having capacitance 2F and plate
(c) (d)
V2 + V1 2V1 separation of 0.5 cm will have area
AP EAMCET-25.04.2018, Shift-I (a) 1130 cm2 (b) 1130 m2
2
Ans. (b) : If the capacitance of the air filled capacitor is (c) 1130 km (d) None of these
C1 = C , JIPMER-2018

Objective Physics Volume-III 830 YCT


Ans. (c): Given, 93. A parallel plate capacitor of capacitance 20µF
C = 2F is being charged by a voltage source whose
d = 0.5 cm = 0.5 × 10–2m potential is changing at the rate of 3 V/s. The
conduction current through the connecting
We know that,
wires and the displacement current through the
Capacitance of a parallel plate, plates of the capacitor, would be, respectively.
εA (a) 60µA, 60µA (b) 60µA, zero
C= o
d (c) zero, zero (d) zero, 60µA
Cd 2 × 0.5 × 10−2 NEET- 2019
A= = Ans. (a) : Given,
εo 8.85 × 10 −12
Capacitance of capacitor (C) = 20 µF
= 1.130 × 109 m2 = 20×10–6 F
A = 1130 km2 dV
91. Two capacitors of 10µF and 20µF are Rate of change of potential, = 3V / s
dt
connected in series with a 30 V battery. The q = CV
charge on the capacitors will be respectively dq dV
(a) 100 µC, 100 µC (b) 200 µC, 100 µC =C
dt dt
(c) 200 µC, 200 µC (d) 100 µC, 200 µC Ic = 20 × 10–6 × 3
AMU-2001 = 60 × 10–6 A
Ans. (c) : In series combination of capacitor, charge on Ic = 60 µA
each capacitor is Same. As we know that, Id = Ic = 60 µA
1 1 1 94. A capacitor of capacitance C, is connected
= + across an AC source of voltage V, given by
Ceq C1 C 2
V = V0 sinωt
C1C 2 The displacement current between the plates of
Ceq =
C1 + C 2 the capacitor, would then be given by
Q1 = Q2 = Q = Ceq V V
(a) I0 = V0 ωC cos ωt (b) Id = 0 cos ωt
 10 × 20  ωC
Q = Ceq V =   × 30 V0
 10 + 20  (c) Id = sin ωt (d) Id = V0 ωC sin ωt
200 ωC
Q= × 30 × 10 −6 = 200 µC NEET- 2021
30
Ans. (a) : Given,
So, charge on each capacitor will be 200 µC. V = Vo sin ωt .....(i)
Now displacement current Id is given by,
92. The capacitance of a parallel plate capacitor Q q = CV
with air as medium is 6 µF. With the dq dV
introduction of a dielectric medium, the =C
dt dt
capacitance becomes 30µF. The permittivity of dV d
the medium is (ε0= 8.85 × 10–12 C2 N–1 m–2) or Id = C = C (Vo sin ωt )
dt dt
(a) 1.77 × 10–12 C2 N–1m–2
= C (V0 ω) cos ωt
(b) 0.44 × 10–10 C2 N–1m–2
(c) 5.00 C2 N–1m–2 Id = Vo ωCcos ωt
(d) 0.44 × 10 –13 C2 N–1m–2 95. The effective capacitance between A and B in
the given figure is
NEET SEP-2020
Ans. (b) : Given, Cair = 6µF, Cmedium = 30 µF
Let dielectric constant of medium be k.
We know, Cmedium = k Cair
30 = k ×6
k=5
Now, permittivity of medium, (a) 1.5 µF (b) 1 µF
µ = µ0 k (c) 3 µF (d) 2 µF
µ = 8.85 × 10–12×5 (e) 2.5 µF
= 0. 44 ×10 –10C2 N–1m–2 Kerala CEE 04.07.2022

Objective Physics Volume-III 831 YCT


Ans. (b): (a) the charge in the capacitor becomes zero
(b) the capacitance becomes infinite
(c) the charge in the capacitor increases
(d) the voltage across the plates increases
[EAMCET-1996, 1991]
Ans. (d) : When the capacitor is charged and then
disconnected, the charge across the plates remain a
constant even when the plates are separated and when
C1 , C2 and C3 in series, the equivalent capacitance in plates are separated the distance between the plates d
series combination– increases and hence the capacitance decreases.
1 1 1 1 1 1 1 3 Hence from, Q = CV,
= + + = + + = = 1µF
C x C1 C 2 C3 3 3 3 3 The voltage across the capacitor increases to maintain
Now Cx is parallel to C4, the equivalent capacitance of the charge constant.
parallel combination – 98. Two capacitors, each having capacitance 40 µF
Cy = Cx + C4 = 1 + 2 = 3µF are connected in series. The space between one
Further, Cy, 3µF, 3µF are in series the equivalent of the capacitors is filled with dielectric
capacitance is, material of dielectric constant K such that the
equivalence capacitance of the system became
1 1 1 1 3
= + + = = 1µF 24 µF. The value of K will be :
Cz 3 3 3 3 (a) 1.5 (b) 2.5
Cz = 1µF (c) 1.2 (d) 3
Now, Cz parallel to 2µF, the equivalent capacitance is, JEE Main-28.07.2022, Shift-I
Cw = Cz + 2 = 1 + 2 = 3µF Ans. (a) Given, C = 40 µF
Cw , 3µF, 3µF are in series and resultant capacitance, Capacitor is filled with dielectric slob of K.
1 1 1 1 3 Thus, equivalent capacitance,
= + + = = 1µF C ( KC )
Ceq 3 3 3 3 KC 2 KC
Ceq = = =
C + ( KC ) C (1 + K ) (1 + K )
Ceq = 1µF
96. The capacitance of two concentrie spherical KC
= 24
shells of radii R1 and R2 (R2 > R1) is 1+ K
(a) 4πεoR1 (b) 4πεοR2 KC = 24 + 24K
R 2 − R1 R 2 R1 40K = 24 + 24K (Q C = 40 µF)
(c) 4πε o (d) 4πε o 40K – 24K = 24
R 1R 2 R 2 − R1
16K = 24
AP EAMCET(Medical)-2014
24
1 K=
Ans. (d) : Let k = 16
4πε o K = 1.5
kQ 99. The capacitance of spherical conductor with
Potential at outer spherical shell, V2 =
R2 radius 1m is
kQ (a) 9 × 10−9 F (b) 10 µF
Potential at inner spherical shell, V1 = (c) 2.5 × 10−10F (d) 1.1 × 10−10 F
R1
UP CPMT-2012
Potential difference, V = V1– V2 Ans. (d) : Given,
kQ kQ r = Radius of sphere = 1 m
= −
R1 R 2 Capacitance of spherical conductor
Q 1 C = 4πε0r
Capacitance, C = = We know that,
V 1 1 
k  −  1
 R1 R 2  = 9 × 109
4πε 0
4πε o R1R 2
C= 1
(R 2 − R 1 ) 4πε 0 =
9 × 109
97. A parallel plate condenser is charged and 1 1× 10 –9
disconnected from the battery. If the plates of ∴ C= = = 0.11×10 –9 F
the capacitor are moved further apart by 9 × 10 9
9
means of insulating handles C = 1.1×10–10F

Objective Physics Volume-III 832 YCT


100. An electric circuit requires a total capacitance (a) 1.5 nF, 9 nC
of 2 µF across a potential of 1000V. Large (b) 3.0 nF, 18 nC
number of 1µF capacitances are available each (c) 1.5 nF, 4.5 nC
of which would breakdown if the potential is (d) 3.0 nF, 9 nC
more than 350 V. How many capacitances are (e) 3.0 nF, 4.5 nC
required to make the circuit? Kerala CEE-2019
(a) 24 (b) 20
Ans. (a) : Given diagram,
(c) 18 (d) 12
AMU-2012
Ans. (c) : Here,
Capacitance of each capacitor
C = 1 µF
Voltage rating of each capacitor = 350V
Supply voltage = 1000V
1nF & 2nF are in parallel & equivalent of these two are
Total capacitance = 2µF in series with 3nF,
Let, n capacitors of 1µF each be connected in series in a
∴ Ceq = (1 + 2)nF series with 3nF
row and m such rows be connected in parallel as shown
Ceq = 3nF series with 3nF
in the figure.
3× 3
∴ Ceq = = 1.5nF
3+3
Ceq = 1.5 nF
Change on circuit
Q = Ceq V = 1.5×6
= 9.0 nC
Q = 9 nC
102. A parallel plate capacitor is charged. If the
plates are pulled apart
Q Each capacitor can withstand 350V. (a) the capacitance increases
1000 (b) the potential difference increases
∴ n= = 2.8
350 (c) the total charge increases
As n cannot be fraction , therefore n = 3 (d) the charge and potential difference remain the
Capacitance of each row of 3 capacitors of 1µF each in same
series is- DCE-2009
1
Cs = µF Ans. (b) : E net = E 1 + E 2
3 Q Q Q
m = + =
∴ µF = 2µF 2Aε 0 2Aε 0 Aε 0
3
Or m = 6 V= E×d
Total number of capacitors = n × m Q
V= d
=3×6 Aε 0
= 18 Q Aε 0
101. Find the total capacitance and total charge on = =C
V d
the capacitors
Q
C=
V
Aε 0
C= .....(i)
d
d = Constant distance Increases Capacitance decreases
hence voltage should increases.
Hence, we can say that potential difference will be
Increases.

Objective Physics Volume-III 833 YCT


Ans. (5):
(b) Energy Stored in Capacitor
103. A 600 pF capacitor is charged by 200V supply.
It is then disconnected from the supply and is
connected to another uncharged 600 pF
capacitor. Electrostatic energy lost in the
process is ____µJ. Since, the surface charge density is given. So, charge on
JEE Main-08.04.2023, Shift-II spherical shell,
Ans. (6) : Given that, qx = σ 4πa2
Capacitance of capacitor (C) = 600 pF = 600 pF = 600 × qy = – σ 4πb2
10–12 F qz = σ 4πc2
C = 6 × 10–10 According to question potential on the spherical shells
Potential difference (V) = 200 V are same,
Potential (x) = Potential (z)
We know that,
Vx = Vz
Initial electrostatic energy stored in capacitor,
qx qy q qx qy q
1 + + z = + + z
( E ) = CV 2 4πε 0 a 4πε 0 b 4πε 0 c 4πε 0 c 4πε 0 c 4πε 0c
2
σ4πa 2 σ4πb 2 σ4πc 2 4σπ  a – b + c 
2 2 2
1
E = × 6 × 10−10 × ( 200 )
2
– + =
2 a b c c
1 c(a – b + c) = a2 – b2 + c2
E = × 6 × 10−10 × 200 × 200 c(a–b) = a2 – b2
2
E = 12 × 10–6 J c=a+b
c=2+3
E = 12 µJ.
c=5
When another capacitor is connected,
105. A parallel plate capacitor of capacitance 2 F is
1 1 1 charged to a potential V. The energy stored in
= +
C' C C the capacitor is E1. The capacitor is now
1 1 1 2 connected to another uncharged identical
= + = capacitor in parallel combination. The energy
C ' 600 600 600 stored in the combination is E2. The ratio E2/E1
is:
C' = 300pF = 300 × 10–12 F = 3 × 10–10 F
(a) 2 : 1 (b) 1 : 2
Final electrostatic energy stored in capacitor, (c) 1 : 4 (d) 2 : 3
1 JEE Main-11.04.2023, Shift-I
(E') = C'V 2
2 Ans. (b) : Given that, capacitor of capacitance = 2 F
1 at Initially, potential = V
E ' = × 3 × 10−10 × 200 × 200
2
Charge of capacitor
E ' = 6 × 10 −6 J Q1 = CV = (2)V
Energy stored in capacitor,
E ' = 6µJ
1 1
Hence, energy lost in the process, E1 = CV 2 = (2)V 2 = V 2 ....(i)
2 2
∆E = E – E ' at Finally,
∆E = 12 – 6 = 6µJ Q 2V
Charge on each capacitor, Q 2 = 1 = =V
104. Three concentric spherical metallic shells X, Y 2 2
and Z of radius a, b and c respectively [a < b < Then, energy stored finally in two capacitors
c] have surface charge densities σ, – σ and σ,  1 Q 22  V 2
E2 = 2 = ....(ii)
respectively. The shells X and Z are at same 2 C  2
potential. It the radii of X & Y are 2 cm and Equation (ii) divided by equation (i)
3cm, respectively. The radius of shell Z is ____ E2 1
cm. =
E1 2
JEE Main-10.04.2023, Shift-I
Objective Physics Volume-III 834 YCT
106. In the circuit shown, the energy stored in the Q Capacitors are in series,
capacitor is n µJ. the value of n is _____ CC  C2 
∴ Ceq = 1 2 =   (Q C1 = C2 = C)
C1 + C 2  2C 
Loss of energy,
1
(
∆U = Ceq V12 − V22
2
)
1 900 × 900 × 10−6 × 10−6
∆U = ×
2 ( 900 + 900 ) × 10 −6 (
100 2 − 02 )
JEE Main-13.04.2023, Shift-II
1 900 × 10−6
Ans. (75) : Current flow in 3Ω and 9Ω
12
∆U = ×
2 2
( 2
)
(100 ) − 0
I1 = = 1A ∆U = 225 × 10–2J
3+9
Thus, the value of X is 225.
Current flow in 4Ω and 2Ω
12 108. A 40 µF capacitor in a defibrillator is charged
I2 = = 2A to 3000V. The energy stored in the capacitor is
4+2 set through the patient during a pulse of
duration 2 ms. The power delivered to the
patient is :
(a) 45 kW (b) 90 kW
(c) 180 kW (d) 360 kW
Ans. (b) : Given,
C = 40 µF
V = 3000 V
t = 2 ms = 2×10–3 s
Potential difference between VA and VC The energy store in the capacitor,
VA – VC = 3×1 = 3V ……(i) 1
Potential difference between VA and VD U = CV 2
2
VA – VD = 2 × 4 = 8V ……(ii) 1
= × 40 × ( 3000 ) ×10 −6
2
Subtracting eq. (1) from eq. (2) 2
VC – VD = 5V ⇒ V = 5V = 180 J
1 1 The power delivered in 2 ms,
U = CV 2 = × 6 × 52 = 75 µJ ....(iii)
2 2 Energy delivered 180 J
P= =
According to question, Time 2 ×10−3 s
Energy stored in the capacitor, U = nµJ ....(iv) = 90×103 W
Compare the value of U from equation (iii) and (iv) = 90 kW
nµJ = 75 µJ 109. A condenser of capacitor 50 µF is charged to 10
n = 75 volt, the energy stored is
(a) 5×10-3J (b) 2.5× 10-3J
107. A capacitor of capacitance 900 µF is charged -3
(c) 3.75×10 J (d) 1.25× 10-3J
by a 100 V battery. The capacitor is AIIMS-1997
disconnected from the battery and connected to Ans. (b) : Energy stored in condenser,
another uncharged identical capacitor such 1
that one plate of uncharged capacitor U = CV 2
2
connected to positive plate and another plate of = 0.5 × 50 ×10–6×(10)2
uncharged capacitor connected to negative = 2.5 × 10–3 J
plate of the charged capacitor. The loss of 110. Two condensers, one of capacity C and the
energy in this process is measured as X × 10–2 C
J. The value of X is _____. other of capacity , are connected to a V-volt
2
JEE Main-30.01.2023, Shift-I battery, as shown.
Ans. (225) :

The work done in charging fully both the


condensers is

Objective Physics Volume-III 835 YCT


1  RR 
(a) CV2 (b) CV 2 4πε 0  1 2  = n4πε 0 R1
4
 R 2 − R1 
3 1
(c) CV 2 (d) CV 2 R 1R 2
4 2 = nR 1
JIPMER-2017 R 2 − R1

UPSEE-2007 R2
Ans. (c) : Energy stored in a condenser, R1
=n
1 R
U = CV 2 2
−1
2 R1
For first condenser- R2 n
1 =
U1= CV 2 (capacity = C) R1 n − 1
2
For second condenser- 112. In the given circuit one of the 3 µF capacitors
1C has 600 µJ of energy. Then the potential
U2 =   ⋅ V 2 difference across 2µF capacitor is
2 2 
1  C
U2 = CV 2  capactity = 
4  2
So, Work done = Energy stored in condensers.
W = U1 + U2
1 1
= CV 2 + CV 2 (a) 40 V (b) 15 V
2 4
3 (c) 60 V (d) 45 V
= CV 2 AP EAMCET (Medical)-24.04.2019, Shift-I
4
111. Capacitance of an isolated conducting sphere of Ans. (d) : Potential energy across 3µF,
radius R1 becomes n times when it is enclosed 1
U = CV 2
by a concentric conducting sphere of radius R2 2
connected to earth. The ratio of their radii 1
600×10–6 J = × 3 × 10−6 × V 2
 R2  2
  is :
R
 1 V = 20 Volts
n 2n Potential across AB,
(a) (b) VAB = 20 + 20 + 20 = 60 Volts
n −1 2n + 1
n +1 2n + 1 So, potential difference across 2µF,
(c) (d)  6 
n n V = 60×   = 45 Volts
JEE Main-25.07.2022, Shift-II 6+2
Ans. (a) : Given, C2 = nC1 113. Two identical condensers are joined as shown
in the figure. When the switch S is closed, the
total energy of the system is U1. If the switch is
opened and both the condensers are filled with
a dielectric of dielectric constant 3. then the
energy of the system becomes U2.
U
The value of 1 is
U2

For sphere (i),


Capacitance (C1) = 4πε0R1
For sphere (ii),
 RR 
Capacitance (C2) = 4πε 0 ⋅  1 2 
 R 2 − R1  (a) 3 : 1 (b) 5 : 1
According to question, (c) 3 : 5 (d) 5 : 3
C2 = nC1 AP EAMCET-24.04.2018, Shift-II

Objective Physics Volume-III 836 YCT


Ans. (c): When the switch S is closed, then initially 8.85 ×10−12 × 50 ×10−4
potential difference across both the capacitor is same. =
2 ×10−3
Then the total energy of the system, = 8.85 × 25 × 10-13
1 1
U1 = CV 2 + CV 2 Ceff = 221.25 × 10–13 F
2 2 Let E be the energy stored
U1 = CV2 Then,
If the switch is opened and both the condensers are 1
E = Ceff V 2
filled with a dielectric of dielectric constant 3. So, the 2
capacitance of capacitors becomes 3C, while potential 1
= × 221.25 × 10−13 × (12 )
2

difference across A is V and potential difference across 2


V = 221.25×6×12×10–13
B is .
3 = 15930 × 10–13 J
Then the energy of the system, = 1.59 × 10–9 J
2 = 1.6 × 10–9 J
( 3C ) × V 2 + ( 3C ) ×  
1 1 V
U2 =  1.6 nJ
2 2 3
115. A capacitor of capacity C1 is charged up to
10 potential V volt and then connected in parallel
= CV 2
6 to an uncharged capacitor of capacity C2. The
U1 CV 2 6 3 final potential difference across each capacitor
Required ratio, = = = will be
U 2 (10 / 6 ) CV 2
10 5
C2 V C1V
114. Three plates A, B and C each of area 50 cm2 (a) (b)
C1 + C 2 C1 + C 2
have separation 3 mm between A and B and 6
mm between B and C. The energy stored when  C   C 
(c) 1 + 2 V (d) 1 − 2  V
the plates are fully charged by a 12 volt battery  C1   C1 
is AIPMT-2002
Ans. (b) : Given, On first capacitor charge, Q1 = C1V

On second capacitor charge, Q2 = 0 (Uncharged)


When they are connected to parallel,
Charge, Q = Q1 + Q2
(a) 2 nJ (b) 1.6 nJ Q = C1V …..(i)
(c) 5 2 nJ (d) 3.2 nJ Capacitance, C = C 1 + C 2 ….. (ii)
The final potential difference across each capacitor,
AMU-2016
Q
ε0 A V=
Ans. (b) : We know that, C = C
d C1V
Where, A and d are area of plates and distance between =
C1 + C 2
the plates respectively.
According to given condition, capacitors are connected 116. The four capacitors, each of 25μF are
in parallel connected as shown in figure. The DC
voltmeter reads 200 V. The charge on each
∴ Ceff = C1 + C2
plate of capacitor is
ε0 A ε 0 A
= +
d1 d2

1 1  1 1 
Ceff = ε o A  +  = ε o A  −
 d1 d 2   3 × 10
−3
6 ×10−3 
εo A  1 1 
Ceff = +
10−3  3 6 
(a) ± 2 × 10–3 C (b) ± 5 × 10–3 C
εA (c) ± 2 × 10–2 C (d) ± 5 × 10–2 C
= o −3
2 × 10 AIPMT-1994
Objective Physics Volume-III 837 YCT
Ans. (b) : 119. Two identical capacitors C1 and C2 of equal
capacitance are connected as shown in the
circuit. Terminals a and b of the key k are
connected to charge capacitor C1 using battery
of emf V volt. Now, disconnecting a and b the
terminals b and c are connected. Due to this,
what will be the percentage loss of energy?

Two capacitors on the left side are in parallel and


potential across each equal to V
Similarly, the capacitors on right side are in parallel and
potential across each equal to V.
Therefore, charge on each plate of capacitor is,
Q = ± 25×10–6×200 (a) 75% (b) 0%
Q = ± 5 × 10 C–3 (c) 50% (d) 25%
117. A 4µF capacitor is charged to 400 V and then NEET Odisha-2019
its plates are joined through a resistance of Ans. (c) : When connected within a & b across C1
1kΩ. The heat produced in the resistance is 1
E1 = CV 2
(a) 0.16 J (b) 1.28 J 2
(c) 0.64 J (d) 0.32 J When connected within b and c across C1 and C2
AIPMT-1989 1 C1C 2
∆E = V2
Ans. (d) : Energy lost as heat through the resistor 2 C1 + C 2
equals the energy stored in the capacitor.
1 C2 2
1 = V
Heat produce = electrical energy stored = CV 2 2 2C
2
1
1 −6
= × 4 × 10 × 160000 = CV 2
2 4
Heat produce = 0.32 J ∆E
% loss in energy = ×100
E1
118. A parallel plate condenser has a uniform
electric field E (V/m) in the space between the 1
CV 2
plates. If the distance between the plates is = 4 × 100
d(m) and area of each plate is A(m2). The 1
CV 2
energy (joule) stored in the condenser is 2
1 = 50 %
(a) ε0 E 2 (b) ε0EAd (c)
2 120. A capacitor of capacitance 6µF is charged upto
1 100 V. The energy stored in the capacitor is
(c) ε0 E 2 Ad (d) E2Ad/ε0 (a) 0.6 J (b) 0.06 J
2
(c) 0.03 J (d) 0.3 J
NEET-2021
UP CPMT-2004
MHT-CET 2020
Ans. (c) : Given, C = 6 × 10–6 F, V = 100 V
AIPMT-2011
The energy stored in the capacitor is given by,
J&K CET-2016
1
Ans. (c) : The capacitance of parallel plate condense is U = CV 2
2

C= 0 1
d = × 6 × 10 –6 × (100) 2
2
And potential difference between the plates is V = Ed
= 3 × 10 –2 J
∴ Energy stored
So, U = 0.03J
1 1  Aε0 
 ( Ed )
2
U = CV =  2
121. A capacitor is charged by a battery and energy
2 2 d 
stored is ‘U’. Now the battery is removed and
1 the distance between plates is increased to four
= Aε0 E d 2

2 times. The energy stored becomes


Objective Physics Volume-III 838 YCT
(a) 3U (b) 2U 124. As shown in the figure below, if a capacitor C
(c) U (d) 4U is charged by connecting it resistance R, then
MHT-CET 2020 energy given by the battery will be
Ans. (d) : As we know that, energy stored in capacitor,
q2 q 2d
U= =
2C 2εo A
When distance between plates is increased to four times
then energy stored in capacitor,
q 2 ( 4d )  q 2d 
U' = = 4 
2εo A  2ε o A 
U' = 4U
122. Two capacitors, one 4 pF and the other 6 pF, 1 1
connected in parallel, are charged by a 100 V (a) CV 2 (b) more than CV 2
2 2
battery. The energy stored in the capacitors is
(a) 12 × 10-8 J (b) 2.4 × 10-8 J 1
-8 (c) less than CV 2 (d) zero
(c) 5.0 × 10 J (d) 1.2 × 10-6 J 2
Manipal UGET-2017 CG PET -2018
Ans. (c) : Given, C1 = 4 pF and C2 = 6 pF Ans. (a) : We know,
Voltage = 100V
Both capacitors connected in parallel. Energy stored in a capacitor
Hence, equivalent capacitance, 1
E = CVC2 .....(i)
C = C1 + C2 = 4 pF + 6 pF = 10 pF 2
C = 10 × 10–12F Where, VC = Voltage drop across capacitor.
1 Apply KVL in Circuit
Energy stored in the capacitor (E) = CV 2
2 V = VR + VC
1
= × 10 ×10 × (100 )
−12 2
We know, in steady state current through resistance = 0
2 VR = IR = 0
1
= × 10 ×10−12 × 104 Hence, V =VC
2 From equation (i)
E = 5.0 × 10–8 J
1
123. Two conductors of the same material have E = CV 2
their diameters in the ratio 1:2 and their 2
lengths in the ratio 2:1. If the temperature 125. The energy per unit volume for a capacitor
difference between their ends is the same then having area A and separation d kept at
the ratio of amounts of heat conducted per
second through them will be potential different V is given by
(a) 4:1 (b) 1:4 1 V2 1 V2
(a) εo 2 (b)
(c) 8:1 (d) 1:8 2 d 2ε o d 2
Manipal UGET-2012
Ans. (d) : We know, 1 Q2
(c) CV 2 (d)
Q K × A × ∆t 2 2C
H= = CG PET- 2017
t l
A AIPMT-2001
H∝
l Ans. (a) : We know,
H1 A1 l2 Total energy Stored in a capacitor,
∴ = × 1
H 2 A 2 l1 E = CV 2
2
H1 d12 l2  πd 2 
= ×  A = πr =
2
 1ε A  ε A
H 2 d 2 2 l1  4  or E =  0  V 2 Q C = 0 
2
2 d   d 
H1  1  1
=  × Volume between the plate = A × d
H2  2  2 Hence, Energy per unit Volume,
H 1
1= 1  ε0 A  2
H 8
2  V 1 ε0 V 2
= 
E 2 d 
=
∴ H1 : H2 = 1 : 8 Ad Ad 2 d2

Objective Physics Volume-III 839 YCT


126. A 600 pF capacitor is charged by a 200V Ans. (a): Given that,
supply. It is then disconnected from the supply Capacitance (C) = 12 pF = 12 × 10–12F
and is connected to another uncharged 600 pF And voltage (V) = 50V
capacitor. Electrostatic energy lost in the 1
process is Energy stored in capacitor = CV 2
2
(a) 6×10−6J (b) 3×10−6J
1
= × 12 × 10−12 × ( 50 )
−9
(d) 3×10−9J
2
(c) 6×10 J
CG PET -2016 2
Ans. (a) : Given that, = 1.5 × 10–8J
Capacitance of Capacitor (C) = 600pF = 600 × 10–12 F 128. A 1µF capacitance of TV is subjected to 4000 V
and potential difference (V) = 200 V potential difference. The energy stored in the
capacitor is
(a) 8 J (b) 16 J
(c) 4×10−3J (d) 2×10−3J
CG PET- 2006
Ans. (a) : Given that,
Capacitance of capacitor (C) = 1µF = 1 × 10–6F
and Voltage (V) = 4000V
We know that,
1
Energy stored in capacitor = CV 2
2
1
= × 1×10 × ( 4000 )
−6 2

1 2
Electrostatic Energy stored (E) = CV 2 1
2 = × 10−6 × 16 × 106
2
= × ( 600 × 10−12 ) × ( 200 )
1 2
= 8J
2
1 129. Two capacitors of capacitance 2µF and 3 µF are
= × 600 × 10−12 × 4 × 104 joined in series. Outer plate of first capacitor is
2
at 1000V and outer plate of second capacitor is
= 1.2 × 10–5J
earthed. Now the potential on inner plate of
After disconnecting, A 600 pF capacitor added.
each capacitor will be
Then equivalent capacitance (C′) (a) 700V (b) 200V
1 1 1 (c) 600V (d) 400V
= +
C′ C C CG PET- 2005
1 1 Ans. (d) : Given that,
= +
600 600 Capacitance, C1 = 2µF
1
=
1 Capacitance, C2 = 3µF
C' 300 Voltage, V1 = 1000 V
C′ = 300 pF
Then, new electrostatic energy,
1
E′ = C′ × V 2
2
1 1 1 1
= × 300 × 10−12 × ( 200 )
2
Equivalent Capacitance, = +
2 Ceq 2µF 3µF
= 0.6 × 10–5J
6
Loss in electrostatic energy (E – E′) Ceq = µF
–5
= 1.2 × 10 – 0.6 × 10 5 5
–5 Charge, Q on each plate = C .V
= 0.6 × 10 eq 1
= 6 × 10–6J 6
= × 1000 ×10 –6
127. Capacitor of 12 pF capacitance is connected to 5
50 V battery then electrostatic potential energy Q = 1200 µC
will be Potential on inner plate ,
−8 −7
(a) 1.5×10 J (b) 2.5×10 J
−5 Q
(c) 3.5×10 J (d) 4.5×10−2J V = V1 –
CG PET- 2007 C

Objective Physics Volume-III 840 YCT


Q 1 1
= 1000 − (a) C(V12 − V22 ) (b) C(V12 + V22 )
C 4 4
1200µC 1 1
= 1000 − (c) C(V1 − V2 )2 (d) C(V1 + V2 ) 2
2µF 4 4
BITSAT-2013
= 1000 –600 JCECE-2018
V = 400 V Ans. (c) : Given,
130. A capacitor is charged to store an energy U. Capacitance of two identical Capacitors = C
The charging battery is disconnected. An Initial potential = V1
Final potential = V2
identical capacitor is now connected to the first Initial energy of the system
capacitor in parallel. The energy in each of the 1 1
capacitors is Ei = CV12 + CV22
2 2
(a) 3 U/2 (b) U
1
(c) U/4 (d) U/2 Ei = C ( V1 + V2 2 )
2

2
BITSAT-2008 When the capacitor are joined, they reach a common
Q2 Totalcharge CV1 + CV2
Ans. (c) : Energy stored in capacitor, U = Potential = =
2C totalcapacity 2C
When battery is disconnected, then the total charge will V + V2
= 1
be equally shared by two capacitor. 2
( Q / 2)
2 ∴ Final energy of combined system
Energy of each capacitor, U' = 2
1  V + V2 
2C Ef = ( 2C )  1 
2  2 
1 Q2
= × .....(ii) Decrease in energy, ∆E = Ei – Ef
4 2C
1 1  V + V2  
2
Comparing the equation (i) and (ii), we get– ∆E =  C ( V12 + V2 2 ) − ( 2C )  1  
U 2 2  2  
U' =
C 2 ( V1 + V2 ) 
2
4
∆E= ( V1 + V2 ) − 2. 
2

131. If the potential of a capacitor having capacity 6 2 4 


µF is increased from 10V to 20V, then increase C ( V + V2 ) 
2

in its energy will be ∆E = ( V12 + V22 ) − 1 


2 2 
(a) 4 × 10–4 J (b) 4 × 10–6 J
(c) 9 × 10 J
–4
(d) 12 × 10–6 J C  2V12 + 2V22 − V12 − V22 − 2V1V2 
∆E =  
BITSAT-2011 2 2 
CG PET - 2019 C  V 2
+ V 2
− 2V V 
∆E =  1 2 1 2

Ans. (c) : Given, 2 2 
Capacitance (C) = 6µF C
∆E = ( V1 − V2 ) 
2
Initial voltage (V1) = 10V 4
Final voltage(V2) = 20V C
∆E = ( V1 − V2 )
2
Increase in energy of a capacitor,
4
E = C ( V22 − V12 )
1 133. A parallel plate capacitor of capacitance C is
2 connected to a battery and is charged to a
1 potential difference V. Another capacitor of
= × 6 ×10−6 ( 202 − 102 ) capacitance 2C is similarly charge to a
2 potential difference 2V. The charging battery is
1 now disconnected and the capacitors are
= × 6 ×10−6 × 300 connected in parallel to each other in such a
2
way that the positive terminal of one is
= 9 × 10–4 Joule connected to the negative terminal of the other.
132. Two identical capacitors, have the same The final energy of the configuration is
capacitance C. One of them is charged to 3
(a) Zero (b) CV 2
potential V1 and the other to V2. The negative 2
ends of the capacitors are connected together. 25 9
(c) CV 2 (d) CV 2
When the positive ends are also connected, the 6 2
decrease in energy of the combined system is- BITSAT-2015
Objective Physics Volume-III 841 YCT
Ans. (b): Given, 135. In the given electrical circuit, if the switch S is
First capacitance = C closed then the maximum energy stored in the
Second capacitance = 2C inductors is:
Change in potential (∆V) = (2V − V)
=V

(a) 3J (b) 9 J
(c) 12 J (d) 6 J
Equivalent Capacitance, Ceq = C + 2C = 3C
AP EAMCET (21.04.2019) Shift-I
1
Stored energy in capacitor, U = Ceq ( ∆V )
2
Ans. (a) :
2
1 1 Q2
3 Energy in First capacitor, E1 = CV 2 =
= CV 2 2 2 C
2
1 42
134. A 100 V battery is connected across the series E1 = × = 8J
combination of the two capacitors of 4µF and 2 1
2
8µF. The energy stored in the series 1 Q
Energy in second capacitor, E2 = .
combination is 2 C
(a) 0.75×10-2 J (b) 1.33×10-2 J 2
1 2
(c) 0.5 J (d) 1 J = .
AP EAMCET (21.09.2020) Shift-I 2 2
Ans. (b) : Given that, E 2= 1 J
Potential of battery, V = 100V Total energy, Eo = E1 + E2
C1 = 4 µF =8+1
C2 = 8 µF = 9J
Now switch is closed then the common potential of
capacitors
C V + C2 V2
Vcommon = 1 1
C1 + C2
Q1 4
V1 = = = 4V
C1 1
Q2 2
Since, capacitors are connected in series so the V2 = = = 1V
equivalent capacitance will be, C2 2
1 1 1 1× 4 + 2 × 1
= + Vcommon =
Ceq 4 8 1+ 2
1 4+8 = 2V
= Hence, the new arrangement of energy,
Ceq 8 × 4
1 1
32 8 E1 = C1V 2Common = × 1× 4 = 2J
Ceq = µF = × 10−6 F 2 2
12 3 1 1
1 E 2 = C2 Vcommon
2
= × 2 × 4 = 4J
Energy stored, E = Ceq V 2 2 2
2 Now from conservation of energy stored in the inductor
1 8
= × (100 ) × 10−6
2
E1  Eo – (E1 + E2)
2 3 = 9 – (2 + 4)
1 8
= × × 104 × 10−6 = 3J
2 3 136. In an oscillating LC circuit, the maximum
4 charge on the capacitor is Q. The charge on the
= × 104 × 10−6
3 capacitor when the energy is stored equally
= 1.33 × 10–2 J between the electric and magnetic field is

Objective Physics Volume-III 842 YCT


Q Q 138. Two identical air filled parallel plate capacitors
(a) (b)
2 3 are charged to the same potential in the
manner shown by closing the switch S. If now
Q
(c) Q (d) the switch S is opened and the space between
2 the plates is filled with dielectric of relative
AP EAMCET (20.04.2019) Shift-1 permittivity εr, then
Ans. (d) : The electric energy stored when energy is
equally distributed.
1 q2
U 'E =
2 C
The energy stored in the capacitor when it is maximum
charged.
(a) the potential difference as well as charge on
1 Q2 each capacitor goes up by a factor εr
UE =
2 C (b) the potential difference as well as charge on
At any time electrostatic energy is equal to magnetic each capacitor goes down by a factor εr
energy. (c) the potential difference across a remains
1 Q2 1 q 2 1 2 constant and the charge on B remains
= = + LI
2 C 2 c 2 unchanged
(d) the potential difference across B remains
q 2 LI 2
Q = constant while the charge on A remains
2c 2 unchanged
According to question − UPSEE - 2012
UE = 2UE'
Ans. (c) : •When the two capacitor gets charge with
1 Q2 1 q2 same potential the voltage across A & B and charged is
= 2×
2 C 2C fixed.
Q2 • When we open the switch capacitor get A is remains
= q2 connected with battery but B is isolated so the voltage
2
across A remains constant.
Q2 And the charge on B is fixed but when we insert the
q=
2 dielectric its capacitance changes so the potential
Q difference also changes.
q=
2 139. A 5.0 µF capacitor is charged to a potential
137. The total electrostatic energy stored in both the difference of 800 V and discharged through a
capacitor is conductor. The energy given to the conductor
during the discharge is
(a) 1.6 × 10-2 J (b) 3.2 J
(a) 9 µJ (b) 40.5µ J (c) 1.6 J (d) 4.2 J
(c) 13.5µ J (d) 18µ J UPSEE - 2009
UPSEE - 2017 Ans. (c) : Given, capacitance (C) = 5µF, voltage (V) =
Ans. (a) : Given, capacitance C1 = 3µF, C2 = 6µF 800 V
Capacitors are connected in series combination, 1
We know that, energy of conductor = CV 2
C × C2 2
So, equivalent capacitance (Ceq) = 1 1
C1 + C2 = × 5 × 10−6 × (800) 2
2
3× 6 18
Ceq = = = 2µF 1
3+6 9 = × 5 × 10−6 × (800)(800)
2
1 2 1
Since, total electrostatic energy (U) = Ceq V = × 5 × 10−6 × 64 × 104
2 2
1
U = × 2 × ( −3) ×10
2 –6 = 32 × 10–2 × 5
2 = 160 × 10–2
U = 9µJ = 1.6 J

Objective Physics Volume-III 843 YCT


140. The energy stored in a capacitor of capacitance Charge conserve, (Q Q = CV )
C having a charge Q under a potential V is
∴ Q = q1 + q2
1 1
(a) Q 2 V (b) C2 V 1µF × 30 = 1µF × V + 2µF ×V
2 2 30 = 3 V
1 Q2 1 V = 10 volt
(c) (d) QV
2 V 2 1
Final energy (Ufinal) = C1V 2 + C 2 V 2
1 2
(e) CV
2 1
= (C1 + C2 )V 2
Kerala CEE- 2014 2
Ans. (d) : Capacitance of capacitor = C 1
= (1 + 2) (10 )
2

Charge on capacitor = Q 2
Potential = V 1
Q = (3)100
We know that, V = 2
C Ufinal = 150µJ
Work done to bring a small charge dQ at the potential V Loss in energy (∆U) = Ufinal – Uinitial
Q = ( 450 − 150 ) µJ
dW = V dQ = dQ
C = 300 µJ
Work done to change it from it, 0 to Q is – 142. Figure given below shows two identical parallel
Q
W = ∫ dW plate capacitors connected to a battery with
0
switch S closed. The switch is now opened and
QQ the free space between the plate of capacitors is
W=∫ dQ
0 C filled with a dielectric of dielectric constant 2.
1 Q2 What will be the ratio of total electrostatic
W= energy stored in both capacitors before and
C 2 after the introduction of the dielectric?
1Q
W= .Q (Q Q = CV)
2C
1
W = VQ
2
141. A capacitor of capacitance value 1 µF charged
to 30 V and the battery is then disconnected. If (a) 2 : 1 (b) 5 : 1
the remaining circuit is connected across a 2 µF (c) 4 : 5 (d) 5 : 4
capacitor, the energy lost by the system is BCECE-2016
(a) 300 µJ (b) 450 µJ JCECE-2016
(c) 225 µJ (d) 150 µJ Ans. (c) : When switch is closed then the total initial
(e) 100 µJ 1 1
energy (Uinitial) = CV 2 + CV 2
Kerala CEE - 2008 2 2
2
Ans. (a) : Case-I U initial = CV ……(i)
When switch is open then the capacitor A is connected
with voltage so voltage is constant. In capacitor B,
charge remains same when dielectric is introduced.
1 Q 2 1 Q2
Energy in 'B' ( E ) = =
2 KC 2 2C
1 1
∴ Uinitial = CV 2 = ×1× (30) × 10−6 1 (CV) 2 C2 V 2 CV 2
2 2 E= = =
2 2C 4C 4
1
= × 900 × 10 −6 =450 µJ Energy in 'A'
2
1 1
Case-II E = KCV 2 = 2CV 2 = CV 2
2 2
CV 2
Total energy of A and B = + CV 2
4
5
Ufinal = CV 2
4

Objective Physics Volume-III 844 YCT


U initial CV 2 4 1 3
Therefore, = = (a) (b)
U final 5 5 2 4
CV 2 4 1
4 (c) (d)
143. Three point charges 1C, 2C and 3C are placed 3 3
at the corners of an equilateral triangle of side COMEDK-2014
1 m. The work done in bringing these charges Ans. (c) : Given, d = 1 mm, t = 0.75 mm
to the vertices of a smaller similar triangle of We know that, series charge on each capacitor-
side 0.5 m is: ∴ Qx = Qy
(a) 2.7 × 1010 J (b) 9.9 × 1010 J Where, subscripts x and y refer air and paper-
10
(c) 10.8 × 10 J (d) 5.4 × 1010 J ∴ CxVx = CyVy
JCECE-2005 ε0 KA
Ans. (b) : Given, three point charge, q1 = 1C, q2 = 2C, Vx C y K(d − t)
q3 = 3C or = = t =
Vy C x ε 0 A t
We know that, (d − t)
1 q q qq qq 
W=U= × 1 2 + 2 3 + 3 1 4(1 − 0.75) 4(0.25) 4 × 25 4
4πε0  d1 d2 d3  = = = =
0.75 0.75 75 3
1 × 2 2 × 3 3 × 1  146. A 10mF capacitor has been charged to a
= 9 ×109 ×  + +
 1 1 1  potential of 100 V. Suddenly if it explodes, the
= 9 × 109 × [ 2 + 6 + 3] energy given out is
9
(a) 50J (b) 105 J
= 9 × 10 × 11 (c) 50 mJ (d) 103 J
= 99 × 109 J COMEDK-2014
= 9.9 ×1010 J Ans. (a) : Given that, C = 10 mF = 10 ×10–3 F, V =
144. A 10 µF Capacitor is charged to 10 V and 100V
disconnected from the battery. If another 1
We know that, U = CV 2
uncharged 10 µF capacitor is connected across 2
it in parallel, the voltage across the 1
combination will be U = × 10 × 10−3 × (100)2
2
(a) 5V (b) 10 V U = 50 J
(c) 20 V (d) 0
147. The plates of a parallel plate capacitor have an
COMEDK-2012
area of 90 cm2 each and are separated by 2.5
Ans. (a) : Given, V = 10 V, C = 10 µF mm. The capacitor is charged by connecting it
to a 400 V supply. The energy per unit volume
u is
(a) 0.075 J m–3 (b) 0.113 J m–3
–3
(c) 0.256 J m (d) 0.475 J m–3
COMEDK-2018
10µF, Ans. (b) : Given that, A = 90 cm2 = 90 × 10–4 m2
d = 2.5 mm = 2.5 × 10–3 m, V = 400 V
We know that, charge on capacitor (Q) = CV
1
Q = 10 × 10–6 = 100 µC We know that, U = CV2
Now, charge will be half on each, 2
Q 100 1 ε0 A 2
= = 50 µC U= × V
2 2 2 d
Q 50 1 (8.85 × 10−12 ) × 90 × 10−4 × 400 × 400
Hence, potential (V) = = =5V U= ×
C 10 2 2.5 ×10−3
145. The region between the parallel plates of a 8.85 × 90 ×16
capacitor is filled with parallel layers of air and U= × 10–9
2 × 2.5
paper (of dielectric constant 4). The space
U = 2.55 × 10–6 J
between the plates is 1mm and the thickness of
paper is 0.75 mm. The ratio of the voltages Volume of the medium in plates (V′),
across air and paper is V′ = A × d
Objective Physics Volume-III 845 YCT
V′ = 90 × 10–4 × 2.5 × 10–3 Ans. (c) : Given, capacitance of capacitor (C) = 600 µF
V′ = 225 × 10–7 m3 = 6 × 10–4 F, voltage (V) = 250 V
U 1
Energy stored per unit volume (u) = We know that, the energy of capacitor (E) = CV2
V' 2
2.55 × 10−6 1
u= = 0.113 Jm–3 = × 6 × 10–4 × (250)2
225 × 10−7 2
= 18.75 J
148. The instrument used by doctors for endoscopy 151. The amount of work done in increasing the
works on the principle of– voltage across the plates of capacitor from 5V
(a) total internal reflection to 10V is ‘W’. The work done in increasing it
(b) reflection from 10V to 15V will be
(c) refraction (a) W (b) 0.6 W
(d) none of the above (c) 1.25 W (d) 1.67 W
BCECE-2007 Manipal UGET-2014
MHT CET-2016
Ans. (a) : The instrument used by doctors for UP CPMT-2014
endoscopy works on the principle of total internal
Ans. (d) : Given, work done to increase voltage from
reflection of light. 5V to 10 V = W and work done to increase voltage
Total internal reflection works on two conditions- from 10V to 15 V = W'
• Light must be propagating from denser to rarer 1
medium. We know that, energy of capacitor (E) = CV2
2
• The angle of incidence must exceed the critical angle. ∴ Work done in changing the potential from 5V to
149. If the potential of a capacitor having capacity 10V,
6µF is increased from 10V to 20V, then
W = C ( V22 − V12 )
1
increase in its energy is : 2
(a) 12×10–6J (b) 9×10–4J 1
(c) 4.5×10 J–6
(d) 2.25×10–6J W = C(102 – 52)
2
AIPMT-1995 1
BCECE-2004 W = C (100 − 25 )
2
Ans. (b) : Given, capacitance of the capacitor (C) = 6 1
µF = 6 × 10–6 F W = C × 75
2
Initial voltage (Vi) = 10 V Similarly, work done in changing the potential from
Final voltage (Vf) = 20 V 10V to 15V,
1 1
We know that, energy of a capacitor (E) = CV 2 W' = C(152 – 102)
2 2
1
1
Now, change in energy (∆E) = CVf2 − CVi2
1 = C ( 225 − 100 )
2 2 2
1
= C ( Vf2 − Vi2 )
1 W' = C × 125
2 2
1
1 × C × 125
= × 6 × 10–6(202 – 102) W' 2 125
2 ∴ = =
W 1 75
1 × C × 75
= × 6 × 10–6(400 – 100) 2
2 ⇒ W' = 1.67 W
1
= × 6 × 10–6 × 300 152. The energy required to charge a parallel plate
2 condenser of plate separation d and plate area
= 3×10–6×300 of cross-section A such that the uniform
= 9 × 10–4 J electric field between the plates is E, is
150. A 600 µF capacitor is connected to a 250 V 1
(a) ε0 E 2 / Ad (b) ε 0 E 2 / Ad
battery. The amount of electrostatic energy 2
stored in it, is 1
(c) ε 0 E 2 Ad (d) ε0 E 2 Ad
(a) 15.00 J (b) 187.5 J 2
(c) 18.75 J (d) 150 J MHT CET-2008
MHT CET-2019 AIPMT-2008
Objective Physics Volume-III 846 YCT
Ans. (c) : We know that, 155. A coil of resistance 10Ω and an inductance 5H
Energy given by cell = Energy stored in capacitor + is connected to a 100 V battery. The energy
heat lost stored in the coil is
1 (a) 325 erg (b) 125 J
Now, energy stored in capacitor = CV2
2 (c) 250 erg (d) 250 J
1 VITEEE-2014
and heat lost = CV2
2 Ans. (d) : Given, resistance (R) = 10 Ω, voltage (V) =
1 1 100 V, inductance (L) = 5 H
∴ Energy given by cell = CV2 + CV2
2 2 We know that, V = IR [Ohm's Law]
2
2CV V 100
= = CV2 ….(i) I= =
2 R 10
Aε0 I = 10 A
Q Capacitance of capacitor (C) = ….(ii)
d 1
Now, energy stored in the coil (E) = × L × I2
Where, A = Plate area of cross-section 2
d = Distance between plate
1
Potential (V) = E × d ….(iii) × 5 × (10)2 = 250 J
E=
Put the value of C and V in equation (i), we get - 2
Aε 156. A cylindrical capacitor has charge Q and
Energy given by cell = 0 ×(Ed)2 length L. If both the charge and length of the
d
capacitor are doubled, by keeping other
= Aε0E2d
parameters fixed, the energy stored in the
153. A condenser has a capacity 2 µF and is charged capacitor
to a voltage of 50 V. The energy stored is
(a) Remains same (b) Increases two times
(a) 25 × 105 J (b) 25 J
(c) Decreases two time (d) Increases four times
(c) 25 × 10 erg (d) 25 × 103 erg
MHT CET-2002 VITEEE-2008
Ans. (d) : Given, capacitance of capacitor (C) = 2 µF = Ans. (b) : We know that,
2 × 10–6 F, voltage (V) = 50 V
We know that, energy stored in a capacitor,
1
E = CV2
2
1
∴ E = × 2 × 10–6 × (50)2
2
E = 2500 × 10–6 J
E = 25 × 10–4 J
E = 25 × 103 erg [Q 1 J = 107 erg] 1 Q2 Q2
Energy stored capacitor (U) = =
154. A 700 pF capacitor is charged by a 50 V 2 C 2C
battery. The electrostatic energy stored by it is 2πε0 L
Where, C =
(a) 17.0 × 10–8 J (b) 13.6 × 10–9 J log e (b / a)
(c) 9.5 × 10 J
–9
(d) 8.7 × 10–7 J
1 Q2
MHT CET-2000 Now, U1 = × loge(b/a)
2 2πε0 L
Ans. (d) : Given, capacitance of capacitor (C) = 700 pF
= 700 × 10–12 F, voltage (V) = 50 V Q2
U∝
We know that, L
1 According to the question,
Energy stored in a capacitor (E) = CV2
2 1 (2Q) 2
U2 = × loge(b/a)
1 2 2πε0 (2L)
E = × 700 × 10–12 × (50)2
2
1 4Q 2
1 U2 = × loge(b/a)
= ×700×2500×10–12 2 2πε0 × 2 × L
2
E = 8.75 × 10–7 J U2 = 2 U1

Objective Physics Volume-III 847 YCT


157. The energy stored in the capacitor as shown in 159. When an additional charge of 2C is given to a
Fig (a) is 4.5×10−6 J. If the battery is replaced capacitor, energy stored in it is increased by
by another capacitor of 900 pF as shown in Fig. 21%. The original charge of the capacitor is :
(b), then the total energy of system is (a) 30 C (b) 40 C
(c) 10 C (d) 20 C
Karnataka CET-2013
Ans. (d) : Let, the charge Q produced when a voltage V
is applied to the capacitance C.
Then, Q = CV
Q2
We know that, E=
2C
Let the new charge energy be given as-
(a) 4.5×10−6J (b) 2.25×10−6J Q' = Q + 2
(c) zero (d) 9×10−6J And the new energy be given as-
VITEEE-2008 E' = 1.21E
Ans. (b) : We know that,
( Q + 2)
2

C V + C2 V2 Then, 1.21E =
Common potential ( VC ) = 1 1 2C
C1 + C2
Taking the ratio of the energy, we get,
C × 100 + C × 0
VC = (Q + 2)
2
C+C
1.21E 2C
VC = 50 V =
1 E Q2
Energy stored = 2 × CV 2 2C
2
= CV2 (Q + 2)
2

= 900 × 10–12 × (50)2 1.21 =


Q2
= 2.25 × 10–6 J
Q+2
158. A capacitor of 8 F is connected as shown in the 1.1 =
Q
figure. Charge on the plates of the capacitor :
1.1Q – Q = 2
0.1Q = 2
2
Q=
0.1
Q = 20C
160. An uncharged sphere of metal is placed inside a
(a) 32 C (b) 40 C charged parallel plate capacitor. The lines of
(c) 0 C (d) 80 C force will look like :
Karnataka CET-2016
Ans. (a) : We know that,

The capacitor is in steady state. No current flow through


20Ω
So, voltage across capacitor = voltage across 4Ω
4 Karnataka CET-2004
V = × 5 = 4 Volts
1+ 4 Ans. (a) : In option (a), all the electric field lines are
Charge store the plates of capacitor, perpendicular to the surface of the uncharged sphere of
Q = CV metal also, these electric field line are slightly curved
Q = 8 × 4 = 32C near the surface of the conducting sphere.
Objective Physics Volume-III 848 YCT
In option (b), the electric field lines are not Ans. (a) : Capacitance of a parallel plate capacitor
perpendicular to the surface of the metallic sphere. So it εA
is incorrect. C= o
d
In option (c), the electric field lines are not When a battery is disconnected the charge on the plate
perpendicular to the surface of the uncharged metallic of the capacitor remain constant.
sphere and also not slightly curved near the surface of Q = Q'
the metal sphere. So, it also incorrect. CV = C'V'
In option (d), the electric field lines are perpendicular to
CV
the surface of the uncharged sphere of metal but these V' =
lines are not slightly curved near the surface of the C'
conductor or conducting sphere of metal. So, it is also C ε 0 A d′ d′
= × =
incorrect. C′ d ε0 A d
161. What fraction of the energy drawn from the d'
charging battery is stored in a capacitor ? ∴ V' = V
d
(a) 75% (b) 100%
d' > d ⇒ V' > V
(c) 25% (d) 50%
So, the potential difference between the plates of the
Karnataka CET-2002 capacitor will be increase.
Ans. (d) : Energy stored in a capacitor is given by 164. Energy stored per unit volume of a parallel
1 plate capacitor having plate area A and plate
U = qV …..(i)
2 separation d when charged to a potential of V
The total energy of the battery is given by volts is (air space in between the plates)
U' = Charge × potential 1 q2
U' = qV (a) C2 V 2 (b)
2 4C
Putting these value in equation (i), we get -
1 V 1  V2 
1 (c) ε0   (d) ε0  2 
U = ⋅U' 2 d 2 d 
2
J&K CET-2009
U'
U = = 50% of the total energy drawn from the Ans. (d) : Energy stored in capacitor of capacitance
2 1
charging battery. E = CV 2
162. A capacitor is charged by a battery and the 2
Capacitance of a parallel plate capacitor
energy stored is U. The battery is now removed
and the separation distance between the plates εA
C= o
is doubled. The energy stored now is d
U Volume of parallel plate capacitor = A × d
(a) (b) U 1
2 CV 2
(c) 2U (d) 4U Thus energy stored per unit volume = 2
J&K CET-2010 Ad
Ans. (c) : We know that, 1  V2 
= εo  2 
Q2 q 2d 2 d 
Energy stored (U) = =
2C 2ε0 A 165. Two capacitors each of capacity 2 µF are
Therefore, U∝d connected in parallel. If they are connected to
∴ U1 = U, U 2 = ? d1 = d, d 2 = 2d 100 V battery, then energy stored in them is
(a) 0.02 J (b) 0.04 J
U1 d1 (c) 0.01 J (d) 200 J
So, = ⇒ U 2 = 2U
U2 d2 J&K CET-2006
Hence, energy stored in capacitor after increase the Ans. (a) : Given, V = 100 Volt
distance between the plates is 2U. Two capacitors are connected in parallel
163. A parallel plate capacitor is charged to a Ceq = C1 + C2
potential of V volts. The battery is then Ceq = 2µF + 2µF = 4µF
disconnected and the distance between the 1
plates of the capacitor is increased using an Energy stored in this system (U) = Ceq V 2
insulating handle. The potential difference 2
1
U = × 4 × 10 × (100 )
2
between the plates of the capacitor will –6

(a) Increase (b) Decrease 2


(c) Not change (d) Become zero U = 2 × 10–2
J&K CET-2009 U = 0.02J
Objective Physics Volume-III 849 YCT
166. The plates of a charged condenser are 169. The energy stored in a condenser of capacity C
connected to a voltmeter. If the plates are which has been raised to a potential V is given by
moved apart, the reading of voltmeter will 1 1
(a) Increase (a) CV (b) CV 2
2 2
(b) Decrease 1
(c) Remain unchanged (c) CV (d)
(d) Information is insufficient 2VC
UP CPMT-2010
J&K CET-2005
Ans. (b) : We know that–
Ans. (a) : We know that -
Q = CV
q = CV = Constant …(i)
And parallel plate capacitor is given by- Q
So, V=
εA C
C= 0 Work done in raising potential V is given by -
r
Where, A = Area of plates dW = V.dQ
r = Separation distance between plates Q
= dQ
From equation (i), we get - C
q = CV = Constant Q
εA W = ∫ dQ
q = 0 × V = Constant C
r 1
If the plates are separated then the distance between W = ∫ QdQ
C
plates will increase, therefore the value of C will
1  Q2 
decrease where the charge (q) will be constant and the W = . 
reading of voltmeter will increase. C  2 
167. In a charged capacitor, the energy stored in 1 Q2
(a) The positive charges W= .
2 C
(b) The negative charges
Now, energy stored in a condenser of capacitor-
(c) The field between the plates
1 Q2
(d) None of these U= [Q Q = CV ]
J&K CET-2003, 1999 2 C
1 ( CV )
2
Ans. (c) : The energy of a charged capacitor are stored
in both electric and magnetic field. = ×
2 C
In electric field the energy is stored due to charges on
1 C× C× V× V
the capacitor. While in magnetic field, the energy is = ×
stored due to the Maxwell's displacement current in the 2 C
capacitor. 1
U = CV 2
168. A parallel plate capacitor is charged and then 2
disconnected from the charging battery. If the 170. If a capacitor of capacitance 10 µF has
plates are now moved farther apart by pulling potential difference of 100V across its ends.
at them by means of insulating handles, then The energy stored in it is
(a) the energy stored in the capacitor decreases (a) 0.05 J (b) 1 J
(b) the capacitance of the capacitor increases (c) 0.005 J (d) 0.1 J
(c) the charge on the capacitor decreases UP CPMT-2003
(d) the voltage across the capacitor increases
Ans. (a) : Given,
Ans. (d) : When a parallel plate capacitor is charged
C = 10µF = 10 ×10 −6 F , V = 100V
and then disconnected from the charging battery. Then
charge across the plates will be constant even when the Energy accumulated in capacitor -
plates are separated. 1
E = CV 2
The capacitance is related to the structure of parallel 2
Aε0 1
plate capacitor as = × 10 × 10−6 ×100 ×100
d 2
If the plates are moved farther, then the distance 1
between the plates d will increase and therefore, the = × 105 × 10−6
2
capacitance is decreased. And we know the formula q = 1
CV, = × 10−1
The voltage across the capacitor is increased to keep the 2
charge constant. = 0.5 × 10 −1 = 0.05J

Objective Physics Volume-III 850 YCT


171. Two capacitors, C1 = 2µF and C2 = 8 µF are Ans. (b) : Let us consider capacitance C of a capacitor
connected in series across a 300V source. Then with a potential V between the plate.
(a) The charge on each capacitor is 4.8 × 10–4 C Therefore, charge + Q and –Q on each plate.
(b) The potential difference across C1 is 60 V So,
(c) The potential difference across C2 is 240 V Q
(d) The energy stored in the system is 5.2 × 10–2 J Potential, V =
C
TS EAMCET 05.08.2021, Shift-I
Work done in giving additional change dQ
Ans. (a) : Given,
Q
C1 = 2µF dW = dQ
C
C2 = 8µF
V = 300V Total work done in giving additional charge,
Q
1
C ∫0
W= QdQ

Q2
W=
2C
Energy stored in capacitor,
1 Q2
U=W=
2 C
Series combination, 173. A parallel plate capacitor filled with a medium
1 1 1 of dielectric constant 10, is connected across a
= +
Ceq C1 C2 battery and is charged. The dielectric slab is
2 × 8 16 replaced by another slab of dielectric constant
Ceq = = = 1.6µF 15. Then the energy of capacitor will.
8 + 2 10
= 1.6 × 10 F−6 −6
(Q1µ = 10 ) (a) increased by 50%
(b) decrease by 15%
Then equivalent charge –
(c) increase by 25%
Q eq = C eq × V
(d) increase by 35%
= 1.6 × 10 −6 × 300 JEE Main-29.06.2022, Shift-I
= 4.8 × 10 −4 C Ans. (a) : Given,
As the capacitors are connected in a series therefore, the K1 = 10, K2 = 15
charge on each capacitor will be same -
1
Q eq = 4.8 × 10−4 C E = ( KC ) V 2
2
Now, the energy stored in the system - 1 1
1 K 2 CV 2 − K1CV 2
U = Ceq V 2
∴ % change = 2 2
2 1
K1CV 2
1 −6 2
U = ×1.6 × 10 × 300 × 300
2 K − K1
= 2 × 100
1 K1
= × 1.6 × 10−6 × 9 × 104
2 15 − 10
= 0.8 × 9 × 10 −2 = × 100
10
−2
U = 7.2 × 10 J % change = 50%
So, the charge on each capacitor is 4.8×10–4C 174. A capacitor C1 of capacitance 5 µF is charged
172. Stored electrostatic energy in a capacitor of to a potential of 30 V using a battery. The
capacity C, which is charged upto Q charge, is: battery is then removed and the charged
Q Q2 capacitor is connected to an uncharged
(a) (b)
2C 2C capacitor C2 of capacitance 10 µF as shown in
C2 Q2 figure. When the switch is closed charge flows
(c) (d) 2
2Q 2C between the capacitors. At equilibrium, the
MP PET-2013 charge on the capacitor C2 is ______ µC.
Objective Physics Volume-III 851 YCT
(d)
JEE Main-28.06.2022, Shift-II JEE Main-25.07.2022, Shift-I
Ans. (100) : Ans. (a) : Given,
Capacitance of capacitor (C) = 2µF = 2× 10–6 F
We know that,
Charge (Q) = CV
1
∴ V= Q ...(i)
C
From equation of straight line,
y = mx + a ….(ii)
Given, Where m is slope; a = Intercept
C1 = 5µF Equating equation (i) and (ii),
V = 30V 1
m=
Before closing the switch, C
Q = CV a=0
= 5 × 30 × 10–6 = 150 µC 1
After closing the switch, ∴ Slope (m) = = 5 × 105 = constant
2 × 10−6
Q 150
V= = = 10V ∴ Potential difference (V) across it's plates with respect
C1 + C2 10 + 5
to the change (Q) varies linear.
Q 2 = C 2V 176. For the following figure, what will be the
= 10 × 10µC potential difference between the points X and Y
Q 2 = 100 µC
175. A condenser of 2 µF capacitance is charged
steadily from 0 to 5C. Which of the following
graph represents correctly the variation of
potential difference (V) across it's plates with
respect to the charge (Q) on the condenser?

(a) 0.22 V (b) 0.86


(c) 1.15 V (d) 2.10
Assam CEE-2020
(a)
Ans. (a) : As shown in the figure,
Equivalent capacity of C1 and C2 is C′eq
1 1 1
= +
C′eq C1 C2
1 1 1
= +
C′eq 2 3
(b) 1 5
=
C′eq 6
6
C′eq = µF
5
Now, charge on C1 or C2 = Q1,
6
Q1 = C′eq × V = × 5 = 6µC
(c) 5
Objective Physics Volume-III 852 YCT
Potential difference across C1, Substituting the value of surface charge density,
Q 6 q2  q
Vx = 1 = = 3V F= Qσ = 
C1 2 2Aεo  A
Similarly, q2
Equivalent capacity of C3 and C4 is C′′eq ∴ F=
2 × πr 2 εo
1 1 1 178. The capacity of a condenser is 4 × 10–6 F and its
= +
C′′eq C3 C 4 potential is 100 V. The energy released on
1 1 1 completely discharging it is ……… .
= + (a) 0.02 J (b) 0.025 J
C′′eq 4 5
(c) 0.05 J (d) 0.10 J
1 9
= AP EAMCET-25.09.2020, Shift-I
′′
Ceq 20
Ans. (a) : Given,
20 Capacity of a condenser (C) = 4 × 10–6 F
∴ C′′eq = µF
9 Voltage (V) = 100 V
Charge on C3 or C4 = (Q2) We know that,
20 100 Energy released on discharging completely,
Q2 = C′′eq V = ×5 = µC
9 9 1
E = CV 2
Potential difference across C3 , 2
Q 100 / 9 25 1
VY = 2 = = V = × 4 × 10−6 × 1002
C3 4 9 2
25 2 E = 0.02 J
VX − VY = 3 − = V
9 9 179. When a 0.75 µF capacitor is charged to a
v X − VY = 0.22 V voltage of 20V, then the magnitude of charge
on each plate is ––––
177. Two circular plates each of radius 'r' and (a) 15 µC (b) 10 µC
charge ‘q’ form a parallel plate capacitor. Then
(c) 20 µC (d) 12 µC
the force of attraction between the plates is
AP EAMCET-24.08.2021, Shift-I
(Medium between the plate is air)
Ans. (a) : Given,
q q
(a) (b) C = 0.75µF = 7.5 × 10−7 F
2ε 0 r 2πε0 r 2
V = 20V
2πε0 r 2 q2
(c) (d)
q 2πε0 r 2
AP EAMCET-24.04.2019, Shift-II
Ans. (d) : Given,
Radius of Circular Plate = r For a Capacitor,
Area of Circular Plate = πr2 Q = C.V
We know that, Q = 7.5 × 10−7 × 20
σ
The electric field between the two plates = Q = 15 ×10−6 = 15µC
εo
180. Two capacitors of equal capacity are initially
σ uncharged. If one of the capacitors is connected
Electric field due to one plate =
2ε 0 across a battery of 10 V, the energy stored in it
is E. If the two capacitors are connected in
q
Surface charge density, σ = series across the battery. The total energy
A stored is
Now, (a) 4E (b) 2E
Force through electric field (F) = E × q E E
σ (c) (d)
∴ F= ×q 4 2
2εo AP EAMCET-28.04.2017, Shift-II

Objective Physics Volume-III 853 YCT


Ans. (d): Given, Electric field energy density,
V = 10 Volt UE 1
= ε0 E 2
According to question, V 2
Case-I, Where, V → volume
B2
UB 2µ 0
∴ =
UE 1 ε E2
0
2
 1 
U B B2 1 Q µ ε = C 
= 2×  0 0 
1 U E E µ 0ε0  1 = C2 
Energy stored (E) = CV2 µ ε 
2  0 0 
1 U B B2 C 2
= C(10)2 ....(i) =
2 UE E2
Case-II,
182. A 8 µF capacitor is charged by a 400 V supply
through 0.1 MΩ resistance. The time taken t.
the capacitor to develop a potential differences
300 V is (Given log10 4 = 0.602)
(a) 2.2 s
(b) 1.1 s
(c) 0.55 s (d) 0.48 s
AP EAMCET(Medical)-2009
Ans. (b) : Given that,
Capacitors are connected in series,
Capacitor ( C ) = 8µF = 8 × 10−6 F
1 1 1 2
= + = V = 400V
C' C C C
C Resistance (R) = 0.1 M Ω = 0.1×106Ω
C' = The charge on the capacitor,
2
Q = C.V
1
Total energy stored (E') = C'V2 Q = 8 × 10−6 × 400 .....(i)
2
1 C We know,
= × × (10)2 V 300
2 2 Q = I× t = × t = ×t .....(ii)
1 R 0.1×106
E' = E [from equation (i)] From equation (i) & (ii) we get,
2
300
181. The ratio of energy stored per unit volume in a 8 × 10−6 × 400 = ×t
solenoid having magnetic induction B to the 0.1× 106
electrostatic energy stored per unit volume in a 8 × 10−6 × 400 ×105
capacitor in electric field E is t=
300
B2 C B2 C 2 t = 1.06 s  1.1 s
(a) 2
(b)
E E2 183. A parallel plate capacitor with air as dielectric
BC2 B2 C 2 is charged to a potential V using a battery.
(c) (d)
E 2
E Removing the battery, the charged capacitor is
BC2 2 then connected across an identical uncharged
(e) parallel plate capacitor filled with wax of
2E 2
dielectric constant K. The common potential of
Kerala CEE 04.07.2022
both the capacitors is
Ans. (b) : We know that, (a) V volt (b) KV volt
Magnetic field energy density,
V
UB B2 (c) (K + 1) V volt (d) volt
= K +1
V 2µ 0 AP EAMCET(Medical)-2009
Objective Physics Volume-III 854 YCT
Ans. (d): The expression for capacitance with air as 185. A 2µF condenser is charged to 500 V and then
εA the plates are joined through a resistance. The
dielectric (C) = 0 heat produced in the resistance (in joule) is
d
(a) 0.10 J (b) 0.15 J
For identical capacitor capacitance with dielectric
(c) 0.25 J (d) 0.30 J
εA
C' = EAMCET-1993
d
Ans. (c) : Given that,
ε
Q C ' = KC where, K = C = 2 µF = 2×10–6F , V = 500 volt
ε0
Energy stored in the capacitor,
C and C ' are connected parallel.
1
So, E = CV 2
2
Resultant capacitance (CR) = C + C '
1
CR= C + KC = × 2 × 10−6 × (500)2
CR = C[K+1] 2
Total charge, (Q) = CRV ' = 25× 10 −2 J
Q = C(1+K)V' E = 0.25Joule
Q 186. Two capacitors 2µF and 4µF are connected in
V' = series and a potential of 100 V is applied across
C(1 + K)
the combination. Energy stored in the
CV capacitors is
= (Q Q = CV)
C(1 + K) (a) 0.0067 J (b) 0.067
V (c) 0.0033 J (d) 0.057 J
V' = volt
1+ K EAMCET-1996
184. Five equal capacitors connected in series have a Ans. (a) : Given,
resultant capacitance of 4µF. The total energy C1 = 2µF, C2 = 4µF, V = 100Volt
stored in these when these are connected in Capacitors are connected in series,
parallel and charged to 400 V, is 1 1 1
(a) 1 J (b) 8 J = +
Ceq C1 C 2
(c) 16 J (d) 4 J
EAMCET-1991 1 1 1
= +
Ans. (b) : Given, C eq 2 4
Ceq = 4µF, V = 400 Volt 1 2 +1
When capacitor are connected in series, =
Ceq 4
1 1 1 1 1 1
= + + + + 4
Ceq C C C C C Ceq = µF
3
1 5 Energy stored in the capacitors,
=
Ceq C 1
E = Ceq V 2
1 5 2
=
4 C 1 4
E = × × 10−6 × (100) 2
C = 20µF 2 3
When capacitor are connected in parallel 2
E = × 10−6 × 104
C′ = C + C + C + C + C 3
C′ = 5C E = 0.67× 10 −2
C ' = 5 × 20 = 100µF E = 0.0067 J
C′= 100 × 10 F –6
187. A body of capacity 4µF is charged to 80 V and
1 another body of capacity 6µF is charged to 30
Energy store in the system (E) = C '.V 2 V. When they are connected the energy lost by
2
4µF capacitor is
1 −6 (a) 7.8 mJ (b) 4.6 mJ
E = × 100 × 10 × 400 × 400
2 (c) 3.2 mJ (d) 2.5 mJ
E = 8Joule EAMCET-2001
Objective Physics Volume-III 855 YCT
Ans. (a): Given that, 1
(20 + 30) × ( 2 )
2
C1 = 4µF , V1 = 80V Uf =
2
C2 = 6µF, V2 = 30V 1
= × 50 × 4
Initial energy of body of capacitance = 4µF 2
Uf = 100 J
1
U i = C1V12 Decrease in energy = Ui – Uf
2 Decrease in energy = 250 – 100 = 150 Joule.
1 189. A parallel plate capacitor of capacity 100 µF is
= × 4 × 10−6 × (80)2
2 charged by a battery of 50 V. The battery
U i = 0.0128J remains connected and if the plates of the
capacitor are separated so that the distance
Final Potential on this body after connection between them becomes double the original
C V + C2 V2 distance, the additional energy given by the
V= 1 1
C1 + C 2 battery to the capacitor in joule is
125
4 × 80 + 6 × 30 (a) × 10−3 (b) 12.5 × 10–3
V= = 50 volt 2
4+6 (c) 1.25 × 10–3 (d) 0.125 × 10–3
1 EAMCET-2002
So final energy (U f ) = × 4 ×10−6 × ( 50 )
2

2 Ans. (a) :Given that,


1 C =100 µF V = 50volt s
= × 4 × 10−6 × 2500
2 1
Initial energy (U i ) = CV 2
Uf = 0.005 J 2
Energy lost by 4µF capacitor 1
= × 100 × 10−6 × ( 50 )
2

Ui – Uf = 0.0128 – 0.005 2
= 0.0078 U i = 125 × 10 −3 J
= 7.8 m Joule. When the battery is kept connected, voltage does not
188. A 20 F capacitor is charged to 5V and isolated. change
It is then connected in parallel with an εA
uncharged 30 F capacitor. The decrease in the Q C = 0 = 100µF
d
energy of the system will be When the distance is double
(a) 25 J (b) 100 J ε A 100
(c) 125 J (d) 150 J C' = 0 = = 50µF
2d 2
EAMCET-2001 So, final energy
Ans. (d) : Given, 1
C1 = 20F, C2 = 30F, V = 5 volt U f = C '× V 2
2
1 1
Initial energy (Ui) = C1V 2 = × 50 ×10−6 × ( 50 )
2
2 2
1
(Ui) = × 20 × ( 5)
2
1
2 = × 50 × 10−6 × 25 × 102
2
= 250 J 1
We know that Uf = × 125 × 10−3
2
Q 1 = C 1V
∴ Additional energy is
Q1 = 20 × 5
125 × 10−3
Q1 = 100 C U i − U f = 125 × 10−3 −
Q2 = (C1 + C2) V 2
Q2 = (20 + 30) V = 50 V 125 −3
= × 10 J
Now another capacitor is connected in parallel. So, total 2
charge is constant and voltage across the capacitors is 190. A parallel plate capacitor of capacity C0 is
same charged to a potential V0. (i) The energy stored in
Q1 = Q2 the capacitor when the battery is disconnected
100 = 50 V and the plate separation is doubled is E1. (ii) The
V = 2 volt energy stored in the capacitor when the charging
battery is kept connected and the separation
1 between the capacitor plates is doubled is E2.
Now, energy (Uf) = × (C1 + C2 ).V 2
2 Then, E1/E2 value is

Objective Physics Volume-III 856 YCT


(a) 4 (b) 3/2 E1 = 8 × 10 −2 J
(c) 2 (d) 1/2
E1 = .08J
EAMCET-2003
When capacitance is unchanged
Ans. (a) : Case -I
C 2 = 2µF = 2 × 10−6 F
When battery is disconnected charge on capacitor
εA Initial charge on capacitor C1 is equal to the final charge
C= 0 ….(i) on capacitors C1 & C2
d
When plate separation is double (2d) then, Q = C 1V 1
V2 (C1 +C2) = C1V1
εA
C1 = 0 V2 (4 + 2) × 10 −6 = 4 × 10 −6 × 200
2d
400
∴ C1 =
C
[From equation (i)] V2 = Volt
2 3
We know that, Energy for the combination of two capacitors is
Q = CV 1
E 2 = ( C1 + C2 ) V2 2
Q 2
V= ….(ii) 2
C 1  400 
Q = C1V 1 = × ( 2 + 4 ) × 10−6 ×  
2  3 
C
Q = V1 = 5.33 × 10 −2 J = 0.0533 J
2
2Q ∴ Amount of energy loss = E1 − E 2 = 0.08 − 0.0533
V1 =
C = 2.67 ×10−2 Joule.
V1 = 2V [ From equation (ii) ] 192. When a dielectric slab is introduced between
1 the plates of a capacitor connected to a battery,
∴ Store energy E1 = C1V12
2 then
1 C (a) charge on capacitor increases
E1 = . × 4V 2 = CV 2 ..... (iii)
2 2 (b) potential difference across the capacitor
Case -II increases
When battery is connected voltage does not change (V1 (c) energy stored increases
= V2) and the separation between plates is doubled. (d) capacity remains the same
εA C CG PET- 2015
C2 = 0 = [ From equation (i) ]
2d 2 Ans. (a) : Potential difference (V) of the capacitor
1 1 CV 2 remains constant suppose dielectric constant K is
∴ E 2 = C2 V22 = ( C / 2 ) V 2 = .....(iv)
2 2 4 introduced between the plates. Now, capacitance of the
Dividing equation (iii) with equation (iv), capacitor is-
E 1 CV 2 C' = KC0
∴ = =4 Where, C0 is original capacitance.
E 2 CV 2
4 Initial energy stored in the capacitor
1
191. A 4 µF capacitor is charged by a 200 V battery. U i = C 0V 2
It is then disconnected from the supply and is 2
connected to another uncharged 2µF capacitor. Final energy stored in the capacitor,
During the process, loss of energy (in J) is 1
(a) 3.43 × 10–2 (b) 2.67 × 10–2 Uf = C'V2
–4
2
(c) 2.67 × 10 (d) 3.43 × 10–4
1
EAMCET-2005 = (KC0)V2
2
Ans.(b) : Given that,
U
C1 = 4µF, V1 = 200V , C2 = 2µF Clearly, f = K > 1
Ui
Energy stored in C1 is given by
1 ⇒ Uf > Ui
E1 = C1V12 As, Q = CV
2
1 ∴ V remains constant and C increases
= × 4 × 10−6 × ( 200 )
2

2 ∴ Q increases

Objective Physics Volume-III 857 YCT


193. In an oscillating LC circuit when 75% of their 4ε 0 A εA
total energy is stored in the inductor's magnetic Cfinal = = 2 0
2d d
field, the charge on the capacitor as fraction of
Cfinal = 2Ci = 2×12µF
maximum charge on it is
(a) 0.2 (b) 0.25 Cfinal = 24µF
(c) 0.45 (d) 0.50 195. The radii of two spherical conductors A and B
TS EAMCET 28.09.2020, Shift-II are in the ratio 3 : 5 Conductor 'A' is in air
Ans. (d) : Given, 75% of total energy is stored in while B is surrounded by a medium of relative
inductor’s magnetic field permittivity 6. The ratio of the capacitances of
∴ Energy stored in capacitor 100 –75 = 25% A and B is ……..
Total energy of the LC circuit is given by (a) 1 : 10 (b) 3 : 5
(c) 18 : 5 (d) 5 : 6
Q2
E= AP EAMCET-23.09.2020, Shift-II
2C
Ans. (a) : Potential at any point of sphere,
∴ If UE = energy stored in capacitor
U = energy stored in LC circuit q
V=
Then, 4πε0 r
U E q 2 / 2C We know that,
= = 25%
U Q 2 / 2C q
Q C=
q 2 V
= 25% q
Q2 C= = 4πε0 r
q
q2 25
= 4πε0 r
Q 2 100
C = 4πε0r
q 5
= = 0.5 Ratio of radius of spherical conductor A & B = 3 & 5
Q 10
Capacitance of spherical conductor A placed in air
q C1 = 4πε0×3
= 0.5
Q Capacitance of spherical conductor surrounded by
Where, Q = charge on capacitor. permittivity
194. A parallel plate capacitor with air between the C2 = 4πε0×5×6
plates has capacitance 12 µF. If the distance The ratio of capacitance of A and B,
between the plates is doubled and the space C1 4πε0 × 3 C 1
between the plates filled with a substance of = ⇒ 1 =
C2 4πε0 × 6 × 5 C2 10
dielectric constant 4. The capacitance of the
capacitor will be is– 196. An air capacitor and identical capacitor filled
(a) 24 µF (b) 72 µF with dielectric medium of dielectric constant 5
(c) 6 µF (d) 12 µF are connected in series to a voltage source of
12V. The fall of potential across C1 and C2 are
AP EAMCET-08.07.2022, Shift-I
respectively
Ans. (a) : We know that, (a) 2 V and 10 V (b) 10 V and 2 V
εA (c) 6 V and 6 V (d) 4 V and 8 V
Q C= 0 ….(i)
d (e) 8 V and 4 V
Let, initially distance between plate = d Kerala CEE-2021
Initial capacitor = 12µF Ans. (b) : According to question,
εεA Capacitor (C) = C1 = C2
Ci = 0 r
d Dielectric constant (K) = 5
εA Voltage = 12 V
Ci = 0 …. (ii) (for air εr=1)
d
When distance between plates is doubled & dielectric
slab of dielectric constant 4 is insure
Then,
Kε0 A
Cfinal =
2d

Objective Physics Volume-III 858 YCT


C1KC2 198. Three identical capacitors, each with
Ceq = (Q C1 = C2 = C) capacitance C, are combined as shown in
C1 + KC2
figures A – D. If the same voltage is applied to
C2 K CK each combination, the one that stores the
Ceq = =
C + KC ( K + 1) greatest energy is
C × 5 5C
Ceq = =
1+ 5 6
Total charge Q flowing in circuit,
Q = Ceq. V
5 5
Q = C.V = × C ×12 = 10C
6 6
Q = 10 C
Potential across C1,
Q 10C
VC1 = = = 10V
C1 C
(a) A (b) B
Potential across C2, (c) C (d) D
Q 10C
VC2 = = = 2V J&K CET-1997
C 2 5C Ans. (c) : We know that,
VC1 = 10V 1
Energy, U= CV 2
VC2 = 2V 2
U∝ C
197. A parallel plate capacitor has uniform electric
field ‘E’ in the space between the plates. If the∴ Whose Cequ. is maximum the energy stored will be
distance between plates is ‘d’ and area of each maximum.
plate is ‘A’, the energy stored in the capacitor Among the all circuit Cequ. of circuit (C) will be
is (ε0 = permittivity of free space) maximum
Cequ. = C + C + C = 3C
1 ε0 EA
(a) ∴ Charge store in circuit C is maximum.
2 d
199. A source of potential difference V is connected
1 ε0 Ad
(b) to the combination of two identical capacitors
2 E2 as shown in the figure. When key 'K' is closed,
1 ε0 E 2 A the total energy stored across the combination
(c) is E1. Now key 'K' is opened and dielectric
2 d
1 constant 5 is introduced between the plates of
(d) ε0 E 2 Ad the capacitors. the total energy stored across
2 the combination is now E2. The ratio E1/E2 will
MHT-CET 2020 be:
Ans. (d) : The capacitance of parallel plate capacitor
εA
Q C= 0
d
Potential difference between the plates,
V
E=
d
V = Ed
1
∴ Total energy, U = CV 2 (a)
1
(b)
2
2 10 5
1ε A
U = 0 × ( Ed )
2
5 5
2 d (c) (d)
13 26
1 ε0 A
× ( Ed )
2
U= JEE Main-26.07.2022, Shift-II
2 d Ans. (c) : Given diagram,
1 ε0 A
U= × E 2d 2
2 d
1
U = ε0 AE 2 ⋅ d
2

Objective Physics Volume-III 859 YCT


When switch K is closed both the capacitor will  
charges. Energy store,  ε ( 7 × 4 ) 5ε (1× 4 ) 
1 Ceff . =  0 + 0  × 10−2
E1 = × 2CV 2 = CV 2   4  4 
2   10   
 10  
E1 = CV2
Ceff. = 1.2ε0
When switch is opened charge on right capacitor remain
1
CV while potential on left capacitor remain same. Energy = Ceff .V 2
Now when the capacitor is filled with dielectric of ‘k’ 2
1
then capacitance increase to ‘kC’ = (1.2 ) ε0 ( 20 ) × ( 20 )
Q2 2
∴ Energy = = 240ε0
2kC
Q = CV It is charge on capacitor B. 201. A parallel plate capacitor of plate area A has a
charge Q. The force on each plate of the
1
And capacitor A is charge to kCV 2 capacitor is–
2
2Q 2 Q2
∴ Total energy- (a) (b)
ε0 A ε0 A
1 Q2 1
E2 = + kCV 2 Q2
2 kC 2 (c) (d) zero
2ε 0 A
1 ( CV ) 1
2

E2 = + CV 2 k BCECE-2009
2 kC 2
2 2
Ans. (c) : The stored energy E = charge in work (W)
1C V 1 W = Fdx
E2 = + CV 2 k
2 kC 2 Q2
1 1  Energy store in capacitor, E = .....(i)
E2 = CV 2  + k  2C
2  k  εA
1 + k 2  But, C = o
1 x
E 2 = CV 2  
2  k  Put in equation (i),
Hence, the ratios of E1/E2 is- Q2 x
∴E=
E1 CV 2 2ε 0 A
=
E 2 CV 2 (1 + k 2 ) W = ∫ Fdx
2k dW
E1 2k F=
= (Q k = 5) dx
E2 1 + k2
d  Q2 x 
E1 2×5 2×5 5 F=  
= = = dx  2ε0 A 
E 2 1 + 25 26 13
Q2
E1 5 F=
= 2ε 0 A
E 2 13
202. In parallel plate capacitor, electric field
200. A parallel plate capacitor with width 4 cm,
between the plate is ‘E’. If the charge on the
length 8 cm and separation between the plates
plates is ‘Q’ then the force on each plate is
of 4mm is connected to a battery of 20 V. A
(a) QE (b) QE2/2
dielectric slab of dielectric constant 5 having
QE
length 1 cm, width 4 cm and thickness 4 mm is (c) (d) QE2
inserted between the plates of parallel plate 2
capacitor. The electrostatic energy of this MHT-CET 2019
system will be ..........εoJ (Where εo is the Ans. (c) : Work done in increasing the separation of the
permittivity of free space) plate,
JEE Main-27.07.2022, Shift-II dW = F.dx .....(i)
Ans. (240) : According to question- ∴Increases in P.E. of capacitor = F.dx
If U is the energy stored per unit volume of capacitor,
ε A ε A 
Ceff . =  0 1 + 0 2  Increases in P.E. of capacitor = U × increases in volume
 d1 d2  = U × A.dx .....(ii)

Objective Physics Volume-III 860 YCT


From equation (i) and (ii), Initial energy stored in capacitor,
F.dx = UA.dx 1
Ui = CV2
1  2
F = UA =  εo E 2  A
2  1
Ui = × 5 × (2)2 = 10 µJ
1 1 V 2
= ( εo AE ) E =  ε o A  E
2 2 d When, supply is disconnected and add discharge
capacitor
 V
Q E = d  Cf = 5 + 5 = 10 µF
Two capacitor attain a common potential,
1
F = ( CV ) E Total charge (Q)
2 Vf =
Total capacitor (Cf )
1
F = QE 10
2 Vf = = 1V
203. Energy per unit volume for a capacitor having 10
area A and separation d kept at potential Energy stored in the capacitor, after combination
difference V is given by 1
Uf = Cf Vf2
1 V2 1 V2 2
(a) ε0 2 (b)
2 d 2ε0 d 2 1
Uf = × 10 × 1 = 5 µJ
2
1 Q2
(c) CV 2 (d) ∴ Electrostatic energy lost in the process = Vi – Vf
2 2C
= 10 – 5
MHT-CET 2011
= 5 µJ
J&K CET - 2002
206. The potential energy of a charged parallel plate
1
Ans. (a) : Energy density = εo E 2 capacitor is U0. If a slab of dielectric constant K
2 is inserted between the plates, then the new
1  V2   V potential energy will be
= εo  2  Q E = d 
2 d  U0
(a) (b) U0K2
204. A capacitor of capacitance 10µF is charged to K
10 V. The energy stored in it is : U0
(a) 1 µJ (b) 100 µJ (c) (d) U 02
K2
(c) 500µJ (d) 1000µJ MP PMT-2009
Karnataka CET-2012
Ans. (a) :
Ans. (c) : Given,
Capacitor of capacitance (C) = 10µF
Potential difference (V) = 10V
1
Energy stored (E) = CV 2
2
1 The potential energy of charged parallel plate capacitor,
= × 10 × (10 )
2

2 Q2
U0 =
= 500µJ 2C
205. A 5.0 µF capacitor is charged by a 2.0 V When dielectric constant inserted between plates,
battery. It is then disconnected from the Then, C' = KC
battery and is connected to another uncharged Q2
5.0 µF capacitor. The loss is the electrostatic New potential energy (U') =
2C '
energy in the process is
(a) 10 µJ (b) 5 µJ Q2
U' = (QC' = KC)
(c) 7.5 µJ (d) 0 2KC
TS EAMCET 30.07.2022, Shift-II 1  Q2 
U' = × 
Ans. (b) : Given, K  2C 
Capacitor (C) = 5 µF
U0
Potential difference (V) = 2V ∴ U' =
Total charge Q = CV = 5 × 2 = 10µC K

Objective Physics Volume-III 861 YCT


207. In the given circuit find the heat produced Initial charge on capacitor,
between A and B (Given C = 1µF) εA
Q = CV = 0 V
d
When dielectric slab is inserted then capacitance,
εA
C′ = kC = k o
d
Q V
Q V′ = =
C' k
∴ Initial potential energy stored in capacitor.
1
(a) 50µJ (b) 60µJ Ui = CV 2
(c) 70µJ (d) 100µJ 2
JIPMER-2019 1 ε0 A 2
Ui = V
Ans. (a) : Given, 2 d
2
C = 1µF = 1 × 10–6 F 1 1  kAε0  V  ε0 AV 2
Uf = C'V '2 =    =
V = 10 volt 2 2  d  k  2kd
1 ∴ Work done on the system,
Heat produced in capacitor = CV 2
2 W = ∆U = U i − U f
1
= × 10–6 × (10)2 ε AV 2 ε0 AV 2
2 W= 0 –
= 50 µJ 2d 2kd
208. A circuit has a self - inductance of 1 H and ε AV 2
 1
W= 0 1 – 
carries a current of 2 A. To prevent sparking, 2d  k 
when circuit is broken, a capacitor which can
ε × 10 × 10 –4 × 12 ×12  1 
withstand 400 V is used. The least capacitance W= 0 1 – 
of the capacitor is 2 × 3 ×10 –3  3
(a) 125 µF (b) 25 µF 1440 ×10 –4
2
W = ε 0× ×
(c) 100 pF (d) 50 µF 2 × 3 × 10 –3
3
Assam CEE-2016 W = 16ε0
Ans. (b) : Energy stored in capacitor = energy stored in αεo = 16ε0
inductance, α = 16
1 2 1 2 210. If the charge on a capacitor is increased by 2C,
CV = LI
2 2 the energy stored in it increases by 44%. The
2
LI 2 1×(2) original charge on the capacitor is (in C)
C= 2 = 2
= 25 µF (a) 10 (b) 20
V (400)
(c) 30 (d) 40
209. A parallel – plate capacitor of plate area 10 cm2 JEE Main-24.06.2022, Shift-II
and plate separation 3 mm is charged to a
Ans. (a) : Let, initial charge on capacitor Q.
potential difference 12V and then the battery is
The energy stored in the capacitor is given by
disconnected .A slab of dielectric constant 3 is
then inserted between the plates. The work Q2
U=
done on the system in the process of inserting 2C
in the slab is αε0. The value of α is ∴ U ∝ Q2
(Take ε0 as the permittivity of free space) 2
U2  Q2 
(a) 8 (b) 12 ∴ = 
(c) 16 (d) 18 U1  Q1 
TS EAMCET 19.07.2022, Shift-I  Q2 
2
U2
Ans. (c) : Given, – 1 =   –1
U1  Q1 
Area of plate capacitor = 10 cm2
Distance between plate = 3mm U 2 – U1 Q 22
Capacitor charged with potential = 12 V ∴ = 2 –1
U1 Q1
Dielectric constant of slab = k
Objective Physics Volume-III 862 YCT
U 2 – U1  Q2  Now, dielectric slab of dielectric constant K is
× 100 =  22 – 1 × 100 introduced between the plates then it becomes KC
U1  Q1  ∴ Energy stored will become K times,
 Q2  1
44 =  22 – 1 × 100 E1 = KCV 2
 Q1  2
2 E1 = K E
Q2
– 1 = 0.44 Surface charge density,
Q12 σ = Q/A
Q 22 Now, charge become Q' = Q × K
= 1 + 0.44 = 1.44
Q12 σ' = K Q/A
Q2 σ=Kσ
= 1.2 Now, surface charge density becomes K times.
Q1
So, Assertion is true but reason is false.
Q2 = 1.2Q1
Given if charge increase by 2C, 213. A 5.0 µF capacitor is charged to a potential
Q 2 = Q1 + 2 difference of 800 V and discharged through a
1.2Q1 = Q1 + 2 conductor. The energy given to the conductor
1.2 Q1 – Q1 = 2 during the discharge is
0.2 Q1 = 2 (a) 1.6×10–2 Joule (b) 3.2 Joule
Q1 = 10C (c) 1.6 Joule (d) 4.2 Joule
211. The capacity of a condenser is 20 ×10–6 F and AIIMS-26.05.2019(E) Shift-2
the potential is 20 V. The energy released on Ans. (c) : Given,
discharging it fully will be Capacitance (C) = 5.0 µ F = 5 × 10−6 F
(a) 2×10–3J (b) 4 × 10–3J Potential difference (V) = 800 V
(c) 8 × 10 J –3
(d) 10 × 10–3J We know that,
UPSEE - 2010 1
Ans. (b) : Given, E = CV 2
2
The capacity of a condenser (C) = 20 × 10–6 F
1
= × 5 × 10−6 × ( 800 )
2
The potential (V) = 20 Volt
The energy released on discharging will be 2
1 1
U = CV 2 = × 5 × 10−6 × 800 × 800
2 2
1 1
= × 20 ×10−6 × 20 × 20 = × 5 × 64 × 10−6 ×104
2 2
1 = 32 × 5 × 10–2
= × 8000 × 10−6
2 = 160 × 10–2
= 4 × 10–3J. E = 1.6 Joule
212. Assertion: A parallel plate capacitor is Hence, the energy given to the conductor during the
connected across battery through a key. A discharge
dielectric slab of dielectric constant k is E = 1.6 Joule
introduced between the plates. The energy
214. A coil of resistance 10 Ω and an inductance 5 H
stored becomes k times.
is connected to a 100 V battery. The energy
Reason: The surface density of charge on the
plate remains constant. stored in the coil is
(a) If both assertion and reason are true and (a) 125 erg (b) 125 J
reason is the correct explanation of assertion. (c) 250 erg (d) 250 J
(b) If both assertion and reason are true but UPSEE - 2010
reason is not the correct explanation of Ans. (d) : Given,
assertion. R = 10Ω , L = 5H , Voltage = 100V
(c) If assertion is true but reason is false. V 100
(d) If both assertion and reason are false. Steady state current (Io) = = = 10A
AIIMS-26.05.2019(E) Shift-2 R 10
Ans. (c) : When capacitor is connected with battery 1
∴ Energy stored in the coil (U) = L Io2
then energy stored in the capacitor 2
1 1
E = CV 2 = × 5 × (10) 2 = 250 J
2 2

Objective Physics Volume-III 863 YCT


Then,
(c) Combination of Capacitor Charge of the capacitor, Q = C . V
Charge on capacitor C1 is Q1 = C1V
215. The equivalent capacitance of the combination = 2 × 10 = 20 µC
shown is Charge on capacitor C2 is Q2 = C2V
= x × 10
= 10x µC
Similarly, charge on capacitor C3 is Q3 = C3V
= 3 × 10
= 30 µC
Total charge stored in the combination of capacitors,
C = Q1 + Q2 + Q3
(a) (b) 4 C = 20µC + 10xµC + 30µC ....(ii)
2
5 From equation (i) and (ii)
(c) 2 C (d) C = 20µC + 10xµC + 30µC = 100µC
3
JEE Main-10.04.2023, Shift-I 50µC + 10xµC = 100µC
Ans. (c) : Since the potential at point A is equal to the 10xµC = 100 µC – 50 µC
potential at point B, no current will flow along Arm AB, 10xµC = 50 µC
hence, the capacitor on arm AB will not contribute to 10x = 50
the circuit. Also because the remaining two capacitor 50
are connected in parallel the net capacitance in the x=
10
circuit can be reduced to x=5
217. Two parallel plate capacitors C1 and C2 each
having capacitance of 10 µF are individually
charged by a 100 V D.C. source. Capacitor C1
is kept connected to the source and a dielectric
slab is inserted between it plates. Capacitor C2
is disconnected from the source and then a
dielectric slab is inserted in it. Afterwards the
capacitor C1 is also disconnected from the
source and two capacitors are finally connected
Parallel combination, in parallel combination. The common potential
Ceq = C + C = 2C of the combination will be ____ V.
216. In the given figure the total charge stored in (Assuming Dielectric constant =10)
the combination of capacitors is 100 µC. The JEE Main-31.01.2023, Shift-II
value of ‘x’ is ______ . Ans. (55) :

JEE Main-15.04.2023, Shift-I


Ans. (5) : Given that, C1 = 2µF Since, capacitors C1 and C2 are finally connected in
C2 = xµF parallel combination, therefore, voltage is equally
C3 = 3µF divided in each capacitor.
Combination of capacitor = 100 µC ....(i) So, common voltage,
Totalcharge 1000 + 10000
We know that, V= =
Q Totalcapacitance 100 + 100
Capacitance, C =
V = 55 V

Objective Physics Volume-III 864 YCT


218. The four capacitors, each of 25 µF are
connected as shown in fig. The dc voltmeter
reads 200 V. The charge on each plate of
Capacitor is

(a) 4/7 (b) 3/22


(c) 7/4 (d) 22/3
MHT-CET-2020
(a) ±2 × 10 −3 C AIIMS-2012
(b) ±5 × 10 −3 C Ans. (b) : From question, equivalent capacitance for
(c) ±2 × 10 −2 C capacitors in series are –
(d) ±5 × 10 −2 C
AIIMS-2015
Ans. (b) : Given figure,

1 1 1 1
= + +
Ceq C1 C 2 C3
1 1 1 1
= + +
Ceq C 2C 3C
1 6 +3+ 2
Capacitance of each capacitor = 25µF =
Ceq 6C
Net capacitance in loop 1 (parallel connection)
C ' = C1 + C 2 = 25 + 25 = 50 µF 6C
Ceq =
Net Capacitance in loop 2 (parallel connection) 11
Charge on capacitors (in series )
C" = C1 + C 2 = 25 + 25 = 50 µF
Q = Ceq V
Net Capacitance in whole circuit (series connection)  6C 
1 1 1 Q=   V.
= +  11 
C"' C ' C '' Charge on capacitor Q4 = C4 V
1 1 1 1 = 4 CV
= + =
C ''' 50 50 25 6C
V
C ''' = 25 µF Charge on C2 11 6 1 3
Hence, = = × =
Charge on C4 4CV 11 4 22
C"' = 25 × 10–6 F
Q 220. Two capacitors each having a capacitance
Since, C"' = 2×10−6 F and a breakdown voltage 5000 V, are
V joined in series. What will be the resultant
Q = C''' × V capacitance and the breakdown voltage of the
Q = 25 × 10–6 × 200 combination?
Q = ± 5 × 10–3C (a) 4 × 10−6 F & 1,000 V
219. A network of four capacitors of capacity equal (b) 10−6 F & 10,000 V
to C1 = C, C2 = 2C, C3 = 3C and C4 = 4C are (c) 2 × 10−6 F & 5,000 V
conducted to a battery as shown in the figure . (d) 10−6 F & 2,500 V
The ratio of the charges on C2 and C4 is AP EAMCET-25.08.2021, Shift-I

Objective Physics Volume-III 865 YCT


Ans. (b) : Given,
C = 2 × 10–6 F
V = 5000 V
In series Combination
CC
Cnet = 1 2
C1 + C2
C 2 × 10−6
Ceq = = = 10−6 F
2 2
Vcombination = V1 + V2 = 2V = 2 × 5000 = 10000 V.
221.A circuit is shown in the following figure for
which C1 = (3 ± 0.011) µF, C2 = (5 ± 0.01) µF
and C3 = (1 ± 0.01) µF. If C is the equivalent
capacitance across AB, then C is given by: (a) 5µF
(b) 1µF
(c) 3µF
(d) 4µF
AP EAMCET-28.04.2017, Shift-II
Ans. (d) : Equivalent capacitance of Capacitors which
(a) (0.9 ± 0.114) µF (b) ( 0.9 ± 0.01) µF are in series,
(c) ( 0.9 ± 0.023) µF (d) ( 0.9 ± 0.09 ) µF 1 1 1 1
= + + .......... +
TS EAMCET 05.08.2021, Shift-II Ceq C1 C2 Cn
Ans. (c) : Given that, C1 = (3 ± 0.011) µF For C1 = C2 = .........Cn = 2 µF
C2 = (5 ± 0.01) µF Ceq =
C
C3 = (1 ± 0.01) µF n
For first row CA = C
From the circuit,
C
C' =
( 3C1 + C2 ) C3 For second row CB =
2
3C1 + C2 + C3 C
For third row CC = and so on .
C' =
( 3 × 3 + 5) ×1 = 14 4
5 + 3× 3 +1 15 From figure all the capacitor are in parallel.
C' = 0.9µF Cfinal = CA + CB + CC + .......... + C∞
and, C C C C
Cfinal = C + + + + .......... +
∆C  ∆C1 ∆C2 ∆C3   ∆C1 ∆C2 ∆C3  2 4 8 ∞
= + + + + + 
C '  C1 C2 C3   C1 C2 C3  C
Cfinal = = 2C
1
∆C ∆C ∆C2 ∆C3 1−
= 2 1 +2 +2 2
C' C1 C2 C3
Cfinal = 2 × 2 × 10–6 (Q C = 2 µF )
∆C  0.011 0.01 0.01  Cfinal = 4 µF
= 2 + +
C'  3 5 1  223. Three capacitors each of capacitance C and of
  0.011 0.01 0.01   breakdown voltage V are joined in series. The
∆C =  2  + +  × C ' capacitance and breakdown voltage of the
  3 5 1   combination will be
∆C = 0.023µF C V V
(a) , (b) 3C,
∴ C = ( 0.9 ± 0.023) µF 3 3 3
222. An infinite number of capacitors each of capacity C
(c) ,3V (d) 3C, 3V
2µF are connected as shown in the figure. The 3
resultant capacity between A and B is AIPMT-2009

Objective Physics Volume-III 866 YCT


Ans. (c): In series, combination of capacitors Ans. (d) : Given that, V1 = 120 V, V2 = 200 V
Veff = V + V + V = 3V
1 1 1 1
And, = + +
Ceff C C C
1 3
=
Ceff C
C
Ceff =
3
Thus, the capacitance and break down voltage of the
combination will be C/3 and 3V. Potential is zero for each capacitor, then their charge
224. The equivalent capacitance of the combination must be same,
shown in the figure is Q1 = Q2
C1V1 = C2V2
C1 × 120 = C2 × 200
3C1 = 5C2
227. A capacitor of capacity 10µF is charged to 40 V
and a second capacitor of capacity 15µF is
charged to 30 V. If they are connected in
(a) 3C (b) 2C parallel the amount of charge that flows from
C 3C the smaller capacitor to higher capacitor in µC
(c) (d) is:
2 2
NEET-2021 (a) 320 (b) 60
Ans. (b) : In this circuit one capacitor is shorted. So, the (c) 200 (d) 250
remaining two capacitor are in parallel. AP EAMCET(Medical)-2002
Given, C1= C2 = C Ans. (b) : Initially, the charge on first capacitance,
So, the equivalent capacitance is - Q1 = C1V1
Ceq = C1+ C2 = 2C Q1 = 10 × 40× 10–6 = 400µC
225. A parallel plate capacitor has 91 plates, all are The charge on second capacitance, Q2 = C2V2
identical and arranged with same spacing
between them. If the capacitance between Q2 = 15 × 30× 10–6 = 450µC
adjacent plates is 3 pF. What will be the When the capacitors are connected in parallel, potential
resultant capacitance? across them will be same.
(a) 273pF (b) 30pF Thus, the common potential
(c) 94pF (d) 270pF Q + Q 2 400 + 450 850
AP EAMCET(Medical)-2015 V= 1 = = = 34V .
C1 + C 2 10 + 15 25
Ans. (d) : Given,
Finally charge on capacitor are.
n = 91, C = 3 pF
When n plates are arranged with the same spacing Q' = C1V = 10 × 34 = 340µC
between them, the combination behaves as (n – 1) Q'' = C2V = 15 × 34 = 510µC
capacitor in parallel since charge on each plate is same. Net charge flow = Q" – Q2
So, equivalent capacitance is Ceq = C(n – 1) ∆Q = (510 – 450) × 10–6
= 3 (91 – 1) ∆Q = 60 µC
= 270 pF 228. The capacities of three capacitors are in ratio
226. Two capacitors having capacitances C1 and C2 1:2:3 their equivalent capacity when connected
are charged with 120 V and 200 V batteries 60
respectively. When they are connected in in parallel is µF more than that.
11
parallel now, it is found that the potential on
each one of them is zero. Then, When they are connected in series. The
(a) 5C1=3C2 (b) 8C1=5C2 individual capacitors are of capacities in µF:
(c) 9C1=5C2 (d) 3C1=5C2 (a) 4,6,7 (b) 1,2,3
AP EAMCET(Medical)-2013 (c) 2,3,4 (d) 1,3,6
BITSAT-2012 AP EAMCET(Medical)-2002
Objective Physics Volume-III 867 YCT
Ans. (b): From question, 230. The capacity of a parallel plate capacitor with
When, capacitor are connected in parallel- no dielectric but with a separation 0.4 cm is
Cp = x + 2x +3x 2µF. The separation is reduced to half and it is
Cp = 6x …... (i) filled with a dielectric of value 2.8. The final
When, capacitor are connected in series- capacity of the capacitor is:
1 1 1 1 (a) 11.2µF (b) 5.6µF
= + + (c) 4.0µF (d) 22.4µF
Cs x 2x 3x
AP EAMCET(Medical)-1997
1 11
= Ans. (a) : Given,
Cs 6x
εA
6x C = 0 = 2µF {d = 0.4 cm}
Cs = .... (ii) d
11
Then, new capacitance-
According to question,
Kε 0 A
60 C' =
Cp = + Cs .... (iii) d
11
2.8ε 0 A
Putting the value of equation (i), (ii) in (iii), we get− C' =
60 6x d/2
6x = +
11 11 εA
C' = 5.6 0
66x – 6x = 60 d
60x = 60 C' = 5.6 × 2× 10–6
x=1 C' = 11.2 µF
So, C1 = 1µF, C2 = 2µF, C3 = 3µF 231. Two identical capacitors are first connected in
229. Three capacitors 3µF, 10µF and 15µF are series and then in parallel. The ratio of
connected in series to a voltage source of 100 V. equivalent capacitance is
The charge on 15µF is : (a) 1 : 1 (b) 1 : 2
(a) 22µC (b) 100µC (c) 1 : 3 (d) 1 : 4
(c) 2800µC (d) 200µC J&K-CET-2013
UP CPMT-2001
Ans. (d) : Given,
AP EAMCET(Medical)-1998
Two capacitor are identical i.e. C1 = C, C2 = C
Ans. (d) : Figure,
When, the capacitor connected in the series-
1 1 1
= +
Ceq C1 C 2
1 1 1
= +
Ceq C C
C
Ceq =
2
Three capacitors are connected in series. So, equivalent When, the capacitor connected in parallel-
capacitance are-
C' = C + C
eq 1 2
1 1 1 1
= + +
Ceq 3 10 15 C'eq = C + C
1 10 + 3 + 2 15 C'eq = 2C
= =
Ceq 30 30
So, the ratio of the series and the parallel
1 1 C
=
Ceq 2 Ceq
= 2
'
Ceq 2C
Ceq = 2µF
Then, charge of Ceq is q = Ceq × V Ceq 1
q = (2) × (100) × 10–6 =
q = 200 µC C'eq 4
Since, in series all the capacitor have same charge. Hence, Ceq : C'eq = 1 : 4
Hence, charge on 15µF is 200 µC.
Objective Physics Volume-III 868 YCT
232. What is the total capacitance of the
combination when 3 capacitors each of
capacitance 9 pF are connected in series?
(a) 3 pF (b) 1/3 pF
(c) 4 pF (d) 1/4 pF
J&K-CET-2019
Ans. (a) : Figure,

JEE Main-27.06.2022, Shift-II


Ans. (23) : The equivalent capacitance of capacitor (In
Given, three capacitor are identical each capacitance is parallel combination)
9 pF εA εA εA
So, the equivalent capacitance of capacitor which is Ceq = 0 + 0 + 0
5b 3b b
connected in series.
 1 1 1
1 1 1 1 Ceq = ε 0 A  + + 
= + +  5b 3b b
Ceq C1 C 2 C3
23ε 0 A
Ceq = ......(i)
1 1 1 1 15b
= + +
Ceq 9 9 9 According to question-
1 3 1 xε A
= = Ceq = 0 .....(ii)
Ceq 9 3 15 b

Ceq = 3pF On comparing equation (i) and (ii), we get-


x = 23
233. A capacitor is connected to a cell of emf E
235. Two capacitors having capacitance C1 and C2
having some internal resistance r. The potential
respectively are connected as shown in figure.
difference across the
Initially, capacitor C1 is charged to a potential
(a) cell is < E (b) cell is E difference V volt by a battery. The battery is
(c) capacitor is > E (d) capacitor is < E then removed and the charged capacitor C1 is
UP CPMT-2004 now connected to uncharged capacitor C2 , by
Ans. (b) : closing the switch S. The amount of charge on
the capacitor C2 after equilibrium is:

Potential difference across capacitor in steady state will


be E when it is fully charged and as there is no current C1C2
(a) V
in steady state. Potential difference across the cell will ( C1 + C2 )
be E.
234. A parallel plate capacitor is made up of stair (b)
( C1 + C2 ) V
like structure with a plate area A of each stair C1C2
and that is connected with a wire of length b, as (c) ( C1 + C2 ) V
shown in the figure. The capacitance of the
x ε0 A (d) ( C1 − C2 ) V
arrangement is , the value of x is ____ ?
15 b JEE Main-26.06.2022, Shift-I
Objective Physics Volume-III 869 YCT
Ans. (a): Initially total charge (Qtotal) = C1 V Then, figure -
Total capacitance (Ctotal) = C1 + C2
The amount of charge on capacitor C2 after equilibrium
is-
C × Q total
= 2
C total
C1C 2
= V
( C1 + C2 ) Since, capacitor C1, C2 and C3 are connected in parallel-
236. The charge on capacitor of capacitance So, C' = C1 + C 2 + C3
15 µF in the figure given below is: C' = 8 + 8 + 8
C' = 24µF
Then, redrawn figure -

From figure, capacitor C' and 8µF are connected in


series.
So, equivalent capacitance-
(a) 60µC
1 1 1
(b) 130µC = +
Ceq C ' 8
(c) 260µC
(d) 585µC 1 1 1
= +
JEE Main-26.06.2022, Shift-II Ceq 24 8
Ans. (a) : From given figure , 1 24 + 8
=
Three capacitor are connected in series, then equivalent Ceq 24 × 8
capacitance of capacitor in series-
24 × 8
1 1 1 1 Ceq = = 6µF
= + + 32
Ceq C1 C 2 C3
Hence, Ceq = 6µF
1 1 1 1
= + + 238. Two parallel plate capacitors of capacity C and
Ceq 10 15 20 3C are connected in parallel combination and
1 6+4+3 charged to a potential difference 18V. The
= battery is then disconnected and the space
Ceq 60
between the plates of the capacitor of capacity
60 C is completely filled with a material of
Ceq = µF dielectric constant 9. The final potential
13
Charge (Q) = Ceq ×V difference across the combination of capacitors
will be _______ V.
60
= × 13 JEE Main-25.07.2022, Shift-II
13
Ans. (8) : From given question -
= 60 µC C
So, charge on each capacitor is same because they are in
series.
237. The equivalent capacitance between points A
and B in below shown figure will be _______ 3C
µF.

18V
We know -
JEE Main-25.06.2022, Shift-I Initial charge (Q) = Ceq × V {Ceq = C + 3C =4C}
Ans. (6) : Two capacitor are short circuited = 4C × 18 V
C = C = C = 8µF = 72 CV
1 2 3

Objective Physics Volume-III 870 YCT


C2 and C3 are connected in series, so equivalent
capacitance.
1 1 1
= +
C'' C3 C 4
1 1 1
= +
C" 2 6
6× 2 6
C'' = = µF
Then, 8 4
Again redrawn figure -
Final charge (Q') = C'eq × V ' = KC × V'
3
= 9CV' µF
4
Q Q = Q'
72 CV = 9CV'
⇒ V' = 8 V
239. The capacitance between the point A and B in
the following figure.

C' and C'' are connected in parallel.


So, the equivalent capacitance between point A to B is
Ceq = C' + C''
3 6
= +
3 4 4
(a) µF
8 9
= µF
9 4
(b) µF
4 240. The four capacitors of 25 µF each are
4 connected as shown in the figure below. If the
(c) µF
5 D.C. voltmeter read 200 V. The charge on each
(d) 2µF plate of the capacitor is ……… .
AP EAMCET-05.07.2022, Shift-I
Ans. (b) : From given figure -

C1 and C2 are connected in series so, equivalent (a) 2 × 10–3 C (b) 5 × 10–3 C
–2
capacitance is (c) 2 × 10 C (d) 5 × 10–2 C
1 1 1 AP EAMCET-23.09.2020, Shift-II
= +
C ' C1 C 2 Ans. (b) : The two capacitors on the left side are in
parallel and potential across each equal to similarly the
1 1 1
= + capacitors on right side are in parallel and potential
C' 1 3 across each equal to V.
3 ×1 Therefore charge on each plate of capacitors is
C' =
3 +1 Q = CV
3 Q = 25 × 10–6 × 200
C ' = µF
4 Q = 5 × 10–3 C

Objective Physics Volume-III 871 YCT


241. In the circuit diagram shown in the adjoining Ans. (c) : We know,
figure the resultant capacitance between P and 1
Electrostatic energy of capacitor (E) = CV 2
Q is 2
1
And, C∝
d
C1 d 2 2d
= = =2
C 2 d1 d
E1 C1 2
= =
E 2 C2 1
E1
E2 =
2
(a) 47µF (b) 3µF Since, E1 = E
E
(c) 60µF (d) 10µF So, E 2 =
2
AP EAMCET-24.09.2020, Shift-I
243. In the given circuit, if the potential difference
Ans. (b) : From given figure, between A and B is 80 V, then the equivalent
Redrawn the circuit- capacitance between A and B, and the charge
on 10µF capacitor respectively, are____

(a) 4µF & 133 µC (b) 164 µF & 150 µC


(c) 15 µF & 200 µC (d) 4 µF & 50 µC
AP EAMCET-19.08.2021, Shift-I
Ans. (a) : From given figure ,
Where, C = (3 + 2) µF = 5µF
Redrawn the circuit –
equivalent capacitance between P and Q are
1 1 1 1
= + +
C PQ 5 20 12

1 12 + 3 + 5
=
C PQ 60

60
C PQ =
20
C PQ = 3 µF

242. The energy of a parallel plate capacitor when
connected to a battery is E. With the battery Since, 12µF, 24µF and 8µF are in series.
still in connection, if the plates of the capacitor 1 1 1 1
So, = + +
are separated, so that the distance between Ceq 12 24 8
them is twice the original distance, then the 1 2 +1+ 3
=
electrostatic energy becomes C 24
eq
E 1 6 1
(a) 2E (b) = =
4 Ceq 24 4
E Ceq = 4µF
(c) (d) 4E
2 Charge, Q = Ceq × V
AP EAMCET -2015 = 4 × 80 × 10–6 = 320 µC

Objective Physics Volume-III 872 YCT


Charge in 10 µF capacitor, So, voltage across C1 is
10 Q
Q' = × 320 V1 =
10 + 5 + 9 C1
10 150
= × 320 V1 = = 75V
24 2
= 133.33 µC. Therefore, Least potential difference is across C3.
 133 µC  30 
Equivalent capacitance of combination is   µF and
244. Three capacitors of capacitances C1 = 2µF, C2 =  31 
3µF and C3 = 5µF are connected in series. A the voltage across C1 is 75 V.
potential difference of 155 V is applied across 245. Assertion : Charges are given to plates of two
the combination. Choose the correct option. plane parallel plate capacitors C1 and C2 (such that
C2 =2C1) as shown in figure. Then the key K is
(a) Least potential difference is across C3.
pressed to complete the circuit. Finally the net
Equivalent capacitance of combination is charge on upper plate and net charge the circuit.
 30  Finally the net charge on upper plate and net
  µF and the voltage across C1 is 75 V. charge on lower plate of capacitor C1 is positive
 31 
(b) Least potential difference is across C1.
Equivalent capacitance of combination is
 30 
  µF and the voltage across C2 is 50 V.
 51 
(c) Least potential difference is across C1.
Equivalent capacitance of combination is
 30 
  µF and the voltage across C3 is 30 V.
 31 
(d) Least potential difference is across C2.
Equivalent capacitance of combination is
Reason : In a parallel plate capacitor both
 30 
  µF and the voltage across C1 is 50 V. plates always carry equal and opposite charge.
 31  (a) If both Assertion and Reason are correct and
AP EAMCET (17.09.2020) Shift-II the Reason is a correct explanation of the
Assertion.
Ans. (a) :
(b) If both Assertion and Reason are correct, but
Reason is not a correct explanation of the
Assertion.
(c) If Assertion is correct but Reason is incorrect.
(d) If both the Assertion and Reason are
incorrect.
AIIMS-2012
Given Ans. (d) : Since the magnetic field inside conducting
C1 = 2 µF,C 2 = 3 µF,C3 = 5 µF plates must be zero, and in parallel plate capacitor, both
the plates must have equal and opposite charge.
Equivalent capacitance in series combination given by:- Hence, If both the Assertion and Reason are incorrect.
1 1 1 1 246. Assertion: If three capacitors of capacitances C1 <
= + +
Ceq C1 C 2 C3 C2 < C3 are connected in parallel then their
equivalent capacitance Cp > Cs.
1 1 1 1
= + + 1 1 1 1
Ceq 2 3 5 Reason: = + +
C p C1 C 2 C3
30
Ceq = µF (a) If both Assertion and Reason are correct and
31 the Reason is a correct explanation of the
Q = Ceq × V Assertion.
30 (b) If both Assertion and Reason are correct but
Q = ×155 × 10–6
31 the Reason is not a correct explanation of the
Q = 150 µC Assertion.

Objective Physics Volume-III 873 YCT


(c) If both the Assertion is correct but Reason is 248. The equivalent capacitance between the points
incorrect. A and B in the following circuit is :
(d) If both the Assertion and Reason are
incorrect.
(e) If the Assertion is incorrect but the Reason is
correct.
AIIMS-2002 (a) 1 µF (b) 2 µF
Ans. (c) : In a parallel plate, capacitance will increase. (c) 4 µF (d) 8 µF
Cparallel = C1 + C2 + C3 BCECE-2005
In a series, capacitance will be less than any parallel MHT-CET- 2001, 2020
plate capacitor CG PET-2015
1 1 1 Ans.(a): Capacitor of capacitance 1.5µF and 1.5µF are
Cseries = + +
C1 C 2 C3 connected in parallel. Hence their equivalent resistance
is 3µF.
So, Cparallel > Cseries
Hence, assertion is correct and reason is incorrect.
247. For the circuit shown in the figure the charge Νοw, three capacitor each of value 3µF are in series.
on 2µF capacitor is Hence, their equivalent capacitance is given by,
1 1 1 1
= + +
C 3 3 3
C = 1µF
249. When two conductors of charges and potentials
C1, V1 and C2, V2 respectively are joined, the
common potential will be:
C V + C 2 V2 C V 2 + C 2 V2 2
(a) 1 1 (b) 1 1 2
(a) 15 µC (b) 20 µC V1 + V2 V1 + V2 2
(c) 30 µC (d) 45 µC C1V1 + C 2 V2
(c) C1 + C 2 (d)
AIIMS-27.05.2018(E) C1 + C 2
Ans. (a) : BCECE-2003
Ans. (d) : When two conductors of charges and
potentials C1, V1 and C2, V2 respectively are joined.
Let the common potential will be V.
By the law of conservation of charge,
Qi = C1V1 + C 2 V2
Q f = C1V + C 2 V
Q = q1 + q 2 ∴ Qi = Qf
C1 V1 + C2 V2 = C1 V + C2 V
2V0 = 1(10 − V0 ) + 5 (10 − V0 )
C V + C 2 V2
2V0 = 10 − V0 + 50 − 5V0 V= 1 1
C1 + C 2
8V0 = 60
250. In the circuit shown, the potential difference
60 across the 4.5 µF capacitor is
V0 =
8
Q Q = CVo
60
Q = 2V0 = 2 ×
8
60
∴ Q=
4
Q = 15µC

Objective Physics Volume-III 874 YCT


8 252. ‘n’ identical capacitors are joined in parallel
(a) 6 volt (b) volt
3 and are charged to potential ‘V’. Now they are
(c) 4 volt (d) 8 volt separated and joined in series, then
(a) the potential difference is ‘nV’ and energy
MHT-CET 2020
increase ‘n’ times.
CG PET -2006
(b) the potential difference remains the same and
Ans. (d) : Since, 4.6µF and CR = 9 µF are in series. energy increases ‘n’ times.
So, the charge on capacitors is same. (c) the potential difference and the total energy of
∴ C1V1 = C2 V2 the combination remain the same.
⇒ 9V1 = 4.5 V2 (d) the potential difference becomes ‘nV’ and
energy remains the same.
⇒ V2 = 2V1
MHT-CET 2020
Given, V1 + V2 = 12 Volt
Ans. (d) : Energy in parallel connection,
V1 + 2V1 = 12
1
⇒ 3V1 = 12 U P = ( nC ) VP2
2
⇒ V1 = 4 volt Energy in series connection,
V2 (Voltage on 4µF) = 2 × 4 = 8V 1C
US =   VS2
251. The potential differences that must be applied 2 n 
across the parallel and series combination of 3 According to energy conservation, energy remains the
identical capacitors are such that the energy same.
stored in them becomes the same. The ratio of U P = US
potential difference in parallel to series
( nC ) VP2 =   VS2
1 1 C
combination is
2 2 n 
1 1
(a) (b) VS = nVP
3 4
253. Three condensers of capacities C1, C2, C3 are
1 1
(c) (d) connected in series with a source of e.m.f. V.
8 6 The potentials across the three condensers are
MHT-CET 2020 in the ratio of
Ans. (a) : Three capacitors are connected in parallel (a) C12 : C 22 : C32 (b) 1 : 1 : 1
CP = C + C + C 1 1 1
CP = 3C (c) C1 : C2 : C3 (d) : :
C1 C2 C3
Three capacitors are connected in series,
MHT-CET 2020
1 1 1 1
= + + Ans. (d) : Three condensers of capacities C1, C2, C3 are
CS C C C connected in series with a source of e.m.f V.
C Q = C1V1 = C2V2 = C3V3
CS =
3 Q Q Q
V1 = , V2 = , V3 =
Energy stored is same in both conditions, C1 C2 C3
1 1 ∴ Q=1
C P VP2 = CS VS2
2 2 Ratio of potential across the three condensers,
2 1 1 1
 VP  CS V1 : V2 : V3 = : :
  = C1 C2 C3
V
 S C P
254. Four capacitors of equal capacity have an
VP CS equivalent capacitance C1 when connected in
=
VS CP series and an equivalent capacitance C2 when
C
Ratio of potential difference in parallel to series connected in parallel. The ratio 2 , is
C1
combination,
(a) 4 (b) 8
VP C 1
= = (c) 16 (d) 12
VS 3 × 3C 3
MHT-CET 2020

Objective Physics Volume-III 875 YCT


Ans. (c): Four capacitors of equal capacity have an Capacitor C1 and C2 are connected in parallel,
equivalent capacitance C1 when connected in series, CR = C1 + C2 …..(ii)
1 1 1 1 1 Q
= + + + Resultant potential ( V2 ) = 1
C1 C C C C CR
C Putting the value of Q1 , CR from equation (i) & (ii),
C1 =
4 CV
V2 = 1 1
Again, connected in parallel, C1 + C2
C2 = C + C + C + C
257. Two parallel plates separated by a distance ‘d’
C2 = 4C
are kept at potential difference ‘V’ volt. A
C 4C
The ratio of 2 = ×4 charge ‘q’ of mass ‘m’ enters in parallel plates
C1 C with some velocity. The acceleration of the
= 16 charged particle will be
255. Five capacitors each of capacity ‘C’ are qd qV
(a) (b)
connected as shown in figure. If their resultant Vm dm
capacity is 2µF, then the capacity of each md qm
condenser is (c) (d)
Vg Vd
MHT-CET 2020
Ans. (b): Given,
Distance between two parallel plates = d
Potential difference = V
Charge = q
Mass = m
We know that,
V
Electric field between the two plates ( E ) =
(a) 2.5 µF (b) 10 µF d
(c) 5 µF (d) 2 µF Force on charge q,
MHT-CET 2020 F = qE
Ans. (b) : Given, resultant capacitor (CR) = 2µF V qV
F = q× = …..(i)
Five capacitor of capacity 'C' are connected in series. d d
1 1 1 1 1 1 Acceleration of the charge particle–
= + + + +
CR C C C C C F
a=
C m
CR = Putting the value of F from equation (i), we get-
5
C = 5 × CR = 5 × 2 × 10 = 10µF
–6 qV
The capacity of each condenser is 10µF. a= d
256. A condenser of capacity ‘C1’ is charged to m
potential ‘V1’ and then disconnected. qV
a=
Uncharged capacitor of capacity ‘C2’ is dm
connected in parallel with ‘C1’. The resultant 258. A capacitor C1 = 4µF is connected in series with
potential ‘V2’ is another capacitor C2 = 1µF. The combination
C2 C1 C1V1 is connected across d.c. source of 200 V. The
(a) (b)
C1 + C2 C2 ratio of potential across C2 to that across C1 is
V1C2 C1 V1
(c) (d)
C1 C1 + C2 (a) 16 : 1 (b) 8 : 1
(c) 2 : 1 (d) 4 : 1
MHT-CET 2020
MHT-CET 2020
Ans. (d) :
Charge (Q) = CV Ans. (d): Given that,
Q1 = C1V1 …..(i) Capacitor (C1) = 4µF, C2 = 1µF, Voltage = 200 V

Objective Physics Volume-III 876 YCT


The above circuit can be redrawn as,

ε0 A εA
C1 = and C2 = 0
d1 d2
Capacitor in series–
Equivalent capacitance in the series combination, Now,
C × C2 CC
Ceq = 1 Ceq = 1 2
C1 + C2 C1 + C2
4 ×1 4 ε0 A ε0 A
Ceq = = = 0.8 µF ×
4 +1 5 d d2
Ceq = 1
Charge flowing through the circuit, ε0 A ε0 A
+
Q = Ceq × V d1 d2
Q = 0.8 × 200  1 1 
Q = 160 µC d ×d 
Potential across capacitor C1, Ceq = ε o A  1 2

1 + 1 
Q 160  d1 d 2 
V1 = = = 40 volts
C1 4
εo A
Ceq =
Potential across capacitor C2, ( d1 + d 2 )
Q 160
V2 = = = 160 volts 1 36π ×10−4
C2 1 Ceq = ×
4π × 9 × 109 ( 3 × 10−3 + 1× 10−3 )
The ratio of potential across C2 to that across C1 is,
V2 4 1
= Ceq = ×10−10
V1 1 4
Ceq = 25×10–12 F
V2:V1 = 4:1
Ceq = 25 pF
259. A metal plate of thickness 2 mm and area 36π
260. Find the equivalent capacitance between point
cm2- is slide into a parallel plate capacitor of
A and B.
plate spacing 6 mm and area 36π cm2. The
metal plate is at a distance 3 mm from one of
the plates. What is the capacitance of this
 1 
arrangement?  Let = 9 ×109 Nm 2C-2 
 4πε0 
(a) 8 pF (b) 15 pF
(c) 25 pF (d) 20 pF (a) 4C (b) 3C
TS-EAMCET-04.05.2019, Shift-1 (c) 2C (d) 1C
Ans. (c): Given, TS-EAMCET-09.09.2020, Shift-1
Thickness of plate = 2mm, Area = 36π cm2 Ans. (b): According to figure–

Objective Physics Volume-III 877 YCT


24×10–6 = V1×4µF+V1×8µF
24×10–6 = V1×(4+8)×10–6
24 × 10−6
V1 =
12 × 10−6
V1 = 2V
262. Five equal capacitors each with capacitance C
are connected as shown in figure. Then, the
equivalent capacitance between A and B is

C
Hence, (a) 5 C (b)
5
Cnet = C1+C2+C3
(c) 3 C (d) C
Cnet = C+C+C
Manipal UGET -2020
Cnet = 3C
CGPET-2015
261. A capacitor of capacitance 4 µF is charged to a
Ans. (d): Given figure,
potential difference of 6 V with a battery. The
battery is removed and in its place another
capacitor of capacitance is 8µF introduced and
the circuit is closed. The potential difference
attained by each of the capacitors in V is
(a) 2 (b) 4
(c) 6 (d) 8
TS-EAMCET-03.05.2019, Shift-2
Ans. (a): Given that,
Capacitance (C) = 4µF, Potential difference (V) = 6V

We can remove the capacitor between C and D It is


balanced Wheatstone bridge.

Charge (Q) = CV
Q = 4 × 10–6 × 6
Q = 24 ×10–6 C
Now,

Q1 = V1×4µF C C
C AB = +
Q2 = V1×8µF 2 2
∴ Q = Q1+Q2 CAB = C

Objective Physics Volume-III 878 YCT


263. Effective capacitance between A and B in the 264. The charge deposited on 4µF capacitor in the
figure shown is (all capacitances are in µF) circuit is

(a) 6×10-6 C (b) 12×10-6 C


(c) 24 ×10-6 C (d) 36 ×10-6 C
Manipal UGET-2009
Ans. (c): Given that,
3
(a) µF
14
14
(b) µF
3
(c) 21 µF
(d) 23 µF
Manipal UGET-2012
Ans. (b): Given figure,

Balance Wheatstone bridge–


3 6
=
4 8
As the capacitor 4µF & 2µF connected in parallel then
The point C and D has no charge–
its resultant are in series with 6µF capacitor,

Ceq =
( 2 + 4 ) × 6 = 3µF
2+4+6
Charge in circuit–
Q = 3µF×12V = 36µC

3 × 6 18
C1 = = = 2µF
3+ 6 9
4 × 8 32 8
and C2 = = = µF
4 + 8 12 3

Q1 C1 4
= =
Q2 C2 2
Q1 = 2Q2
Hence,
Q = Q1+Q2
Ceq = C1 + C 2 36µC = 2Q 2 + Q 2
8 6 + 8 14 36µC
= 2+ = = µF Q2 = = 12µF
3 3 3 3

Objective Physics Volume-III 879 YCT


Hence, 8
CGD = µF
Q1 = Q–Q2 3
Q1 = 36µC – 12µC ∴ Capacitance between points E and F, are connected
Q1 = 24×10–6 C in parallel, So
265. In the following circuit the resultant CEF = 4µF + 4µF = 8µF
capacitance between A and B is 1µF. Then, the
value of C is

32 11 Capacitors between points G and F, are connected in


(a) µF (b) µF
11 32 series, therefore,
23
µF
32
µF 1 1 1 8 +1 9
(c) (d) = + = =
32 23 CGF 1 8 8 8
CG PET 2019
8
Ans. (d): Given, CGF = µF
9

Capacitance between A and B, CAB = 1µF.


∴ Capacitors between point C and D are connected in 8 8 24 + 8 32
CGD = + = = µF
parallel so capacitance. 3 9 9 9
CCD = 2µF+2µF = 4µF
∴ Capacitors between point E and F connected in
series, so the capacitance between points E and F, is
1 1 1 2 +1 3
= + = =
C EF 6 12 12 12
12
C EF = = 4µF
3

1 1 1
= +
C AB C 32 / 9
1 1 9
= +
C AB C 32
Here,
CAB = 1µF
Capacitors between points G and D are connected in 1 9 32 − 9 23
= 1− = =
series, therefore, C 32 32 32
1 1 1 1+ 2 3 32
= + = = C= µF
CGD 8 4 8 8 23

Objective Physics Volume-III 880 YCT


266. Four capacitors are connected as shown in the C1 and C2 are in series combination–
figure below. The equivalent capacitance 1 1 1
between the points P and Q is = +
C C Ceq 1 2

1 1 1 1+1 2
= + = =
Ceq 4 4 4 4
1 1
=
Ceq 2
Ceq = 2µF
1
(a) 4µF (b) µF ∴ Ceq and C3 capacitor are joined into the parallel
4 combination–
3 4 C'eq = Ceq + C3
(c) µF (d) µF
4 3 C'eq = 2+4
CG PET -2018 C'eq = 6µF
Ans. (d): Given figure, 268. Three capacitors C1, C2 and C3 are connected
to a battery of V volt as shown in figure. The
charges and potentials are shown in figure.
Then, the correct answer is

Three, 1µF capacitors are connected in series


combination. So, their resultant capacitance,
1 1 1 1
= + + =3
C1 1 1 1
(a) Q1=Q2=Q3, V1=V2=V3=V
1
C1 = µF (b) Q1=Q2+Q3, V=V1+V2+V3
3
(c) Q1=Q2+Q3, V=V1+V2
Now, capacitor C1 is connected in parallel combination
(d) Q2=Q3, V2=V3
with 1µF capacitor.
Hence, equivalent capacitance between the P and Q, CG PET -2016
CPQ = C1+1 Ans. (c): Given circuit,
1 4
C PQ = + 1 = µF
3 3
267. Three capacitors each of 4 µF are to be
connected in such a way that the effective
capacitance is 6 µF. This can be done by
(a) Connecting all of them in series
(b) Connecting them in parallel
According to Kirchoff's first law-
(c) Connecting two in series and one in parallel
We know that in series combination charge remains
(d) Connecting two in parallel and one in series
same and in parallel combination of capacitors voltage
CG PET- 2017
remain same.
Ans. (c): To get the equivalent capacitance 6µF, the Q1 = Q2 + Q3
capacitor are arranged in the following manner. As C2 and C3 are in parallel,
Then, V2 = V3
Total voltage V will equal to the combination or voltage
across C1 and voltage across parallel combination of C2
and C3.
So,
V = V1+V2 and Q1 = Q2 + Q3

Objective Physics Volume-III 881 YCT


269. Equivalent capacitance between A and B for Ans. (d) : Equivalent capacitance is given by-
circuit shown in figure.

(a) 3 µF (b) 2 µF
(c) 4 µF (d) 8 µF
CG PET- 2007
Ans. (c) :Given that, We know that,
Q = C.V
Q
V=
C
1
V∝
C
Hence, V1 : V2 = C2 : C1
C2
V1 = ×V
C1 + C 2
9
V1 = × 12
13.5
90
V1 = × 12
135
V1 = 8 Volt
271. Three identical capacitors are connected
together differently. For the same voltage to
every is
Hence, net capacitance between A & B (a) the three in series
Cnet = C1 + C2 (b) the three in parallel
= 2 +2 (c) two in series and the third in parallel with it
Cnet = 4 µ F (d) two in parallel and the third in series with it
CG PET- 2006
270. Potential difference across capacitor 4.5 µF
capacitance is Ans. (b) : If two or more capacitors are joined together
in parallel then each capacitor is connected
independently to same voltage source.
This means that each capacitor has the same voltage
across its plate.
272. In the circuit given below, the charge in µC, on
the capacitor having 5 µF is

8
(a) V (b) 4V
3 (a) 4.5 (b) 9
(c) 6V (d) 8V (c) 7 (d) 15
CG PET- 2007 BITSAT-2020

Objective Physics Volume-III 882 YCT


Ans. (b): According to Figure, all the capacitors are connected in
series,
Hence, net capacitance is,
1 1 1
Cnet = + +
C1 C2 C3
1 1 1 1
= + +
C 2 3 6
1 3 + 2 +1 6
Here, 2µF and 5µF are in parallel, = =
Then, Cnet = 5 + 2 = 7 µF C 6 6
Cnet = 1µF
Being series each capacitor is same charge,
Hence,
Q = Cnet .V (V = 10 V)
= 1×10
Q = 10µ
274. In the given circuit the potential drop across
We know that,
the capacitor must be
Q=C×V
Q
V=
C
1
V∝
C
Then, V1 : V2 = C2 : C1 V
(a) V (b)
 V1 = 6 V  2
C1  
V2 =
C1 + C 2
× V1 C1 = 3 µF  (c)
V
(d)
2V
C2 = 7µF  3 3
3 BITSAT-2016
V2 = ×6
10 Ans. (c) : Given,
V2 = 1.8Volt
Hence, charge on (Q5µF) = C×V
= 5 ×1.8 = 9µC
273. In the given figure, the charge on 3 µF
capacitor is

In steady state, no current will pass through the


capacitor,
In the lower loop,
(a) 10 µC
2V – 2iR – iR – V = 0
(b) 15 µC
i = V/3R
(c) 30 µC
For the upper loop,
(d) 5 µC
V – VC – iR – V = 0
BITSAT-2019
VC = iR
Ans. (a) :
V
|VC| = ×R
3R
V
|VC| =
3

Objective Physics Volume-III 883 YCT


275. In the figure below, what is the potential Ans. (b) : Since a middle arm which connect the upper
difference between the point A and B and short the circuit. So, it will neglected.
between B and C respectively in steady state

The effective circuit is shown in figure.


(a) VAB = VBC = 100V The capacitance of upper series,
(b) VAB = 75V, VBC = 25V 1 1 1 1 1
= + + + ....∞
(c) VAB = 25V, VBC = 75V Ceq 1 3 9 27
(d) VAB = VBC = 50V
2
BITSAT-2016 ∴ C= µC
3
Ans. (c) : Given,
2
CAB = 2C = 2 × µF
3
4
CAB = µF
3
277. Four capacitors of capacitances 2 µF, 3µF, 4µF
and xµF are connected to a battery of emf 6V
At steady, current through capacitors is zero. Hence and of negligible internal resistance, as shown
effect of resistors in the circuit can be neglected. in the figure. If the ratio of the charges on x µF
When capacitors are connected in series with a battery 3
and 4µF capacitances is , then the value of x
of potential V. 8
Potential difference across each capacitor is is
 C2 
V1 =  V
 C1 + C2 
 C1 
V2 =  V
 C1 + C2 
Now,
 2 
V1 = VAB =   × 100 = 25V
2 + 6
(a) 2 (b) 5
 6 
V2 = VBC =   × 100 = 75V (c) 3 (d) 8
2 + 6 AP EAMCET (20.04.2019) Shift-1
So, correct answer is option (c).
Ans. (b) :
276. What is equivalent capacitance of circuit
between points A and B?

2 4
(a) µF (b) µF
3 3
Let, charge on 2µF, 3µF, 4µF and xµF are q2, q3, q4 and
(c) Infinite (
(d) 1 + 3 µF ) qx
BITSAT-2017 q4 = C4x = 4µF × 6 = 24 µC

Objective Physics Volume-III 884 YCT


As given, Q 5 ×10−3
V2 = =
qx 3
= C2 4 × 10−6
q4 8 V2 =1.25 kV
3 So, the net potential difference in the upper branch,
qx = 24 × Vu = V1 + V2
8
Vu = (1.25 + 1) kV
qx = 9 µC Vu = 2.25 kV
Potential difference on x µF, 279. Three uncharged capacitors of capacities C1,
q 9  q C2 and C3 are connected as shown in the figure
V1 = x = V =  A, B and C are at potentials V1, V2 and V3,
x x  C
respectively, then the potential at O is
9
Same volt will be across 2 µF too,
x
 9
Now, remaining  6 −  volt potential difference will
 x
drop across 3 µF
As 3 µF and (x + 2)µF are in series.
So, charge on them will be same q3 = qx + 2
 9 9 C1V1 + C2V2 + C3V3
3  6 −  = ( x + 2) (a)
C1 + C2 + C3
 x x
6x – 9 = 3x + 6 C1V1 + C2V2 − C3V3
(b)
3x = 15 C1 + C2 + C3
x=5 C1V1 − +C2V2 − C3V3
(c)
278. Four capacitors marked with capacitances and C1 + C2 + C3
breakdown voltages are connected as shown in
(d) Zero
the figure. The maximum emf of the source, so
AP EAMCET (22.04.2018) Shift-1
that no capacitor breaks down is
Ans. (a) :

(a) 10.5 kV (b) 5.25 kV


(c) 2.25 kV (d) 1.25 kV
AP EAMCET (23.04.2018) Shift-1 By Kirchhoff's law, net charge at a junction = 0
Q1 + Q2 + Q3 = 0
Ans. (c) :
Let, the potential of O = V0
Q = CV
Q
VA – V0 = 1
C1
Q1 = C1(VA – Vo)
Similarly,
Q2 = C2(VB – Vo)
Q3 =C3(VC – Vo)
In the upper branch, the capacitor is connected in series,
At, V A = V 1, V B = V 2, V C = V 3 (given)
so they have the same charge. So the minimum charge
Q1 + Q2 + Q3 = 0
is
C1(V1 – Vo) + C2(V2 – Vo) + V3(V3 – Vo) = 0
Q = C1V1
C1V1 + C2V2 + C3V3 = C1Vo + C2Vo + C3Vo
Q = 5 × 1 × 10–3 C
V C + V2 C2 + V3 C3
This charge is same for second connected capacitor. Vo = 1 1
So, potential for the second capacitor for this charge, C1 + C2 + C3

Objective Physics Volume-III 885 YCT


280. An infinite number of capacitors 2.0 µF, 4.0 µF, 282. Three capacitors of 2µF, 3µF and 6µF are
8.0 µF, 16.0 µF,......... are connected in series. joined in series and the combination is charge
The equivalent capacitance of the system is by means of a 24 V battery. The potential
difference between the plates of the 6 µF
(a) ∞ (b) 0.25µF
capacitor is
(c) 0.5µF (d) 1.0µF (a) 4 V (b) 6 V
UPSEE - 2018 (c) 8 V (d) 10 V
Ans. (d) : Given that, UPSEE - 2010
An infinite number of capacitors 2 µF , 4 µF, 8 µF, 16 Ans. (a) : Equivalent capacitance of capacitors
µF ..... etc., are connected in series, connected in series.
1 1 1 1 1 1 1 1
= + + ......∞ = + + = 1 µF
Ceq 2 4 8 Ceq 2 3 6
1 Hence, charge flowing through the circuit,
1 q = Ceq.V
= 2
Ceq 1 − 1 q = 1 × 24
2 q = 24 µC
1 Potential difference between the plates of 6µF
=1 capacitor,
Ceq
q
The equivalent capacitance of the system is, V' =
C
Ceq = 1
24
281. The total capacitance of the system of V' =
capacitors in figure. Between A and B is 6
V' = 4
283. The equivalent capacitance of the combination
of the capacitors is :

(a) 1 µF (b) 2 µF
(c) 3 µF (d) 4 µF
UPSEE - 2013
Ans. (b) : Given, (a) 3.20 µF (b) 7.80 µF
(c) 3.90 µF (d) 2.16 µF
UPSEE - 2005
Ans. (a) :

In figure, C1 and C2 are in series,


Equivalent capacitance of two parallel capacitors,
∴ C1 = 1 µF
= 10 µF + 6 µF
Now, C1 and C5 are in parallel,
= (10 + 6) µF
∴ C1P = 1 + 1 = 2 µF
= 16 µF
Now, C1P and C3 are in series, This 16 µF capacitor is in series combination with 4µF
capacitor,
∴ C2S = 1 µF
∴ Equivalent capacitance of the entire combination -
Now, C2S and C4 are in parallel, 1 1 1 1+ 4
= + =
C 2P = 1 + 1 Ceq 16 4 16
16
C2P = 2 µF ⇒ Ceq = = 3.20µF
5
Objective Physics Volume-III 886 YCT
284. Three capacitors connected in series have an  V1 + V2 = 5
effective capacitance of 4 µF. If one of the C1V1 = C2V2 V = 5 − V 
capacitance is removed, the net capacitance of  1 2

the capacitor increases to 6 µF. The removed 2(5 – V2) = 3(V2)


capacitor has a capacitance of 10 – 2V2 = 3V2
(a) 2 µF (b) 4 µF 5V2 = 10
V2 = 2V
(c) 10 µF (d) 12 µF
286. In the circuit below, the potential difference
(e) 24 µF between A and B is
Kerala CEE- 2013
Ans. (d) : Let there are three capacitors with
capacitances C1, C2, C3 respectively and C1 is removed.
In case-I
1 1 1 1
= + + ….(i)
Ceq1 C1 C2 C3
1 1 1
= + ….(ii) (a) 10 V (b) 20 V
Ceq2 C2 C3
(c) 30 V (d) 40 V
From (i) and (ii), JCECE-2017
1 1 1 Ans. (a) :
= +
Ceq1 C1 Ceq2
1 1 1
= +
4 C1 6
1 1 1
= −
C1 4 6
1 3− 2
=
C1 12
⇒ C1 = 12 µF
285. Three capacitors are connected in the arms of a
triangle ABC as shown in figure. 5 V is applied
between A and B. The voltage between B and C
Capacitor 1µF and 3µF are in series,
is
1 1 1
= +
Ceq C1 C 2
1 1 1 3 +1 4
= + = =
Ceq 1 3 3 3
3
Ceq = µF
4
So, Resultant circuit will be,
(a) 2 V (b) 1 V
(c) 3 V (d) 1.5 V
(e) 0.5 V
Kerala CEE - 2010
Ans. (a) : Given,

Both have same charges-


Objective Physics Volume-III 887 YCT
3 Ans. (c):
Capacitor µF and 3µF are in parallel.
4
So,
3 15
Ceq = + 3 = µF
4 4

q q q q
+ + + = 20
C C C C
20C
q= = 5C
4
q = 5C
15 q q
1 µF and µF are in series, VN − VQ = + = 5 + 5 = 10V
4 C C
15 VN – 10 = VQ

4 VQ = –10V (VN = 0)
Ceq =
15 q
1+ VP − VN = 2 × = 2 × 5 = 10V
4 C
15 4 15 VP = VN + 10
Ceq = × = µF
4 19 19 VP = 0 + 10
Now, the charge supplied by battery, VP = 10V
q = Ceq V 288. Minimum number of 8µF and 250 V capacitors
15 are used to make a combination of 16µF and
= × 190 × 10–6
19 1000 V are
q = 150 µC (a) 4 (b) 32
Potential across V1 (c) 8 (d) 3
q 150
V1 = = = 150V JCECE-2010
C 1 Ans. (b) : To create 1000V, we need to combine 4
Potential across V2 capacitors in series.
q 150 × 4
V1 = = = 40V 8µF
C 15 So, total capacity becomes = 2µF
4
So, potential difference between A and B are,
In order to obtain capacity of 16µF, 8 rows of this
 1 
VAB =   × 40 combination will be needed in parallel.
1+ 3 
Total capacity = 2 µF × 8
1 = 16 µF
VAB = × 40 = 10V
4 Total number of capacitor = 4 × 8 = 32
287. Four equal capacitors are connected to a
289. What is equivalent capacitance of the network?
battery as shown in the adjoining figure. The
Each capacitor has 1 µF capacitance :
potentials of P and Q are

1
(a) µF (b) 2µF
3
3
(c) µF (d) 3µF
(a) 5 V and 5 V (b) 10 V and 5 V 2
(c) 10 V and –10 V (d) 5 V and –5V JCECE-2006
JCECE-2015 WB JEE - 2002
Objective Physics Volume-III 888 YCT
Ans. (d): Ans. (d) : Given that,
Reactance (x) = 25Ω
400
Frequency (f) = Hz
π
1
Reactance (x) =
C× ω
1
x=
Each capacitor has 1µF C × 2πf
C1 = 1µF, C2 = 1µF, C3 = 1µF 1
25 =
Capacitor are connected in parallel, 400
C × 2π×
Ceq = C1 + C2 +C3 π
Ceq = 1 + 1 + 1 1
Ceq = 3µF C=
800 × 25
290. Four 10 µF capacitors are connected to a 500 V
1
supply as shown in the figure. The equivalent C=
capacitance of the network is : 2 × 104
C = 0.5 × 10–4
C = 50 × 10–6 F
C = 50 µF
292. How many 6µF, 200 V condensers are needed
to make a condenser of 18µF, 600 V?
(a) 3 (b) 27
(c) 9 (d) 18
(a) 40 µF (b) 20 µF COMEDK 2016
(c) 13.3 µF (d) 10 µF Ans. (b) : To get potential of 600V, 3 capacitors (6µF,
JCECE-2005 200V) must be connected in series
Ans. (c) : Given that, C = 6 µF
C1 = C2 = C3 = C4 = C = 10µF 1 1 1 1
Since, C1, C2 and C3 are connected in parallel to C4 = + +
Ceq 6 6 6
So, equivalent capacitance of C1, C2 and C3
1 1+1+1
1 1 1 1 =
= + + Ceq 6
C ' C1 C2 C3
1 1 1 1 1 3
= + + =
C ' 10 10 10 C eq 6
10 1 1
C' = µF =
3 Ceq 2
Equivalent capacitance of C' and C4, Ceq = 2µF
Ceq = C' + C4
To get 18 µF, the above combination must be repeated
10 for 9 times which should be connected in parallel.
Ceq = + 10
3 So, we get
40 (Ceq)final = 18µF
Ceq =
3 So, finally we would get 18µF, 600V condenser
Ceq = 13.3 µF So, total number of required condenser = 9 × 3 = 27
291. Reactance of a capacitor of capacitance C µF 293. Two parallel plate of area A and separated by
400 two different dielectric as shown in the figure.
for AC frequency Hz is 25Ω, the value of C The net capacitance is–
π
is :
(a) 75 µF (b) 100 µF
(c) 25 µF (d) 50 µF
JCECE-2004

Objective Physics Volume-III 889 YCT


15ε0 A 10ε0 A For third capacitor,
(a) (b)
4d 7d ε0 A
C3 =
11ε0 A 12ε0 A 3 ( 3d )
(c) (d)
5d 3d ε0 A
C3 = ....(iii)
BCECE-2018 9d
Ans. (a) : From equation (i), (ii) and (iii), we get-
The connection in parallel,
Ceq = C1 + C2 + C3
εA εA εA
= 0 + 0 + 0
3d 6d 9d
Kε0 A
C= ε0 A  1 1 1 
d Ceq =  + + 
d 3 6 9
3ε A 6ε A
C1 = 0 = 0 ε A  6+3+ 2 
d/2 d Ceq = 0  
d  18 
5ε0 A 10ε0 A
C2 = = ε0 A  11 
d/2 d Ceq =  
Q C1 and C2 are in series. So, d  18 
C × C2 11 ε0 A
Cnet = 1 Ceq =
C1 + C2 18 d
6ε0 A 10ε0 A 295. In the figure shown, the voltage across C3 will
× be
= d d
6ε0 A 10ε0 A
+
d d
2
ε A
60  0 
d  15 ε0 A
Cnet =  =
ε0 A 4 d
16
d
294. Three parallel plate air capacitors are
connected in parallel. Each capacitor has plate
area ‘A/3’ and separation between the plates is
C2 V C1V
‘d’, ‘2d’, ‘3d’ respectively. The equivalent (a) (b)
C1 + C2 + C3 C1 + C2 + C3
capacity of combination is (ε0 = absolute
permittivity of free space)
(c)
C3 V
(d)
( C1 + C2 ) V
(a) 7Aε0 /18d (b) 5Aε0 / 9d C1 + C2 + C3 C1 + C2 + C3
(c) 11Aε0 /18d (d) 9Aε 0 /17d MHT-CET 2020
MHT-CET 2016 Ans. (d) :
MHT-CET 2020
ε0 A
Ans. (c) : Capacitor, C =
d
For first capacitor,
ε0 A
C1 = ....(i)
3d
For second capacitor, C'
εA
C2 = 0
3 × ( 2d )
ε0 A
C2 = ....(ii)
6d

Objective Physics Volume-III 890 YCT


The capacitor C1 and C2 are connected in parallel. According to question,
So, C' = C1 + C2 Total energy stored in both the combination is same.
Now, C' and C3 are connected in series so, equivalent Es = Ep
capacitance, 9 C1V 2 1
= N 2C2 V 2
Ceq =
C '× C3
=
( C1 + C2 ) C3 2 N1 2
C'+ C3 ( C1 + C 2 + C3 ) N1 N 2 C2
C1 =
9
Q = Ceq V
297. Two parallel plate air capacitors of same
Q=
( C1 + C 2 ) × C3 × V capacity ‘C’ are connected in series to a battery
( C1 + C2 + C3 ) of emf ‘E’. Then one of the capacitors is
completely filled with dielectric material of
Voltage across C3, constant ‘K’. The change in the effective
V3 =
Q
=
V(C1 + C 2 ) × C3
−=
( C1 + C 2 ) × V capacity of the series combination is
C3 ( C1 + C 2 + C3 ) C3 ( C1 + C2 + C3 ) C  K – 1 2  K – 1
(a)  K + 1 (b)  K + 1
296. A series combination of N1 capacitors (each of 2   C  
2
capacity C1) is charged to potential difference C  K +1  C  K – 1
(c) (d)
‘3V’. Another parallel combination of N2 2  K – 2  2  K + 1 
capacitors (each of capacity C2) is charged to MHT-CET 2017
potential difference ‘V’. The total energy Ans. (a) : Two parallel plate capacitor are connected in
stored in both the combinations is same. The series.
value of C1 in terms of C2 is C× C C
C1 = =
C 2 N1 N 2 C2 N12 N 22 C +C 2
(a) (b)
9 9 And when one of them is completely filled with a
C N C N dielectric, it capacity become KC.
(c) 2 1 (d) 2 2
9N 2 9N1 ∴ Now resultant,
1 1 1
MHT-CET 2018 = +
C2 C KC
Ans. (a) : In series combination,
KC
1 1 1
= + + .........
1 C2 =
Ceq C1 C1 C1 K +1
∴ The change in capacitance
1 N ∆C = C2 − C1
= 1
Ceq C1 KC C
∆C = −
C K +1 2
Ceq = 1
N1  K 1
∆C = C  − 
Potential difference, V = 3V (Given)  K +1 2
1  2K − K − 1 
Es = CV2 ∆C = C  
2  2 ( K + 1) 
1 C1
( 3V ) C  K − 1
2
Es = ∆C =
2  K + 1
2 N1
9C1V 2 298. When three capacitors of equal capacities are
Es =
2N1 connected in parallel and one of the same
In parallel combination, capacity is connected in series with its
combination. The resultant capacity is 3.75 µF.
Ceq = N2C2
The capacity of each capacitor is
Potential difference = V
(a) 5 µF (b) 6 µF
1
∴ Energy store, Ep = N 2 C2 V 2 (c) 7 µF (d) 8 µF
2 MHT-CET 2017
Objective Physics Volume-III 891 YCT
Ans. (a) : 300. The difference in the effective capacity of two
similar capacitors when joined in series and
then in parallel is 6 µF. The capacity of each
capacitor is
(a) 2 µF (b) 4 µF
(c) 8 µF (d) 16 µF
MHT-CET 2015
Let capacitance of each capacitor be C and connected in Ans. (b) : When capacitor in parallel combination,
parallel,
Cp = C + C= 2C
Cp = C1 + C2 + C3
When capacitor in series.
∴ Cp = C + C + C
C× C C
Cp = 3C Cs = =
C+C 2
Now, the capacitors are in series,
Cp − Cs = 6µF
1 1 1
= + C
Ceq Cp C 2C − = 6µF
2
1  1 1
= + 4C − C
Ceq  3C C  = 6µF
2
1 1 1  C = 4µF
= +1
Ceq C  3  301. In the given figure, three capacitors each of
1 4 capacitance 6 pF are connected in series. The
=
Ceq 3C total capacitance of the combination becomes

4Ceq
C=
3
4 × 3.75
C=
3
C = 5µF
299. A capacitor C1 = 4 µF is connected in series
with another capacitor C2 = 1 µF. The (a) 2 × 10–12 F (b) 3 × 10–12 F
combination is connected across DC source of (c) 6 × 10–12 F (d) 9 × 10–12 F
200 V. The ratio of potential across C2 to C1 is MHT-CET 2008
(a) 2 : 1 (b) 4 : 1
Ans. (a) : Total capacitance
(c) 8 : 1 (d) 16 : 1
1 1 1 1
MHT-CET 2015 = + +
Ceq C1 C2 C3
Ans. (b) : Given, C1 = 4µF, C2 = 1µF, V = 200V
In series combination, 1 1 1
= + +
C × C2 4 × 1 6 6 6
Ceq = 1 = = 0.8µF
C1 + C2 4 + 1 1 3
=
Q = C V = 0.8 × 200 = 160 µC C eq 6
eq
Q 160 Ceq = 2pF = 2 × 10−12 F
∴ V1 = = = 40 volt
C1 4 302. A capacitor of 20 µF is charged upto 500 V is
Q 160 connected is parallel with another capacitor of
V2 = = = 160 volt
C2 1 10 µF, which is charged upto 200 V. Then, the
V2 160 4 common potential will be
= = (a) 400 V (b) 200 V
V1 40 1
(c) 100 V (d) 50 V
V2 : V1 = 4 : 1 MHT-CET 2001
Objective Physics Volume-III 892 YCT
Ans. (a): 304. Two capacitors when connected in series have a
C1 = 20µF, C2 = 10µF, V1 = 500 V, V2 = 200V capacitance of 3µF, and when connected in
Common potential, parallel have a capacitance of 16µF. Their
Q C V + C2 V2 individual capacities are
V = net = 1 1 (a) 1µF, 2µF (b) 6 µF, 2µF
Cnet C1 + C2
(c) 12 µF, 4µF (d) 3 µF,16µF
V=
( 20 ×10 ) × 500 + (10 ×10 ) × 200
−6 −6
VITEEE-2017
( 20 + 10 ) ×10−6 Ans. (c) : Let, capacitor C1 and C2,
V = 400 V In series,
303. The equivalent capacitance between ‘a’ and ‘b’ CC
for the combination of capacitors shown in Cs = 1 2 = 3µF ...(i)
C1 + C2
figure where all capacitances are in microfarad
is In parallel,
Cp = C1 + C2 = 16 µF …(ii)
From equation (i) & (ii), we get -
C1C2
=3
16
C1C2 = 48 …(iii)
48
C2 =
C1
Put the value of C2 in equation (ii), we get-
(a) 6.0µF (b) 4.0µF 48
C1 + = 16
(c) 2.0µF (d) 3.0µF C1
VITEEE-2018 C12 − 16C1 + 48 = 0
Ans. (a) : C12 − 12C1 − 4C1 + 48 = 0
C1 (C1 – 12) –4(C1 – 12) = 0
(C1–12) (C1– 4) = 0
C1= 12µF, 4µF
From equation (iii)
48
C1= 12µF, C2= = 4µF
12
48
C1 = 4 µF, C 2 = = 12µF
In given figure 4
C2 and C3 are in parallel, 305. Two capacitors C1 and C2 in a circuit are
∴ C' = C2 + C3 = 1.0 + 3.0 = 4.0 µF joined as shown in figure. The potentials of
Similarly, C4 and C5 are in parallel, points A and B are V1 and V2 respectively.
Then the potential of point D will be
∴ C" = C4 + C5 = 6.0 + 2.0 = 8µF
As, C′ and C1 are in series,
1 1 1 1 1
= + = +
( V1 + V2 )
'
C1 C1 C ' 4 4 C2 V1 + C1V2
(a) (b)
'
C = 2µF
1
2 C1 + C2
As, C6 and C'' are in series, C1V1 + C2 V2 C2 V1 + C1V2
(c) (d)
1 1 1 1 1 1 C1 + C2 C1 + C2
= + = + =
C'2 C6 C" 8.0 8.0 4
VITEEE-2017
C′2 = 4µF
Ans. (c) :
Now, C1' and C'2 are in parallel.
So,
Ceff = C1' + C'2 = 2 + 4 = 6µF If we consider potential at D is V.

Objective Physics Volume-III 893 YCT


Potential across C1 = (V1 – V) Therefore, their equivalent capacitance is,
Potential across C2 = (V − V2) 5C
×C
Ceq × C1
(Q Q = CV) C′eq = = 2
Charge on C1 is, Q1 = C1 (V1 –V) Ceq + C1 5C + C
and charge on C2 is, Q2 = C2 (V – V2) 2
2
When capacitors are in series, so charge on each 5C 5C
C'eq = =
capacitor are same. 7C 7
Thus, Q1 = Q2
307. The capacitance of a parallel plate
C1 (V1 –V) = C2(V–V2) capacitor with air as dielectric is C. If a
C1 V1 – C1V = C2V – C2V2 slab of dielectric constant K and of the
C1V1 + C2V2 = C2V + C1V same thickness as the separation between
C1V1 + C2 V2 the plates is introduced so as to fill 1/4th of
V=
C2 + C1 the capacitor (shown in figure), then the
306. In given circuit, C1= C2= C3= C initially. Now, a new capacitance is
3
dielectric slab of dielectric constant K = is
2
inserted in C2.

C C
(a) ( K + 2 ) (b) ( K + 3)
4 4
C
(c) ( K + 1) (d) None of these
4
VITEEE-2013
Ans. (b) : We know,
The equivalent capacitance become
5C 7C
(a) (b)
7 5
2C C
(c) (d)
3 2
VITEEE-2016 ε0 A
Capacitance =
Ans. (a) : d
ε0 A
Initially for air, C =
d
3A
ε0
Finally for air, C' = 4 = 3  ε0 A  = 3C
d 4  d  4
A
Kε0
When dielectric slab of dielectric constant K = 3/2 is For dielectric, Ck = 4 = K  ε0 A  = KC
d 4  4  4
inserted in C2.
3 Then,
C2 = C 3C KC
2 Ceq = +
Equivalent capacitance for C2 and C3, are in parallel 4 4
Ceq = C2 + C3 Ceq =
( K + 3) C
3 4
= C+C
2 308. Two capacitors of capacities 1µF and CµF are
connected in series and the combination is
5C
Ceq = charged to a potential difference of 120V. If the
2 charge on the combination is 80µC, the energy
Now, Ceq and C1 are in series, stored in the capacitor of capacity C in µJ is

Objective Physics Volume-III 894 YCT


(a) 1800 (b) 1600 C0 × C2
(c) 14400 (d) 7200 C1 =
C1 + C2
VITEEE-2012
C1 =
( 2K1C ) × ( 2K 2C )
Ans. (b) :
2K1C + 2K 2 C
2K1K 2 C
C1 =
K1 + K 2
C1 2K1K 2
=
C K1 + K 2
310. Two capacitors of capacities C and 2C are
Given,
connected in parallel and then connected in
V = 120 Volts.
series with a third capacitor of capacity 3C.
Q = 80 µC The combination is charged with V volt. The
Capacitor (1µF), CµF are in series combination, charge on capacitor of capacity C is
1.C C
Ceq = = 1
1 + C (1 + C ) (a)
2
CV (b) CV

Q = CeqV 3
C (c) 2CV (d) CV
−6
80 × 10 = ×120 ×10−6 2
(1 + C ) VITEEE-2011
8 + 8C = 12 C Ans. (d) : Given that, capacitors of capacitance C and
C = 2 µF 2C are in parallel.
The energy store in capacitor, Then, Ceq = C + 2C = 3C
Q 2 80 × 80 ×10−6 × 10−6 Now, Ceq and third capacitance 3C are in series then
E= =
2C 2 × 2 × 10−6 3C × 3C
E = 1600 µJ C′eq =
3C + 3C
309. A parallel plate capacitor has capacitance C. If
3
it is equally filled the parallel layers of C′eq = C
materials of dielectric constant K1 and K2 its 2
capacity becomes C1. The ratio of C1 and C is Now, the charge on capacitor of capacity C is,
K1 K 2 Q = C′eq × V
(a) K1 +K2 (b)
K1 + K 2 3
Q = CV
K + K2 2K1K 2 2
(c) 1 (d)
K1 K 2 K1 + K 2 311. The series combination of two capacitors
shown in figure is connected across 1000V. The
VITEEE-2012
magnitude of the charges on the capacitors will
JEE Main - 29.01.2023 (Shift-I) be
Ans. (d) : Capacitance
εA
C= 0
d

(a) 3×10−9C
K1Aε0 2K1Aε0 (b) 2×10−9C
∴ C0 = =
d/2 d (c) 2.5×10−9C
K 2 Aε0 2K 2 Aε0 (d) 3.5×10−9C
C1 = =
d/2 d VITEEE-2006

Objective Physics Volume-III 895 YCT


Ans. (b): Given that, C1 = 3pF, C2 = 6 pF 313. The equivalent capacitance between A and B
is:

We know,
Q = CV 100
(a) 50 pF (b) pF
When capacitor are in series combination then, charge 3
on capacitors are same, (c) 150 pF (d) 300 pF
Q1 = Q2 = Q Karnataka CET-2019
Q Q Ans. (b) :
VS = +
C1 C2
 1 1 
1000 = Q  + 
 C1 C2 
 C + C2 
1000 = Q  1 
 C1C2 
1000 × C1C2
Q= fig (i)
C1 + C 2
C1 × C2
1000 × 3 × 10−12 × 6 ×10 −12 Q C1 & C2 are in series then Ceq =
Q= –12
(Q1pF=10 C) C1 + C2
( 3 + 6 ) × 10−12 100 × 100
Ceq = = 50 pF
18 100 + 100
Q = × 10−9
9
⇒ Q = 2 × 10–9 C
312. The difference between equivalent capacitances
of two identical capacitors connected in parallel
to that in series is 6µF. The value of
capacitance of each capacitor is :
Fig (ii)
(a) 2µF
In fig (ii)
(b) 3µF Two capacitor are in parallel and one in series.
(c) 4µF Parallel capacitor = 50 + 50 = 100 pF
(d) 6µF Now,
Karnataka CET-2020 100 pF and 50 pF are in series
Ans. (c) : Let two identical capacitor are C & C.
Then, Cp = C + C = 2 C
C2 C2 1 1 1
CS = = = +
C + C 2C CAB 100 50
50 × 100 100
According to the question, CAB = = pF
Cp – Cs = 6 150 3
314. Two capacitors of 3 µF and 6µF are connected
C2
2C − =6 in series and a potential difference of 900 V is
2C applied across the combination. They are then
4 C2 – C2 = 6 × 2 C disconnected and reconnected in parallel. The
3 C2 = 6 × 2 C potential difference across the combination is :
(a) zero (b) 100 V
C2 = 4 C
(c) 200 V (d) 400 V
C = 4 µF
Karnataka CET-2018
Objective Physics Volume-III 896 YCT
Ans. (d): We know, 4
Q = CV × 6 × 10–6
=
3
Q = 8µC
Energy stored in the system.
1
E = CV 2
2
1 4
= × × 10−6 × ( 6 )
2

2 3
When capacitors are connected in series then charges = 24µJ
are same. 316. Two capacitors of 10pF and 20pF are connected
So, C1 V1 = C2 V2 to 200 V and 100 V sources respectively. If they
V1 C2 6 are connected by the wire, what is the common
= = =2
V2 C1 3 potential of the capacitors?
V1 = 2V2 (a) 133.3 V (b) 150 V
Q V = V1 + V2 (c) 300 V (d) 400 V
900 = 2V2 + V2 Karnataka CET-2014
900 = 3V2 Ans. (a) :
V2 = 300 V
V1 = 600 V
When it connected in parallel,

Given,
C1 = 10 pF, V1 = 200V
Let the potential difference across the combination is C2 = 20 pF, V2 = 100 V
V′. Common potential
C V + C2 V2 C V + C2 V2
Then, V′ = 1 1 V= 1 1
C1 + C2 C1 + C2
3 × 600 + 6 × 300 10 × 200 + 20 ×100
= =
3+6 10 + 20
3600 4000
= V = V
9 30
V = 400 volt V = 133.3 V
315. A system of two capacitors of capacitance 2µF 317. See the diagram. Area of each plate is 2.0 m2
and 4µF is connected in series across a and d = 2 × 10–3 m. A charge of 8.85 × 10–8 C is
potential difference of 6V. The electric charge given to Q. Then the potential of Q becomes
and energy stored in a system are :
(a) 36µC and 108µJ
(b) 8µC and 24µJ
(c) 1µC and 3µJ
(d) 10µC and 30µJ
Karnataka CET-2017
Ans. (b) : Given, C1 = 2µF, C2 = 4µF, V = 6 V
In series combination the equivalent capacitance
CC  2× 4  4 (a) 13 V (b) 10 V
C= 1 2 =  µF = µF
C1 + C2  2 + 4  3 (c) 6.67 V (d) 8.825 V
Electric charge Karnataka CET-2013
Q = CV Ans. (c):
Objective Physics Volume-III 897 YCT
6×3
C1 = = 2µF
6+3
C1 and 2µF in parallel,
Ceq = C + C1 = 2 + 2 = 4µF
V = 2V
1
Energy stored (E) = Ceq V 2
A = 2. 0 m2 2
d = 2 × 10–3m, 1
= × 4 × 22
q = 8.85 × 10–8C, 2
Capacitance = 8 µJ
ε0 A ε0 A 319. How many 6µF, 200V condensers are needed to
C1 = , C2 =
d 2d make a condenser of 18µF, 600 V ?
3 (a) 9 (b) 18
Ceq = C1 + C2 = ε0 A (c) 3 (d) 27
2d
q = Ceq V Karnataka CET-2008

q q ( 2d ) Ans. (d) : Each capacitor is equivalent of 6 µF, 200V


V= =
Ceq 3ε0 A

8.85 ×10−8 × 2 × 2 × 10−3


V=
3 × 8.85 × 10−12 × 2
2
V= ×10
3
20 VAB = 200 + 200 + 200 = 600V
V= = 6.67V
3 1 1 1 1 3
V = 6.67 V = + + =
CAB C C C C
318. The total energy stored in the condenser system
C 6
shown in the figure will be : CAB = = = 2µF
3 3
For 3 capacitors in series,
Ceq = 2µF
According to the question, Ceq = 18µF
So, finally we would get 18 µf, 600V condenser.
So, total number of required condenser 9 × 3 = 27
(a) 2µJ (b) 4µJ 320. A gang capacitor is formed by interlocking a
(c) 8µJ (d) 16µJ number of plates as shown in figure. The
Karnataka CET-2008 distance between the consecutive plates is 0.885
Ans. (c) : cm and the overlapping area of the plates is 5
cm2. The capacity of the unit is :

(a) 1.06 pF (b) 4 pF


(c) 6.36 pF (d) 12.72 pF
Karnataka CET-2006
6µF and 3µF in series, Ans. (b):

Objective Physics Volume-III 898 YCT


Q2 = 12 × 6×10–6 = 72 µC
After connecting positive and negative plates of both
capacitor,
Qnet = 72 – 36 = 36µC
Cnet = 3 + 6 = 9µF
d = 0.885 cm = 0.885 × 10–2 m, A = 5 × 10–4 m2 Q 36
For parallel plates, ∴ Vnet = net = = 4V
Cnet 9
εA
C= 0 323. The capacity of parallel plate capacitor in air
d
and on immersing it into oil is 50 µF and 110
8.854 × 10−12 × 5 × 10−4 µF respectively. The dielectric constant of oil is
=
0.885 ×10−2 (a) 0.45 (b) 0.55
= 0.5 pF (c) 1.10 (d) 2.20
∴ Capacity of 8 capacitors, Ceq = 8 × C J&K CET- 2004
= 8 × 0.5 Ans. (d) : Given that,
= 4 pF Capacitor in oil (Coil) = 110 µF
321. An air filled parallel plate condenser has a Capacitor in air (Cair) = 50µF
capacity of 2 pF. The separation of the plates is Coil
doubled and the interspace between the plates Dielectric constant of oil =
Cair
is filled with wax. If the capacity is increased to
6 pF, the dielectric constant of wax is : 110
= = 2.2
(a) 2 (b) 3 50
(c) 4 (d) 6 324. Four capacitors are connected as shown in
Karnataka CET-2005 figure. The equivalent capacitance between A
Ans. (d) : We know that, and B is
Capacity of air parallel plate,

C= 0
d
Capacity of parallel plate capacitor when immersed in
liquid and distance is doubled,
KAε0
C' =
2d
KC (a) 4 µF (b) 0.25 µF
C' =
2 (c) 0.75 µF (d) 1.33 µF
K×2 J&K CET- 1999
6=
2 Ans. (d) :
k=6
322. Two capacitors of capacitance 3µF and 6µF are
charged to a potential of 12 V each. They are
now connected to each other, with the positive
plate of each joined to the negative plate of the
other. The potential difference across each will
be :
(a) 4 V (b) 6 V
(c) zero (d) 3 V
Karnataka CET-2002
Ans. (a) : Given that,
C1 = 3µF, C2 = 6µF,
Potential (V) = 12 V Equivalent capacitance between point A and B,
Q Q1 = VC1 1 4
Ceq = + 1 = µF = 1.33µF
= 12 × 3×10–6 = 36 µC 3 3
And Q2 = VC2

Objective Physics Volume-III 899 YCT


325. In the circuit shown in the figure, each (a) 4 V (b) 6 V
capacitor is of 3 µF. The equivalent capacity (c) 8 V (d) 4.5 V
between A and B is
UP CPMT-2013
Ans. (c) :

3
(a) µF (b) 3 µF
4
(c) 6 µF (d) 5 µF
J&K CET- 1998
Ans. (d) :

1 1 1 1
= + =
Ceq 4.5 9 3
Ceq = 3µF
Charge on q = 3 × 12 = 36 µC
So, potential difference across 4.5 µF capacitor,
Charge
=
Capacitance
36
= = 8V
The equivalent capacity between point A and B, 4.5
6×3 328. The equivalent capacitance between points A
Ceq = +3
6+3 and B is
Ceq = 5µF
326. The plates of a charge parallel plate capacitor
are moved apart. This will result in increase of
(a) charge (b) potential
(c) capacitance (d) field
J&K CET- 1998
(a) 5/6 C (b) 11/5 C
Ans. (b) : Parallel plate capacitor,
(c) 6 C (d) 5/11 C
εA
C= 0 TS EAMCET 18.07.2022, Shift-II
d
Q Qd Ans. (a) :
Potential difference (V) = =
C ε0 A
When, d increases, potential difference between the
plate increases.
327. In the adjoining, the potential difference across
the 4.5µF capacitor is

C × 5C
CAB =
C + 5C
5C
=
6

Objective Physics Volume-III 900 YCT


329. Three capacitors each of 4 µF are to be We know,
connected in such a way that the effective ∴ Charge (Q) = CeqV
capacitance is 6 µF. This can be done by Q = 2×100×10–6 = 200 µ C
connecting them In series, all the capacitor have same charge.
(a) All in series Hence, the charge on 15 µF capacitor is 200 µC .
(b) All in parallel 331. The equivalent capacitance between points A
(c) two in parallel and one in series and B of circuit shown in the figure is
(d) two in series and one in parallel
TS EAMCET 31.07.2022, Shift-II
Ans. (d) : According to question,
(a) 9 µF (b) 1 µF
(c) 4.5 µF (d) 6 µF
SRMJEEE - 2013
Ans. (a) :

C1 .C2
Ceq = + C3
C1 + C2
4× 4
Ceq = +4 When capacitances are connected in parallel,
4+4 Ceq = C1 + C 2 + C3
16
Ceq = + 4 Ceq = 3 + 3 + 3 = 9µF
8
Ceq = 2 + 4 = 6 µ F 332. When two identical capacitors are in series,
they have 4 µF capacitance and when in
Hence, the capacitors each of 4 µ F are to be connected, parallel 16 µF. What is the capacitance of
two in series and one in parallel in such a way that the each?
effective capacitance is 6 µ F. (a) 6 µF (b) 4 µF
(c) 8 µF (d) 16 µF
330. Three capacitors of capacitances 3 µF, 10 µF SRMJEEE - 2014
and 15 µF are connected in series to a voltage Ans. (c) : According to question,
source of 100 V. The charge on 15 µF capacitor
is
(a) 50 µC (b) 100 µC Capacitor in series (Cs) = 4µF
(c) 200 µC (d) 280 µC 1 1 1
∴ = +
SRMJEEE - 2011 Cs C C
Ans. (c) : Given, 1 2
C1 = 3 µ F, C2=10 µ F, C3 = 15 µF , V = 100V =
4 C
∴ Capacitors are connected in series, C = 8µF
1 1 1 1
= + +
Ceq C1 C2 C3
1 1 1 1
= + +
Ceq 3 10 15
1 10 + 3 + 2 15
= = Capacitor in parallel (CP) = 16µF
Ceq 30 30
CP = C + C
1 1 16 = 2C
=
Ceq 2 C = 8 µF
Ceq = 2µF Hence, the capacitance of each capacitors are 8µF .

Objective Physics Volume-III 901 YCT


333. Two capacitors of capacities C1 and C2 are 335. Two spheres of radii r and R carry charges q
charged up to the potential V1 and V2 then and Q respectively. When they are connected
condition for not flowing the charge between by a wire, there will be no loss of energy of the
on connected them in parallel is system if
(a) C1 = C2 (b) C1V1 = C2V2 (a) qr = QR (b) qR = Qr
(c) qr2 = QR2 (d) qR2 = Qr2
C C
(c) V1 = V2 (d) 1 = 2 Assam CEE-2019
V1 V2
Ans. (b) : We know that,
JIPMER-2014 A capacitor of sphere (C1) = 4πε0R
Ans. (c) : We know that the charge can only flow when And other capacitor of sphere (C2) = 4πε0r
there is a potential difference. Where, R and r are the radius
So the condition of not flowing the charge is V1 = V2 For no loss of energy when potential of both capacitor is
Because when V1 = V2 the potential difference = 0 and same.
then no charge will flow. Q q
=
But, official answer given by commission is option (b) C1 C2
334. Three capacitors of capacitances 1µF, 2µF and Q q
=
4µF are connected first in a series combination, 4πε0 R 4πε0 r
and then in a parallel combination. The ratio of qR = Qr
their equivalent capacitances will be Hence, There will no loss of energy of the system if qR
(a) 2 : 49 (b) 49 : 2 = Qr.
(c) 4 : 49 (d) 49 : 4 336. The equivalent capacitance of the following
JIPMER-2010 combination between a and b is
Ans. (c) : Given,
C1 = 1 µ F, C2 = 2 µ F, C3 = 4 µ F
When capacitors are connected in series,

1 1 1 1
= + +
Cs C1 C2 C3
2C1C 2C3 2C1C2
1 1 1 1 (a) (b) + C3
= + + C1 + C2 + C3 C1 + C2
Cs 1 2 4
C1C2 2C1C2
(c) (d)
1 8+ 4+ 2 C1 + C2 C1 + C2
=
Cs 8 Assam CEE-2018
1 14 7 Ans. (d) :
= =
Cs 8 4
4
CS = µF
7
When capacitors are connected in parallel,
C1 C 2
= =1
C1 C 2
It is a balance bridge so C3 Redundant
Cp = C1 + C2 + C3
CP = 1+2+4
= 7µF
CS 4 / 7
=
CP 7 C1 .C2 CC
Ceq = + 1 2
Cs 4 C1 + C2 C1 + C2
=
Cp 49 2C1C2
Ceq =
C1 + C 2
Cs : Cp = 4 : 49
Objective Physics Volume-III 902 YCT
337. The equivalent capacitance CAB for the points A 10 10
and B of the given combination capacitances is Ans. (b): =
10 10
It is a balance bridge so 5µF is redundant

(a) 10µF (b) 12.5µF


(c) 15µF (d) 32µF 5µF
Tripura-2020
Tripura-27.04.2022 A B
Ans. (a) :
5µF
5µF and 5µF are in parallel
Ceq = 5 + 5 =10µF
339. The equivalent capacitance between A and B in
the given circuit is _____

6 12
=
9 18
It is a balance bridge so 24µF is redundant
(a) 3 µF (b) 1 µF
C1 = 6µF, C2 = 12µF
(c) 2 µF (d) 1.5 µF
C3 = 9µF, C4 = 18µF AP EAMCET-23.08.2021, Shift-I
C1C2 C3 C 4
Ceq = + Ans. (b) :
C1 + C2 C3 + C 4
6 ×12 9 × 18
Ceq = +
6 + 12 9 + 18
72 162
Ceq = +
18 27
Ceq = 4 + 6
Ceq = 10µF
338. The equivalent capacitance of the system
shown in the figure between the points A and B
is

(a) 5µF (b) 10µF


(c) 20µF (d) 40µF
AP EAPCET-12.07.2022, Shift-II
Objective Physics Volume-III 903 YCT
3µF

[Cnet = 2.3 + 4.6 ]

Hence equivalent between a and b is 2.3µF


341. A capacitor of 10 µF charged up to 250 volts is
connected in parallel with another capacitor of
5 µF charged up to 100 volts. The common
Hence, equivalent capacitance between A and B is 1µF. potential is
(a) 500 V (b) 400 V
340. In the figure shown, each capacitor C1 is 6.9 µF
(c) 300 V (d) 200 V
and C2 is 4.6µF. Calculate the equivalent:
CG PET-2021
capacitance between points 'a' and 'b'.
Ans. (d): Given, C1 = 10µF, C2 = 5µF
V1 = 250V, V2 = 100V
Total charge
Q = Q1 + Q2
C1 CV = C1V1 + C2V2 (C = C1 + C2)
(a) 48.3 µF C1V1 + C2 V2
V=
(b) 9.2 µF C
(c) 11.5 µF C V + C2 V2
V= 1 1
(d) 2.3 µF C1 + C2
AMU-2019 (10 × 10−6 × 250 ) + ( 5 × 10−6 ×100 )
V =
Ans. (d) : (10−6 × 5 + 10−6 × 10 )
3000
V=
15
V = 200 Volt
C1 342. The equivalent capacitance of the combination
Given, C1 = 6.9µF as shown in figure between A and B is
C2 = 4.6 µF

(a) 2 µF (b) 1 µF
(c) 3 µF (d) 6 µF
CG PET-2021

Objective Physics Volume-III 904 YCT


Ans. (b): Ans. (b) :

For minimum value of capacitance

1 1 1 1 6
= + + ⇒ Cnet = µF
Cnet 1 2 3 11

For maximum value of capacitance


Cnet = 1 + 2 + 3 = 6µF
344. Two capacitors of same capacity are first
joined in series and then in parallel. The ratio
of resultant capacity in series to that in parallel
combination will be
(a) 2 : 1 (b) 1 : 4
(c) 4 : 1 (d) 1 : 2
MHT-CET 2020
Ans. (b) : For series combination,

1 1 1 1 1 2
= + = + =
Ceff C1 C2 C C C
C
Ceff =
2
For parallel combination,
C'eff = C1 + C2
= C + C = 2C

Ratio of series to parallel combination


Ceff C / 2 1
= =
C'eff 2C 4

Ceff : C'eff = 1: 4

Hence equivalent capacitance between A and B is 1µF. 345. Five capacitors each of capacitance ‘C’ are
connected as shown in the figure. The ratio of
343. If 3 capacitors of values 1 µF, 2 µF and 3µF are
equivalent capacitance between P and R and
available the maximum and minimum values of
the equivalent capacitance between P and Q is
capacitances. One can obtain by different
combinations of the 3 capacitors together are
respectively
6 6
(a) 4 µF µF (b) 6 µF µF
5 11
11 11
(c) 1µF µF (d) 3 µF µF
6 8
EAMCET-1992
Objective Physics Volume-III 905 YCT
(a) 3 : 1 (b) 2 : 3 346. Capacitors of capacities C1, C2 and C3 are
(c) 1 : 1 (d) 5 : 2 connected in series. If the combination is
MHT-CET 2020 connected to a supply of ‘V’ volt, then the
Ans. (b) : Equivalent capacitance between P and R, potential difference across capacitor C1 is
equivalent capacitance of left side & Right side, C C + C1C3 + C1C 2
(i) (a) 2 3
C 2C3 V
V
(b)
C1 + C 2 + C3
C 2C3 V
(c)
C 2C3 + C1C3 + C1C 2
C1C 2 C3
(d)
V
MHT-CET 2020
Ans. (c) : According to question,

(ii)

Total equivalent capacitance,


1 1 1 1 C 2 C3 + C1C3 + C1C 2
= + + =
C C1 C 2 C3 C1C 2 C3
C1C 2 C3
C=
C 2 C3 + C1C3 + C1C 2
Q Q = CV
C1C 2 C3
1 1 1 Q= V
= + C 2C3 + C1C3 + C1C 2
C1 C C
Potential difference across capacitor C1 is
C1 = C / 2
Q C 2 C3 V
V1 = =
1 1 1 1 3 C C1 C 2 C3 + C1C3 + C1C 2
= + + = ⇒ C2 =
C2 C C C C 3 347. The equivalent capacity of two capacitors in
CPR = C1 + C2 series is 3µF and in parallel is 16µF. Their
C C 5C individual capacities in µF are
= + =
2 3 6 (a) 14, 2 (b) 12, 4
Similarly, for capacitor connected between points P and (c) 10, 6 (d) 8, 8
Q, equivalent capacitance, MHT-CET 2020
C Ans. (b) : Given, Cseries = 3µF, Cparallel = 16µF
(C PQ ) = C +
4 For series combination,
5C 1 1 1
= = +
4 Cseries C1 C 2
Ratio of equivalent capacitance, 1 C + C2
C PR 5C / 6 4 2 = 1
= = = Cseries C1C 2
C PQ 5C / 4 6 3
1 C1 + C 2
= ....(i)
C PR : C PQ = 2 : 3 3 C1C 2

Objective Physics Volume-III 906 YCT


For parallel combination, (a) 24µC (b) 100µC
(c) 2.4µC (d) 30µC
AP EAMCET (23.04.2019) Shift-I
Ans. (a) : From diagram,
4×6
Cparallel = C1 + C2 Ceq = = 2.4 µF
16 = C1 + C2 ....(ii) 4+6
Putting this value in equation (i), we get –
1 16
= ⇒ C1C 2 = 48µF ....(iii)
3 C1C 2
Q (C1 − C2)2 = (C1 + C2)2 − 4C1C2
= (16)2 − 4 × 48
C1 − C2 = 8 ....(iv) As we know, potential drop across parallel branch AB,
Adding equation (iv) and equation (ii), we get – VAB = 10V
2C1 = 24
∴ Q = Ceq VAB
C1 = 12µF
Q C1 − C2 = 8 Q = 2.4 × 10 × 10–6
12 − C2 = 8 Q = 24 µC
C 2 = 4 µF Since in a series capacitor branch, there is equal storage
348. Two capacitors of capacitances 3 µF and 6 µF of charge in each capacitor.
are charged to a potential of 12 V each. They So, 4µF capacitor store a charge of 24 µC.
are now connected to each other, with the
positive plate of each joined to the negative 350. A capacitance of 2 µF is required in an
plate of the other. The potential difference electrical circuit across a potential of 1.0 kV. A
across each will be large number of 1 µF capacitors are available
(a) 4 V (b) 12 V which can withstand a potential difference not
(c) zero (d) 3 V
more than 300 V. the minimum number of
Manipal UGET-2016
capacitors required to achieve this is
Ans. (b) : Given that,
Capacitor (C1) = 3 µF (a) 24 (b) 32
and C2 = 6 µF (c) 8 (d) 16
Potential V1 = V2 = 12 V AP EAMCET (22.04.2019) Shift-II
The common potential across capacitors is given by – Ans. (b) : Given that,
C1V1 + C 2 V2
V= Capacitance, C = 2 µF
C1 + C 2
Potential, V = 1 kV = 1000 V
3 × 10 −6 × 12 + 6 × 10 −6 × 12 C1 = 1 µF
V=
( 3 + 6 ) ×10−6 Potential difference is not more that 300 V
36 × 10 −6 (1 + 2 )
V=
9 ×10 −6
12 × 10 −6
V=
10−6
V = 12 V
349. The charge on 4µF capacitor, in the given
circuit is So, for maximum bearable capacitance, we take n= 4,
In series,
1 1 1 1 1
= + + + =4
C 1 1 1 1
C = 1/4 µF
Total capacitance of rows,
1 m
m × = µF
4 4
Objective Physics Volume-III 907 YCT
Since, it is given that the total capacitance of circuit is 2 1
(a) µF
µF. 5
m 31
2µF = µF (b) µF
4 5
m = 2µF × 4µF = 8µF (c) 48µF
Thus, the minimum number of capacitor = m × n (d) 36µF
= 8 × 4 = 32
Karnataka CET-2011
351. In this diagram, the PD between A and B is 60
Ans. (c) : Given , equivalent capacitance between point
V, The PD across 6 µ F capacitor is : A and B is (Ceq) = 3µF

(a) 4V (b) 10V


(c) 5V (d) 20V
Karnataka CET-2012
Ans. (b) :

∴Charge on the combination,


Q = VC
= 60×1 = 60µC
∴Potential difference (PD) across 6µF capacitors,
Q 60
V1 = = = 10V
C 6
352. In the given network, the value of C, so that an
equivalent capacitance between points A and B
is 3 µF, is :

1 1 1
= +
Ceq C 16 / 5
1 1 1 1 5 1
= + ⇒ − =
3 C 16 / 5 3 16 C
1 16 − 15 1
= =
C 48 48
C = 48 µF

Objective Physics Volume-III 908 YCT


353. All capacitors used in the diagram are identical
and each is of capacitance C. Then the effective
capacitance between the point A and B is :

(a) 1.5 C (b) 6 C


(c) 1C (d) 3C
Karnataka CET-2010 In series,
1 1 1 1
Ans. (a) : = + =
Ceq 6 6 3
Ceq = 3µF

In parallel series, Charge in the circuit,


Q = VC = 12 × 3 = 36 µC
Since, the capacitors 4 µF and 2 µF are connected in
parallel,
Q1 C1 4
= = ⇒ Q1 = 2Q 2
Effective capacitance between the point A and B, Q2 C2 2
In Series, Also, Q = Q1 + Q2
1 1 1 2
= + = 36 = Q1 + Q2 = 2Q2 + Q2
Ceq 3C 3C 3C Q2 = 12 µC
3C
Ceq = = 1.5C
2
354. The charge deposited on 4µF capacitor in the
circuit is :

∴ Q1 = Q – Q2
= 36 – 12
= 24 µC = 24 × 10–6C
355. If the equivalent capacitance between P and Q
of the combination of the capacitors shown in
(a) 6 × 10–6 C
figure below is 30 µF, the capacitor C is
(b) 12 × 10–6 C
(c) 24 × 10–6 C
(d) 36 × 10–6 C
Karnataka CET-2009
Ans. (c) :

(a) 60 µF (b) 30 µF
(c) 10 µF (d) 5 µF
J&K CET- 2011
Objective Physics Volume-III 909 YCT
Ans. (a): Effective capacitance of C2 and C3 are in series,
1 1 1
= +
Ceq 2 2
Ceq = 1µF
Now, C1 and Ceq are in parallel,
C' = 1 + 1 = 2µF
eq

Now, C'eq and C4 are in series,


1 1 1 3
From the figure, = + =
''
Ceq 2 4 4
Capacitance are in parallel,
∴ Equivalent capacitance, 4
C''eq = µF
C' = 20 µF + 20 µF + 20 µF = 60 µF 3
Now, Q Effective capacitance between the points A and B is
4
µF
3
357. Four capacitors each of 1 µF are connected as
C and 60 µF are in series and equivalent to Capacitance shown. The equivalent capacitance between P
30 µF and Q is
Then,
1 1 1
= +
30 C' C
1 1 1 2 −1 1
= − = =
C 30 60 60 60
C = 60 µF
356. Four capacitors are connected in a circuit as
shown in the following figure. Calculate the
effective capacitance between the points A and B 1
(a) 4 µF (b) µF
4
3 4
(c) µF (d) µF
4 3
J&K CET- 2003
Ans. (c) :

4 24
(a) µF (b) µF
3 5
(c) 9 µF (d) 5 µF
J&K CET- 2008
In parallel, point P and S
Ans. (a) : Ceq = 1 + 1 + 1 = 3µF

In series, point P and Q, we get –


1 1 1 4
'
= + =
Ceq 3 1 3
3
C'eq = µF
4

Objective Physics Volume-III 910 YCT


358. The equivalent capacitance in the following C' = 2µF
circuit is Q C' and C3 are in parallel connection,
Ceq = 2 + 3 = 5µF
360. Three capacitors 3µF, 6µF and 6µF are
connected in series to a source of 120 V. The
potential difference, in volt, across the 3µF
capacitor will be
(a) 2 µF (b) 1 µF
(a) 24 (b) 30
(c) 1.5 µF (d) 3 µF (c) 40 (d) 60
J&K CET- 2000
Ans. (d) :
Ans. (b) :

Since, capacitor 1.5 µF and 1.5 µF are in parallel, Equivalent capacitance for series connection,
∴ 1.5 µF + 1.5 µF = 3 µF 1 1 1 1 4 1
Now, = + + = =
Ceq 3 6 6 6 1.5
Ceq = 1.5 µF
Q Charge (Q) = VCeq
Equivalent capacitance in point A to B in series, Q = 120 × 1.5
1 1 1 1 3 Q = 180 µC
= + + =
Ceq 3 3 3 3 Potential difference across 3µF,
1 Q 180
= 1 µF V= =
Ceq 3 3
V = 60 Volt
Ceq = 1 µF
361. Equivalent capacitance between A and B in the
359. In the figure below, the capacitance of each figure is
capacitor is 3 µF. The effective capacitance
between A and B is

3
(a) µF (b) 3 µF (a) 20 µF (b) 8 µF
4
(c) 6 µF (d) 5 µF (c) 12 µF (d) 16 µ↓↓↓F
WB JEE 2016
Ans. (d) :
Ans. (b) :

C2 and C4 are in Parallel connection,


C = 3 + 3 = 6 µF
Q C1 and C are in series connection,
1 1 1 3 1
= + = =
C' 3 6 6 2

Objective Physics Volume-III 911 YCT


Equivalent resistance in triangle DCE, Resistance in Capacitors in series,
side DC and EC are in series. So, 2C × V1 = C × V2
1 1 1 2 2V1 = V2 …(i)
= + = Q V1 + V2 + V1 = 60 …(ii)
C 4 4 4
From equation (i) and (ii), we get
C = 2 µF
V2 = 30 V
V1 = 15 V
363. Four capacitors of equal capacitance have an
equivalent capacitance C1 when connected in
series and an equivalent capacitance C2 when
connected in parallel. The ratio C1/C2 is
(a) 1/4 (b) 1/16
(c) 1/8 (d) 1/12
WB JEE 2009
Resistance in side AB are in series. Ans. (b) : For series combination of capacitance,
1 1 1 2 1 1 1 1 1 4
= + = = + + + =
C' 4 4 4 C1 C C C C C
C' = 2µF C1 = C/4
Now, Equivalent resistance Now, parallel combination of capacitances,
C2 = C + C + C + C = 4C
Now,
C1 C / 4 1
= =
C2 4C 16
The ratio of C1 /C2 =1/16
364. The equivalent capacitance between points P
and Q in the following circuit is:
Now, all capacitor are in parallel,
So,
Ceq = 2 + 4 + 2
Ceq = 8 µF
(a) 2 µF (b) 4 µF
362. In the adjoining figure, the potential difference
8 16
between X and Y is 60V. The potential (c) µF (d) µF
difference between the points M and N will be 3 3
TS EAMCET 29.09.2020, Shift-II
Ans. (c) :

(a) 10 V (b) 15 V
(c) 20 V (d) 30 V
WB JEE 2012
Ans. (d) :

V1 Connected in series in upward arm


V1 V2 1 1 1
V2 = +
C1 2 4
1 3
=
Potential difference between X and Y, C1 4
V = 60 V C1 = 4/3 µF

Objective Physics Volume-III 912 YCT


Similarly connect in series, in backward arm, C1 × C 2 2 × 3 6
C= = = µF
C2 = 4/3 µF C1 + C 2 2 + 3 5
Between P and Q in C1 and C2 are connected in parallel Charge in each capacitor,
configuration, q = CV
C = C1 + C2 6
4 4 q = × 10
C= + 5
3 3 q = 12 µF
8
C= µF ∴ Potential difference across 2 µF in capacitor,
3
q
365. How will you connect 4 (four) capacitors, each V1 =
C1
of capacitance 4 µF for having equivalent
capacitance 1.6 µF ? 12 × 10 −6
V1 =
(a) Two in parallel and two in series 2 × 10 −6
(b) All four in series V1 = 6V
(c) All four in parallel
367. The number of ways one can arrange three
(d) Three in parallel and one in series identical capacitors to obtain distinct effective
GUJCET 18.04.2022 capacitances is
Ans. (a) : (a) 8 (b) 6
(c) 4 (d) 3
J&K CET- 2010
Ans. (c) :
case1 –

C
Ceq =
3
case2 -
Two in parallel and two in series
366. The potential difference across 2 µF capacitor
in the circuit shown in 2C
Ceq =
3
case3 -

Ceq = 3C
(a) 12 V (b) 4 V
(c) 6 V (d) 18 V
CG PET-2021 case 4-
Ans. (c) :

3C
Ceq =
2
So, for distinct equivalent capacitance 4 ways are
Net emf in the circuit, in figure, possible.
Enet = E2 – E1 368. Five identical capacitors, of capacitance 20µF
Enet = 16 – 6 each, are connected to a battery of 150 V, in a
Enet = 10 V combination as shown in the diagram. What is
While the equivalent capacitance, the total amount of charge stored?
Objective Physics Volume-III 913 YCT
Capacitor in parallel,
C' = C3 + C1 = C + C = 2C
=2×9
'
C = 18 PF
'
Now, C and C2 are connected in series, the effective
(a) 15 × 10–3C (b) 12 × 10–3C capacitance is given by -
(c) 10 × 10–3C (d) 3 × 10–3C 1 1 1
= +
WB JEE 2019 C C' C 2
''
Ans. (d) : Given, V = 150V, C = 20 µF 1 1 1 3
Given network is balanced whetstone's bridge Hence = + =
C '' 18 9 18
equivalent circuit is,
C" = 6µF
''
Now, C and C4 are connected in parallel,
Hence the equivalent capacitance between A and B is –
Ceq = C'' + C4
Ceq = 6 + 9
Ceq = 15 µF
370. The total charge on the system of capacitance
C1 = 1µF, C2 = 2µF,C3 = 4µF and C4 = 3µF
connected in parallel is
(Assume a battery of 20V is connected to the
combination)
(a) 200µC
So, equivalent capacitance = 10µF + 10µF = 20 µF
Total charge stored, Q = CV (b) 200C
Q = 20 µF × 150 V (c) 10µC
−3
Q = 3 × 10 C (d) 10C
369. In the arrangement of capacitors shown in the JEE Main-26.07.2022, Shift-I
figure, if each capacitor is 9 PF, then the Ans. (a) :
effective capacitance between the points A and
B is

(a) 10 PF (b) 15 PF
(c) 20 PF (d) 5 PF
UP CPMT-2012
TS EAMCET (Engg.)-2015
Ans. (b) : Given, C1 = C2 = C3 = C4 = C = 9PF
q = CV
total charge = q1 + q2 + q3 + q4
= C1V + C2V + C3V + C4V
= V (C1 + C2 + C3 + C4)
= 20 [1 + 2 + 4 + 3] × 10–6
= 20 × 10 × 10–6
= 200 µC

Objective Physics Volume-III 914 YCT


371. The equivalent capacitance between the points 373. 25 capacitors each of capacitance 1µF are
A and B of the network shown in the figure is connected in series to a battery of 100V. The
total charge stored on capacitors is–
(a) 2.0 × 10–5C
(b) 2.5 × 10–3C
(c) 4.0 × 10–6C
(d) 1.5 × 10–6C
AP EAMCET-07.07.2022, Shift-II
Ans. (c) : Given,
Number of capacitor (N) = 25
(a) 100F (b) 50F Capacitance of each capacitor (C) = 1µF = 1 × 10–6F
(c) 150F (d) 60F Voltage (V) = 100 V
AP EAMCET-25.04.2017, Shift-I We know that,
Ans. (d) : Capacitor C2 and C3 are connected in series. Total charge stored on capacitor = Ceq V
1 1 1 2 1
= + = = The capacitor are connected in series,
Ceq 150 150 150 75
1 1 1 1 1
Ceq = 75 F ∴ = + + + ...........
Ceq C1 C 2 C3 C 25
Capacitor Ceq And C1 are in parallel.
Hence, their equivalent capacitance, 1 1
= −6 (1 + 1 + 1 + 1 + ........... + 1)
C′eq = Ceq + C1 = 75 + 75 = 150 F C eq 10
Capacitor C'eq and C4 are connected in series. 1 25
=
So, Ceq 10 −6
1 1 1 1 1 5
= + = + = 10 −6
C ''
eq C '
eq
C 4 150 100 300 ∴ Ceq =
25
300
C''eq = Total charge stored in the total capacitor,
5 Q = Ceq × V
C''eq = 60 F 10 −6
= × 100
372. The equivalent capacity between the points X 25
and Y in the circuit with C = 1 µF is Q = 4 × 10–6 C
374. Four identical metal plates are located in air at
equal distance d from each other. The area of
each plate is S. If the outer most plates are
connected by a conducting wire as shown in the
figure the capacitance between points A and B
will be

(a) 2 µF (b) 3 µF
(c) 1 µF (d) 0.5 µF
AP EAMCET(Medical)-2007
MP PET-2012
ε 0S 3 ε0S
Ans. (a) : Given, C = 1 µF (a) (b)
d 2 d
1 ε0S 2 ε0S
(c) (d)
2 d 3 d
TS-EAMCET-06.05.2019, Shift-1
Capacitor across AB is short circuited so no charge Ans. (b) : Given that,
flows through it. It is redundant. Area of plate = S
∴ Ceq = C + C = 2C = 2 × 1 = 2µF Distance between two plate = d

Objective Physics Volume-III 915 YCT


ε0S In steady state, when all the capacitors are charge, no
Capacitance (C) =
d current flow in the circuit so, potential across resistors is
So, equivalent capacitance between point A and B, zero.
1 1 1 2 As in parallel combination, potential is same in both
= + =
C' C C C wires CD and EG, i.e. l00V.
C q
C' = V=
2 C
And C′ and C are in parallel- VAF 4 2
CAB = C' + C Q = =
VFB 14 7
C 3
= +C= C 2
2 2 ∴ VAF = × 100 = 28.57V
3 ε0S 7
∴ C AB = × VFB = 100 – VAF
2 d
375. = 100 – 28.5
5µF F 2µF = 71.5 V
E G 376. When two capacitors are connected in parallel
5µF 2µ F the resulting combination has capacitance
C D 10 µF .The same capacitors when connected in
2µF series results in a capacitance 0.5 µF .The
20Ω
respective values of individual capacitors are
A 40Ω B
100V (a) 1.9 µF and 0.2 µF
Find potential difference points A & F and F & (b) (8+2 5 ) µF and (2–2 5 ) µF
B.
(a) VAF= 10.2V, VFB = 15.4 V (c) (5+2 5 ) µF and (5–2 5 ) µF
(b) VAF= 22.3V, VFB = 28.9 V (d) 12 µF and 1.7 µF
(c) VAF= 28.5V, VFB = 71.4 V (e) 5 µF and 2 µF
(d) VAF= 42.1V, VFB = 53.1 V
Kerala CEE -2018
TS-EAMCET.11.09.2020, Shift-2
Ans. (c) : According to question,
Ans. (c) :

In parallel connection,
CP = 10 µF
CP = C1 + C2 = 10µF ....(i)

Objective Physics Volume-III 916 YCT


In series connection, So, series connection,
1 1 3 1 11
= + + =
C AB C 4C C 4C
4C
C AB =
11
CS = 0.5µF
1 1 1 C1 + C 2 378. Find the equivalent capacitance between two
= + = points A&B, for given figure (electric circuit)
CS C1 C 2 C1C 2
[Capacitance of each capacitor is C=3 µF ]
C1C 2
CS =
C1 + C 2
(From (i) C1 + C2 = 10 µF)
C1C 2
0.5 =
10
C1C2 = 5 µF ....(ii)
2 2
Q (C1 – C2) = (C1 + C2) – 4C1C2
(a) 1 µF
= (10)2 – 4 × 5 = 80 µF
(b) 3 µF
C1 – C2 = 4 5µF ....(iii)
(c) 2 µF
From equation (i) and (iii), we get -
(d) 4 µF
( ) (
C1 = 5 + 2 5 µF , C2 = 5 − 2 5 µF ) GUJCET-PCE- 2021
377. A network of six identical capacitors, each of Ans. (c) : Given, C = 3 µF
value C, is made as shown in the figure.

The equivalent capacitance between the points


A and B is :
4C
(a) (b) 3C/4
11
(c) 3C/2 (d) 3C
(e) 4C/3 Series connection,
Kerala CEE 2006 1 1 1 3
Ans. (a) : In the given circuit capacitor's (1), (2) and (3) = + =
C AB 2C C 2C
are connected in series, hence equivalent capacitance
is– 2C
C AB = (Q C = 3 µF)
3
2×3
CAB =
3
C AB = 2µF
379. The equivalent capacitance CAB between A and
B for the combination drawn here is
1 1 1 1 3 C
= + + = ⇒ C' =
C' C C C C 3
Adding to the capacitor in parallel,
C '' = C '+ C
C 4C
C" = + C =
3 3
(a) 4µF (b) 2µF
(c) 5µF (d) 3µF
Tripura-2021

Objective Physics Volume-III 917 YCT


Ans. (d): Given circuit, C3 ( C1 + C 2 ) V
Q=
C1 + C 2 + C3
If the capacitor C3 breaks down,
C'eq = C1 + C2
Charge, Q' = C'eq V = (C1 + C2)V
Change in total charge,
C3 ( C1 + C 2 ) V
In down arm, 2µF and 2µF are in series. Q' – Q = (C1 + C2)V –
C1 + C 2 + C3
So,
In upward arm, 2µF and 2µF are in series.  C3 
Q' = ( C1 + C 2 ) V 1 − 
So,  C1 + C 2 + C 3

Now, middle arm capacitor 1µF are in parallel 381. Two identical air core capacitors are connected
combination with C1 and C2. in series to a voltage source of 15 V. If one of
Therefore, equivalent capacitance Ceq = C1 + C2 + 1 the capacitors is filled with a medium of
Ceq = 3µF dielectric constant 4, the new potential across
this capacitor is
∴ CAB = 3 µF
(a) 5 V (b) 8 V
380. Three capacitors C1’ C2 and C3 are connected (c) 10 V (d) 12 V
as shown in the figure to a battery of V volt. If Manipal UGET-2017
the capacitor C3 breaks down electrically the
Ans. (c) : According to question,
change in total charge on the combination of
capacitors is

1 1 1
= +
C1 C C
(a) (C1 + C2) V [ 1- C3 / (C1 + C2 + C3)]
C1 = C/2
(b) (C1 + C2) V [ 1 - (C1 + C2 )/(C1 + C2 + C3)]
Then charge (Q) = C1V
(c) (C1 + C2) V [ 1 + C3 / (C1 + C2 + C3)]
C
(d) (C1 + C2) V [ 1- C2 / (C1 + C2 + C3)] = 15 × …..(i)
2
AIIMS-2010 When it is filled with dielectric,
Ans. (a) Given circuit, C1 = 4C , C2 = C
1 1 1
= +
C3 4C C
4C
C3 =
5
C1 and C2 are in parallel and C3 in series, Charge is conserved,
1 1 1 Q = C3V′
= + 4C
Ceq C3 C1 + C 2 Q= ×V' …. (ii)
5
C3 ( C1 + C 2 ) From equation (i) and (ii), we get -
Ceq =
C1 + C 2 + C3 C 4C
× 15 = V'
Hence, 2 5
Total charge, V′ = 9.4 V
Q = CeqV V '  10 Volt

Objective Physics Volume-III 918 YCT


Aε0 (K1 × K 2 ) Aε 0 (K1 − K 2 )
(a) A (b)
(d) Effect of Dielectric Charging d(K1 + K 2 ) d
and Discharging of Capacitor Aε 0 K1K 2 Aε 0 (K1 + K 2 )
(c) (d)
(K1 + K 2 ) d
382. Choose the correct relation between
AIIMS-2001
polarization ‘P’ and electric susceptibility ‘χe’
of dielectric material. (E = electric field) Ans. (d) : In parallel combination, the equivalent
capacitance is,
χ χ
(a) P = e2 (b) P = e Ceq = C1 + C2
E E
(c) P = χeE (d) P = χe2 E ε AK1 ε 0 AK 2
Ceq = 0 +
d d
MHT-CET 2020
εA
Ans. (c) : Ceq = 0 ( K1 + K 2 )
d
Polarization(P)
Electric susceptibility (χe) = 385. In a parallel plate with air capacitor of
Electric field (E)
capacitance 8 µF, if the lower half of the air
P = χeE space is filled with a material of dielectric
383. A capacitor is charged by using a battery which constant 3, its capacitance changes to:
is then disconnected. A dielectric slab is (a) 3 µF (b) 12 µF
introduced between the plates which results in
(c) 16 µF (d) 24 µF
(a) increase in the potential difference across the
TS EAMCET 28.09.2020, Shift-II
plates and reduction in stored energy but no
change in the charge on the plates Ans. (c) : Given, k = 3
(b) decrease in the potential difference across the εA
Initial capacitor (C) = 0 = 8µF
plates and reduction in the stored energy but d
no change in the charge on the plates εA kε A
Ceq is parallel combination of 0 and 0 .
(c) reduction of charge on the plates and increase d 2d
of potential difference across the plates
The area has become half of original for each capacitor
(d) increase in stored energy but no change in in parallel.
potential difference across the plates
ε A (1 + k )
Ans. (b) : If a dielectric slab of dielectric constant k is Ceq = 0
filled in between the plates of a capacitor after charging 2d
the capacitor the potential difference between the plate ε A 1+ 3 
Ceq = 0  
1 d  2 
reduces to times and potential energy of capacitor
k
Ceq= 8 × 2
1
reduces to times but there is no change in the charge Ceq = 16 µF
k
on plates. 386. A capacitor of capacitance 50 pF is charged by
384. Two materials having the dielectric constants 100 V source. It is then connected to another
K1 and K2 are filled between two parallel plates uncharged identical capacitor. Electrostatic
of a capacitor, which is shown in figure. The energy loss in the process is _____ nJ.
capacity of the capacitor is : JEE Main-27.06.2022, Shift-I
Ans. (125) : Given,
C1 = 50 pF, V1 = 100 V
If it is connected to another uncharged identical
capacitor the capacitance and voltage becomes.
C2 = 50 pF, V2 = 0

Objective Physics Volume-III 919 YCT


1 C1C2 Work done = Change in potential energy
( V1 − V2 )
2
Energy loss =
2 C1 + C2 Q1Q 2 1 1
=  − 
4πε0  r2 r1 
1 50 × 10−12 × 50 × 10−12
× (100 − 0 )
2
= ×
2 ( 50 ×10 + 50 × 10 )
−12 −12
 1 1 
= 9 × 109 × 10 × 12 × 10–12  − 
 0.06 0.1 
=125 nJ
So, the energy loss in whole process is 125 nJ. = 7.2 8.1J

387. A parallel plate capacitor is formed by two 389. A capacitor of capacitance 20 µF is charged by
a battery of potential 24.3 V. The capacitor is
plates each of areas 30π cm2 separated by then disconnected from the battery and is
1mm. A material of dielectric strength 3.6 × 107 connected to another uncharged capacitor of
Vm–1 is filled between the plates. If the capacitance 10 µF. After some time, the second
maximum charge that can be stored on the capacitor is disconnected, discharged fully and
capacitor without causing any dielectric is again connected to the first capacitor. If the
breakdown is 7 × 10–6 C, the value of dielectric process is repeated several times, the charge on
constant of the material is: the first capacitor at the end of the fifth process
 1  is ––––––– µC.
 Use = 9×109 Nm 2C-2 
 4πε 0  (a) 256 (b) 128
(a) 1.66 (b) 1.75 (c) 64 (d) 32
(c) 2.25 (d) 2.33 AP EAMCET-24.04.2017, Shift-II
JEE Main-24.06.2022, Shift-I Ans. (c) : Charge on 20 µF capacitor (Q) = CV
AP EAMCET-23.04.2019, Shift-II Q = 20 × 24.3 µC
Then it is connected to 10µF capacitor.
Ans. (d) : Given, Area (A) = 30π cm2, E = 3.6 × 107
1 2
Vm–1, Q = 7 × 10–6C, ε0 = So, the share on 20µF capacitor, × Q
4π× 9 × 10 9 3
5
σ 2
Electric field inside the dielectric (E) = After 5th time =   × Q
Kε 0 3
32
E=
Q

Q = × 20 × 24.3
AKε0 AEε0 243
= 64µC
7 × 10−6
K= 390. A capacitor is discharged through a resistance
1
30π× 10−4 × 3.6 ×107 in R-C circuit. The variation of logei with time t
4π× 9 ×109 is shown by a dotted line in the figure, where i
36 × 7 is the discharging current. If the resistance in
K= the circuit be doubled, the variation of logei
30 × 3.6 with time t would be best represented by the
K = 2.33 line
388. Two positive point charges of 10 µC and 12 µC
are placed 10 cm apart in air. The work done
to bring them 6 cm closer is
(a) 8.1 J (b) 3.2 J
(c) 9 J (d) 13.5 J
AP EAMCET-04.07.2022, Shift-I
(a) P (b) Q
Ans. (a) : Given, r1 = 10 cm = 10×10–2 = 0.1m
(c) R (d) S
r2 = 6cm = 6×10–2 =0.06, Q1 = 10 µC, Q2 = 12 µC
AMU-2013
1 Q1Q 2
Initial potential energy = Ans. (b) : In R-C circuit, Discharging current at any
4πε0 r1 time t is -
1 Q1Q 2 E − RCt
Final potential energy = i= e
4πε0 r2 R

Objective Physics Volume-III 920 YCT


Taking log from both sides 22 3
(a) (b)
E t  1  E 3 22
loge i = log e − = −  t + log e
R RC  RC  R 7 4
(c) (d)
1 4 7
The slope logei versus t graph = − AIPMT-2005
RC
Ans. (b) : Equivalent capacitance for three capacitor in
Hence, when R doubled, the magnitude of slope
series are
decrease and current also decreases.
1 1 1 1
So, graph represent these fact. = + +
Ceq C1 C 2 C3
391. Two condensers, one of capacity C and the
Given, C1 = C , C2 = 2C, C3 = 3C
C
other of capacity , , are connected to a V volt 1 1 1 1
2 = + +
battery, as shown. C eq C 2C 3C
1 ( 2C )( 3C ) + ( C )( 3C ) + ( C )( 2C )
=
Ceq ( C )( 2C )( 3C )
1 6C 2 + 3C 2 + 2C 2
=
Ceq 6C3
1 11C 2
=
Ceq 6C3
The work done in charging fully both the
condensers is 6C
Ceq =
11
2 1 2
(a) 2CV (b) CV Charge on capacitor in series
4
Q = Ceq V
3 2 1 2
(c) CV (d) CV 6C
4 2 = V
11
AIPMT-2007
Charge on capacitor C4 = 4CV
Ans. (c) : The capacitor in circuit are in parallel order
Charge on C 2 6C V
C 3C =
C' = C + = Charge on C 4 11
2 2 4CV
The work done in charging the equivalent capacitor is 6 3
= =
stored in the form of potential energy. 11× 4 22
1 393. A parallel plate condenser with oil (dielectric
W = C ' V2 constant 2) between the plates has capacitance
2
C. If oil is removed, the capacitance of
1 3C 2 capacitor becomes
= V
2 2
(a) 2 C (b) 2 C
3 2
W = CV C C
4 (c) (d)
2 2
392. A network of four capacitors of capacity equal
AIPMT-1999
to C1 = C,C2= 2C, C3 = 3C and C4 = 4C are
connected to a battery as shown in the figure. Ans. (d) : When oil is present then the capacitance =
The ratio of the charges onC2 and C4 is KAε 0 2Aε 0
= =C
d d
ε0 A C
=
d 2
When oil is removed, the capacitance becomes
ε0 A C
C' = =
d 2

Objective Physics Volume-III 921 YCT


394. Two thin dielectric slabs of dielectric constants The electric field inside the dielectric will be less than
K1 and K2 (K1 < K2) are inserted between plates the electric field outside the dielectrics. Hence, the
of a parallel plate capacitor, as shown in the electric field inside the dielectrics cannot be zero.
figure.

Thus, it is clear that K2 > K1, the drop in electric field


must be more than K1.
395. The plates in parallel plate capacitor are
separated by a distance with air as the medium
The variation of electric field E between the between the plates. In order to increase the
plates with distance d as measured from plate P capacity by 66% a dielectric slab of dielectric
is correctly shown by constant 5 is introduced between the plates.
What is the thickness of the dielectric slab?
d d
(a) (b)
4 2
(a) (b) 5d
(c) (d) d
8
AP EMCET(Medical)-2010
Ans. (b) :

(c) (d)

AIPMT-2014
Ans. (c) :
ε0 A
The capacity in air C =
d
The capacity when dielectric slab of dielectric constant
5 is introduced between the plate.
ε0 A
C' =
t
d−t+
K
ε0 A
When a dielectric material is inserted in between the C' =
t
plates of capacitor, the dielectric becomes electrically d−t+
5
polarized. The polarization charges induced on the two
uur 166
faces of the slab produce their own electric field E 0 , C' = C
uur 100
which opposes the external field E 0 .
C ' 166
ur = ..........(i)
Hence, the resultant field E within the dielectric is C 100
uur uur
smaller than E 0 but is in the same direction as E 0 and is C' d
ur
ur E 0
= ........ (ii)
C d−t+ t
given by E = .
K 5

Objective Physics Volume-III 922 YCT


Putting value of eqn (i) in (ii), we get 2 6 1
= −
t 4t 3K 7 3
d−t+ d−
100 5 = 5 2 11
= =
166 d d 3K 21
 4t  42 = 33K
100d = +166d − 166  
5 K=
42
 4t  33
166   = 66d 14
5 K=
11
66d × 5
∴t= 397. A parallel plate capacitor of capacity 5 µF and
166 × 4
plate separation 6 cm is connected to a 1 V
d battery and is charged. A dielectric of dielectric
t= constant is 4 and thickness is 4 cm introduced
2
into the capacitor. The additional charge that
d
sSo, is the thickness of dielectric slab flows into the capacitor from the battery is:
2 (a) 2 µC (b) 3 µC
7 (c) 5 µC (d) 10 µC
396. Capacitance of a capacitor becomes times its
6 AP EAMCET(Medical)-2001
original value if a dielectric slab of thickness Ans. (c) : Given, C = 5µf , V = 1 Volt, d = 6 cm, t =
2
t= d is introduced in between the plates d, is 4cm
3 Charge on capacitor Q = CV
the separation between the plates. The = (5 ×10–6 F) × 1 Volt
dielectric constant of the dielectric slab is:
= 5 × 10–6 C
14 11
(a) (b) Q = 5 µC
11 14
The capacitance after dielectric is introduced
7 11
(c) (d) ε0 A ε0 A / d
11 7 C' = =
 t   t 
AP EAMCET(Medical)-2004 d−t −   t− 
Ans. (a) :  K  1−  K

 d 
εA  
Capacitance (C1) = 0 .....(i)
d C 5
C' = =
If electric slab of constant K of thickness t = d
2  t−t/K  4 − 4/4 
1−   1−  
3  d   6 
is introduced, then capacitance becomes, 5
C' = =10µF
ε0 A ε0 A  4 −1 
C2 = = ......(ii) 1−  
 1 1 − t t   6 
d − t 1 −  d  + 
 K  d kd  Now, charge on capacitor plate will be
Equation (i) become Q ' = C ' V = 10 µF × 1V = 10 µC
ε0 A Additional charge transferred = Q ' – Q
C2 =
 2  2  = 10 µC – 5 µC
d  1 −  + 
  3  3K  = 5 µC
C1 398. A dielectric of thickness 5 cm and dielectric
C2 =
1 2  constant 10, is introduced in between the plates
 +  of a parallel plate capacitor having plate area
 3 3K 
500 cm2 and separation between plates 10 cm.
7 C1 The capacitance of the capacitor is (If ε0 =
C1 =
6 1 2  8.8×10-12 SI units):
 + 
 3 3K  (a) 8pF (b) 6pF
1 2 6 (c) 4pF (d) 20pF
+ =
3 3K 7 AP EAMCET(Medical)-1999
Objective Physics Volume-III 923 YCT
Ans. (a) : Given, 400. The plates of a parallel plate capacitor are
t = 5 cm, K = 10, A = 500 cm2, d = 50 cm, charged upto 200 V. A dielectric slab of
thickness 4 mm is inserted between its plates.
We know, Then, to maintain the same potential difference
ε0 A between the plates of the capacitor, the distance
C1 = between the plates is increased by 3.2 mm. The
d
dielectric constant of the dielectric slab is
If dielectric constant introduced between the plate C2 =
Kε 0 A (a) 1 (b) 4
(c) 5 (d) 6
d
So, equivalent capacitance between, C1 and C2 are in EAMCET-2004
series. Ans. (c) : Given,
1 1 1 Voltage = 200V, t = 4mm, d – d' = 3.2mm
= + Charge (q) = CV
Ceq C1 C 2
εA
ε 0 A Kε o A q= 0 V ......(i)
. d
C1C 2
Ceq = = d d
When dielectric is inserted q remains constant and V
C1 + C 2 ε 0 A Kε o A
+ decreases by K
d d
ε0 A
Kε 0 A q= V .......(ii)
=  ' t 
d (1 + K )  d − t + 
 K
−12 −4
10 × 8.85 × 10 × 500 × 10 n
From eq (i) and (ii), we get
=
5 × (1 + 10 ) ×10 −2
t
d '− t +
Ceq = 8.04545×10 –12
8pF K
399. When a dielectric slab of the same area of ⇒ d '− d = t − t
cross-section and thickness equal to 2/3 of the K
separation is introduced between the plates of a 4
parallel plate capacitor, its capacitance ⇒ 3.2 = 4 −
K
becomes 2.25 times the original value. The
dielectric constant of the material of the slab is 4
⇒ = 0.8
(a) 1.5 (b) 4.5 K
(c) 5.0 (d) 6.0 ⇒K=5
EAMCET-1994 401. A 10µF capacitor is charged to a potential
2 difference of 50V and is connected to another
Ans. (d) : Given, t = d, C ' = 2.256 uncharged capacitor in parallel. Now the
3
common potential difference becomes 20 V.
ε0 A
Capacitance without dielectric slab, C = The capacitance of second capacitor is
d
(a) 10 µF (b) 20 µF
Capacitance with dielectric slab of t thickness is,
(c) 30 µF (d) 15 µF
ε0 A
C' = Manipal UGET-2011
t
d−t+ Ans. (d) :
k
ε0 A εA
C' = = 0
2 2d d 2d
d− d+ +
3 3k 3 3k
ε0 A
⇒ 2.25C =
1 2 
d + k 
3 3  totalcharge
Common Potential (V) =
1  ε0 A  Ceq
⇒ 2.25C = Q C = 
1 2   d 
 +  Total charge = CV
 3 3k 
= 10 µF × 50 V
⇒ k = 6.6 ~ 6
Q = 500 µC
Objective Physics Volume-III 924 YCT
500µC 404. The capacitance of a parallel plate capacitor
So, 20 = with air as medium is 3µF. With the
10 + C
introduction of a dielectric medium between
10 + C = 25
the plates, the capacitance becomes 15 µF. The
C = 25 – 10
permittivity of the medium in SI unit is ε0 =
C = 15µF
8.85 × 10–12 SI unit
402. The function of a dielectric in a capacitor is (a) 15 (b) 8.845 × 10–11
(a) to reduce the effective potential on plates
(c) 0.4425 × 10–10 (d) 5
(b) to increase the effective potential on plates
MHT-CET 2020
(c) to reduce the capacitor of plate
(d) to decrease the capacitance Ans. (c) : Parallel plate capacitor capacitance
ε A
Ans. (a) : Dielectric is used to polarized by electric (C) = o
field. It is an Insulator. The main work of the dielectric d
is to store energy and current and release while need.

Kε o A
In dielectric medium capacitance ( C ') =
d
Material in dielectric like
Ceramic, Plastic Mica, Glass,
A
So, C = Kε o
d
Hence, function of a dielectric in a capacitor is to C' 15
K= , K= =5
reduced the effective potential on plates. C 3
403. An air filled parallel plate capacitor charged to ε
potential ‘V1’ is connected to uncharged K=
parallel plate capacitor having dielectric εo
constant ‘K’. The common potential of both is ε = Kε o = 5 × 8.854 × 10–12
‘V2’ . The value of ‘K’ will be
ε = 0.4425 × 10–10
V1 – V2 V1 – V2
(a) (b) 405. Two identical parallel plate air capacitors are
V1 + V2 V2
connected in series to battery of e.m.f. ‘V’. If
V1 – V2 V1 one of the capacitor is inserted in liquid of
(c) (d)
V1 V1 – V2 dielectric constant ‘K’, then potential
difference of the other capacitor will become
MHT-CET 2020
K +1 K
Ans. (b) : Air filled capacitor, capacitance C1 = C (a) (b)
KV V (1 – K )
Capacitance of the dielectric field capacitor C2 = KC
Q + Q2 K KV
Common potential V2 = 1 (c) (d)
C1 + C 2 V ( K + 1) K +1
Uncharge dielectric filled capacitor Q2 = 0 MHT-CET 2020
C1V1 + 0 Ans. (d) :
V2 =
C1 + C 2
CV1
V2 =
C + KC
V1
V2 =
1+ K Initial capacitor capacitance = C
V1 − V2 After dielectric C2 = KC
K=
V2 C1 = C

Objective Physics Volume-III 925 YCT


Potential difference A cross capacitor C2 (a) 4 µA (b) 6 µA
C2 V KCV KV (c) 8 µA (d) 2 µA
V2 = = =
C1 + C 2 C + KC K + 1 MHT-CET 2020
KV Ans. (b) : Given,
V2 =
K +1 dV
C = 2µF, = 3V / s
406. Two spherical conductors of capacities 3µF and dt
2µF are charged to same potential having radii Charge across the plate of capacitor (Q) = CV
3 cm and 2 cm respectively, if σ1 and σ2 Differentiating w.r.t. time-
represent surface density of charge on
respective conductors then σ1/σ2 is dQ dV
=C
(a) 2/3 (b) 3/4 dt dt
(c) 1/3 (d) 5/3 dV
i=C
MHT-CET 2020 dt
Ans. (a) : i = 2 × 3×10–6 = 6 µA
408. Two capacitors of capacitance C are connected
in series. If one of them is filled with dielectric
substance K, what is the effective capacitance?
KC
(a) (b) C (K+1)
(1 + K )
KQ
Potential for spherical conductors (V) = 2KC
R (c) (d) None of these
1+ K
V Q
= Manipal UGET-2010
K R
Ans. (a) : Given, Two capacitors of capacitance = C
Q
= constant dielectric constant = K
R
When plates of capacitor are separated by a dielectric
Q1 Q 2 medium of dielectric constant K,
=
R1 R 2 Kε 0 A
It is Cm = = KC0
Q1 R 1 d
= ..... (i)
Q2 R 2 Cm = KC0 [Here, C0 = C]
Q Therefore, Cm = KC
σ= Their effective capacitance
A
Q 1 1 1
σ= = +
4πR 2 Ceq KC C
Q1 1 1+ k
=
σ1 4πR 12 Ceq KC
=
σ2 Q2 KC
4πR 22 ∴ Ceq =
K +1
σ1 Q1 R 22 409. The dielectric constant of air is
= ×
σ2 Q 2 R12 (a) 8.9×10−12 C2N−1m−2 (b) 1
σ1 R 1 R 2 2 (c) infinite (d) None of these
= × CG PET- 2012
σ2 R 2 R 12
Ans. (b) : The dielectric constant of air is equal to 1.
σ1 R 2 2  R 2 = 2cm  The dielectric constant is the ratio of the permittivity of
= =  R = 3cm 
σ2 R1 3  1  a substance to the permittivity of free space.
407. Capacity of a parallel plate air consider is 2µF ε
K=
and voltage between the plates is changing at εo
the rate of 3 V/s. The displacement current in
the capacitor is For air ε = εo, K = 1

Objective Physics Volume-III 926 YCT


Note- Qd
Some dielectric constant of different materials – V' =
εrε0A
Air → 1.00059
V
Bakelite → 4.9 ∴ V' =
8
Nylon → 3.4
Qd Qd
Paper → 3.7 =
εr ε0 A 8ε0 A
410. Calculate the area of the plates of a one farad
parallel plate capacitor if separation between 1 1
plates is 1 mm and plates are in vacuum =
εr 8
(a) 18 × 10 m 8 2
(b) 0.3 × 10 m 8 2
εr = 8
(c) 1.3 × 108m2 (d) 1.13 × 108m2
BITSAT-2009 412. When a capacitor is connected to a battery
Ans. (d) : We know that, capacitance of parallel plate (a) an alternating current flow in the circuit
Kε 0 A (b) no current flow in the circuit
formula C=
d (c) a current flow for sometime and finally it
Where – decreases to zero
A = area of plate (d) current keeps on increasing and reaches
K = Permittivity of dielectric of material maximum after sometime
ε0 = Permittivity of vacuum AP EAMCET (21.09.2020) Shift-I
We have separation between plates is 1 mm.
Ans. (c) : When a capacitor is connected to a battery
εA
So, C = 0 [Q K = 1] current starts flowing in a circuit which charges the
d capacitor until the voltage between plates becomes
dC equal to the voltage of the battery.
A=
ε0 413. In a parallel plate capacitor the separation
between plates is 3x. This separation is filled by
1×10−3
A= = 1.13 × 10 8
m 2
two layers of dielectrics, in which one layer has
8.854 × 10−12 thickness x and dielectric constant 3 k, the
411. The capacity of parallel plate condenser is 5µF. other layer is of thickness 2x and dielectric
When a glass plate is placed between the plates constant 5k. If the plates of the capacitor are
of the condenser, its potential difference connected to a battery, then the ratio of
reduces of 1/8 of the original value. The
potential difference across the dielectric layers
magnitude of relative dielectric constant of
glass is is
(a) 4 (b) 6 (c) 7 (d) 8 1 4
(a) (b)
AP EAMCET (22.09.2020) Shift-II 2 3
Ans. (d) : QPotential of capacitor 3 5
(c) (d)
Q 5 6
V=
C AP EAMCET (21.04.2019) Shift-I
εA
∴ C= Ans. (d) : In ideal capacitor
d
KεA
Q Qd Qd Q ε r =1 for air  C=
∴ V= = =   d
εA εA ε r ε 0 A  & ε = ε r⋅ε0  Given,
d
Qd d1 = x, K1 = 3k
V= ....(i) d2 = 2x, K2 = 5k
ε0 A
If glass plate is inserted then potential 3kε0 A
∴ C1 =
V x
V' =
8 5kε0 A
C2 =
Q Q 2x
V' = ' =
C ε r ε0 A Q V=
Q
d C

Objective Physics Volume-III 927 YCT


Q Q ε0 A / 3 ε0 A
Case (i) V1 = = ∴ C1 = =
C1 3kε0 A d 3d
x εA
C1 = 0 ....(i)
Qx 3d
V1 = ....(i)
3kε0 A ε A / 3 ε0 A
C2 = 0 =
Q Q ( d + d ) 6d
Case-(2) V2 = =
C 2 5kε0 A ε0 A
C2 = ....(ii)
2x 6d
V2 =
2Qx ε A /3 εA
....(ii) C3 = 0 = 0 ....(iii)
5kε0 A ( d + d + d ) 9d
∴ Dividing equation (1) by (2), we get- Equivalent capacitor
Qx Ceq = C1 + C2 + C3 ....(iv)
V1 3kε 0 A Putting the value of equation (i), (ii) & (iii) in equation
= (iv)
V2 2Qx
5kε 0 A ε0 A ε0 A ε0 A
Ceq = + +
3d 6d 9d
V1 Qx 5kε 0 A
= × ε A  1 1
V2 3kε 0 A 2Qx = 0 1 + + 
3d  2 3 
V1 5 ε0 A  6 + 3 + 2 
= =
V2 6 3d  6 
414. A capacitor is made of flat plate of area A and ε A 11 
= 0  
a second plate of stair-like structure as sown in 3d  6 
A
the figure. The area of each stair is and the 11ε A
3 Ceq = 0
height is d. the capacitance of the arrangement 18d
is 415. One plate of a parallel palate capacitor is
connected to a spring as shown in the figure.
The area of each plate of the capacitor is A and
the distance between the plate is d, when the
battery is not connected and the spring is
unscratched. After connecting the battery, in
the steady state the distance between the plates
ε0 A 6ε 0 A is 0.75 d, then the force constant of the spring is
(a) (b)
3d 11d
3ε0 A 11ε0 A
(c) (d)
d 18d
AP EAMCET (20.04.2019) Shift-1
Ans. (d) :
3 ε0 V 2 A 8 ε0 V 2 A
(a) (b)
8 d3 3 d3
9 ε0 V 2 A 32 ε 0 V 2 A
(c) (d)
32 d 3 9 d3
AP EAMCET (22.04.2018) Shift-II
Given data, Ans. (d) : From the equation–
Q Area of each stair is A/3 Force between plates of capacitor = spring force
height of each stair = d We know that,
εA ε = ε0 ε r  E=
Q
Formula, C =   2ε o A
d for air ε r = 1

Objective Physics Volume-III 928 YCT


Q2 t
Then, F = QE = d + 1.6 − t + =d
2ε o A K
t
Q2 1.6 − t + =0
E= = Kx K
2ε 0 A
t
Q2 − + t = 1.6
K= ....(i) K
2ε 0 Ax
2
When battery is not connected, distance between plate = d − + 2 =1.6
K
After connecting the battery distance, d1 = 0.75d
∴ x = d − 0.75d = 0.25d 2
− = 1.6 − 2
Q K
Q V= 2
C − = −0.4
Q = CV K
Put the value of Q in equation (i) 2 2 × 10
K= = =5
( CV )
2
CV 2 2 0.4 4
K= =
2ε 0 A ( 0.25 ) d 2ε 0 A ( 0.25 ) d K=5
2 417. A parallel plate capacitor has a capacity 80 ×
 ε0A  2 ε0 2 A 2
  V .V 2 10–6F, when air is present between its plates.
( )
2
d 0.75d
K=  1 
The space between the plates is filled with a
=
2ε 0 A ( 0.25) d 2ε 0 A ( 0.25 ) d dielectric slab of dielectric constant 20. The
capacitor is now connected to a battery of 30V
ε02 A 2 V 2 by wires. The dielectric slab is then removed.
K=
2 × 0.75 × 0.75d 2 × 0.25 × ε0 A × d Then, the charge passing through the wire is
(a) 12 × 10–3C (b) 25.3 × 10–3C
 ε AV 2 
K = 0 
–3
(c) 120 × 10 C (d) 45.6 × 10–3C
 9 d3  AP EAMCET (22.04.2018) Shift-1
 32 
Ans. (d) : Given, K = 20, V = 30V
32  ε 0 AV 2 
K=   Capacitance in air (Cair) = 80 × 10−6F = 80 µF
9  d3 
Capacitance in dielectric
416. The plates of a parallel plate capacitor are
Cdielectric = K Cair
charges upto 100 V. A 2 mm thick insulator
sheet is inserted between the plates. Then to = 20 × 80
maintain the same potential difference, the Cdielectric = 1600 µF
distance between the plates is increased by 1.6
mm. The dielectric constant of the insulator is Charge in air (qair) = Cair × V = 80 × 30
(a) 6 (b) 8 = 2400 µC
(c) 5 (d) 4 Charge stored in dielectric
AP EAMCET (20.04.2019) Shift-1 qdielectric = Cdielectric × V
Ans. (c) : Given, = 1600 × 30
Initial voltage (V) = 100V, t = 2 mm = 48000 µC
Q When dielectric is removed then charge stored
Q V=
C q = qdielectric − qair
Q Voltage 'V' & charge Q remains constant then the
q = (48000 − 2400) µC = 45600 µC
capacitor must be constant
Cinitial = Cfinal q = 45.6 × 10−3C
ε0 A ε0 A 418. A parallel plate capacitor of area A, plate
= separation d and capacitance C is filled with
d t
d + 1.6 − t + three dielectric materials having dielectric
K constants k1, k2 and k3 as shown. If a single
1 1 dielectric material is to be used to have the
=
d d + 1.6 − t + t same capacitance C in the capacitor, then its
K dielectric constant k is given by

Objective Physics Volume-III 929 YCT


(a) Zero (b) 4πε0a
 b 
(c) 4πε0b (d) 4πε0 a  
b−a 
UPSEE - 2014
K1K 2 Ans. (c) : Given, the Radius of inner sphere = a,
(a) K = K1 + K2 + 2K3 (b) K = + 2K 3
K1 + K 2 The Radius of outer Sphere = b
1 1 1 1 1 1 1 The expression to calculate the capacitance when outer
(c) = + + (d) = + sphere is earthed.
K K1 K 2 2K 3 K K1 + K 2 2K 3
ab
UPSEE - 2015 C1 = 4πε0 ....... (i)
b−a
Ans. (d) : In the given figure, we can divide it into three
different capacitors. Let the capacitance with dielectric The expression to calculate the capacitance when inner
constant K1, K2 and K3 be C1, C2 and C3 capacity, the sphere is earthed
d 4πε0 ab
width of capacitor = and area is A C2 = 4πε0 b +
2 2
b−a

We have, C1 =
( 2 ) K = ε AK
ε0 A 1
0 1
 b2 
C2 = 4πε0   ........ (ii)
(d 2) d  b−a 
Difference between equation (i) and (ii)
ε (A )K ε AK
C2 =
0 2 2
= 0 2  b2  ab
( 2)
d d C2 – C1 = 4πε0   − 4πε0
 b−a  b−a
ε 0 AK 3 2ε 0 AK 3 b − a 
C3 = = C2 – C1 = 4πε0 b 
d ( )
2
d  b − a 
The capacitors C1 and C2 are in parallel C2 – C1 = 4πε0 b
ε0 A 420. Capacitance of a capacitor made by a thin
∴ C' = C1 + C2 = ( K1 + K 2 ) metal foil is 2µF. If the foils is folded with
d
This combination is in series with C3 paper of thickness 0.15 mm, dielectric constant
The net capacitance of paper is 2.5 and width of paper is 400 mm,
the length of the foil will be
1 1 1
= + (a) 0.34 m (b) 1.33 m
C'' C' C3
(c) 13.4 m (d) 33.9 m
1 1 1
= + UPSEE - 2012
C" ε A 2ε 0 AK 3
0
( K1 + K 2 ) Ans. (d) : Given, C = 2 × 10−6 F
d d
1 d d Width of paper (b) = 400 mm = 0.4 m
= + Thickness of paper (t) = 0.15 mm = 15 × 10–5 m
C" ε 0 A ( K1 + K 2 ) 2ε 0 AK 3
Dielectric constant of paper (K) = 2.5
1 d  1 1  As capacitance for parallel plate capacitor is,
=  + 
C" ε0 A  ( K1 + K 2 ) 2K 3  Kε0 A
C=
 ε AK  d
Where,  C" = 0 
 d  Kε0 ( l × b )
C=
1 1 1 t
Then, = +
K ( K1 + K 2 ) 2K 3 Ct
l=
419. A spherical condenser has inner and outer Kε0 b
spheres of radii a and b respectively. The space
2 × 10−6 × 15 × 10−5
between the two is filled with air. The difference l= = 33.89 33.9
between the capacities of two condensers formed ( 2.5 × 8.85 ×10−12 × 0.4 )
when outer sphere is earthed and when inner
sphere is earthed will be l = 33.9 m.

Objective Physics Volume-III 930 YCT


421. A dielectric introduced between the plates of a Ans. (b): Given that,
parallel plate condenser
Dielectric constant, K1 = 8, K2 = 4
(a) decreases the electric field between the plates
We know that,
(b) decreases the capacity of the condenser
(c) increases the charge stored in the condenser εA
C= 0
(d) increases the capacity of the condenser d
UPSEE - 2011 K1 ε 0 A
∴ C = 2 = 8 ε 0 A = 4C
Ans. (d) : A dielectric introduced between the plates of 1
d 2 d
a parallel plate condenser increases the capacity of the
condenser by K times. K 2ε0 A
C2 = 2 = 4 ε 0 A = 2C
Kε0 A d 2 d
C' =
d Capacitance will increase due to parallel combination
 ε0 A  C' = C1 + C2
C' = KC Q
 d = C 
  = 4C + 2C
422. In the condenser shown in the circuit is = 6C
charged to 5 V and left in the circuit, in 12 s the 424. A dielectric slab is inserted between the plates
charge on the condenser will become : of an isolated charged capacitor. Which of the
following quantities remain unchanged?
(a) The charge on the capacitor
(b) The stored energy in the capacitor
(c) The potential difference between the plates
10 e
(a) C (b) C (d) The electric field in the capacitor
e 10
UPSEE - 2005
10 e2 Ans. (a) : If a dielectric slab is inserted between the
(c) 2 C (d) C
e 10 plates of a charged capacitor, the intensity of electric
(e = 2.718 ) field potential difference of capacitor and the energy
UPSEE - 2006 stored all reduce to 1 times and K. Capacity of the
JCECE -2004, 2007 K
capacitor increase K times but the charge on the
Ans. (a) : Given, C = 2F, V = 5V, t = 12 sec, R = 6Ω
capacitor remains unchanged.
Exponential decay of charge take place.
425. Ten identical batteries each of emf 2 V are
In C – R Circuit, discharging take place.
−t
connected in series to a 8 Ω resistor. If the
q = Q e RC current in the circuit is 2A, then the internal
Where, R in resistance, C is Capacitance and resistance of each battery is
Q = CV (a) 0.2 Ω (b) 0.3 Ω
Hence, q = CV e–t/RC (c) 0.4 Ω (d) 0.5 Ω

(
q = 2 × 5× e
−12
)
6× 2
Coulomb (e) 1 Ω
Kerala CEE- 2013
10 Ans. (a) : Given, emf of battery = 2V,
q= C
e Resistance (R') = 8Ω, Current = 2A
423. A capacitor having capacitance 1 µF with air is Then, total potential V = 10 × 2 = 20 Volt
filled with two dielectrics as shown. How many
times capacitance will increase? Total resistance R = nr + R'
R = 10r + 8
V
Current (I) =
R
V
(a) 12 (b) 6 2=
10r + 8
(c) 8/3 (d) 3
10 × 2
UPSEE - 2006 2=
10r + 8
CGPET-2010
Objective Physics Volume-III 931 YCT
10r + 8 = 10 Ans. (e): 1st condition,
2 Air filled capacitor
r=
10
r = 0.2 Ω
Then internal resistance of each battery is 0.2Ω
426. An uncharged parallel plate capacitor filled
with a dielectric of dielectric constant K is
connected to an air filled identical parallel The capacitance
capacitor charged to potential V1. If the
εA εA
common potential is V2, the value of K is C= 0 = 0
d 8
V − V1 V1
(a) 2 (b) nd
2 condition,
V2 V1 − V2
V2 V1 − V2
(c) (d)
V1 − V2 V2
V1 − V2
(e)
V1 + V2
Kerala CEE- 2013
Ans. (d) : Let, air filled capacitance (C1) = C
Dielectric filled capacitance (C2) = K.C Now, capacitance
Air filled capacitor potential = V1 ε0 A ε0 A
C= =
Dielectric filled capacitor potential = 0 (zero)  d1 d 2  d − 4 + 4
 + 
Common potential,  K1 K 2  1 2

V2 =
C1V1 ε0 A εA
= = 0
C1 + C 2 d−4+2 d−2
C.V1 By comparing both condition,
V2 =
C + KC ε0 A ε0 A
=
V1 8 d−2
V2 = d–2=8
1+ K
V1 d = 10 mm
1+K= So, Distance between new plates is 10 mm.
V2
428. Two identical air core capacitors are connected
V in series to a voltage source of 15 V. If one of
K = 1 −1
V2 the capacitors is filled with a medium of
V − V2 dielectric constant 4, the new potential across
K= 1 this capacitor is :
V2
(a) 5 V (b) 8 V
427. The plates of a parallel plate capacitor with air (c) 10 V (d) 12 V
as medium are separated by a distance of 8 (e) 3 V
mm. A medium of dielectric constant 2 and
Kerala CEE 2006
thickness 4 mm having the same area is
introduced between the plates. For the Ans. (c) : Let capacitance of each capacitor is C.
capacitance to remain the same, the distance The resultant of capacitance in series
between the plates is
(a) 8 mm (b) 6 mm
(c) 4 mm (d) 12 mm
(e) 10 mm
Kerala CEE 2007
Objective Physics Volume-III 932 YCT
1 1 1 2 1st condition → capacitor in Air (without Dielectric)
= + =
C' C C C εA
C1 = 0
C d
C' =
2
C
Charge, Q = C'V = ×15 …(i)
2
The capacitors is field with a medium of dielectric
constant 4.
C1 = 4C, C2 = C
d
1 1 1 5 2nd condition now, t= thickness dielectric is
= + = 2
C ' 4C C 4C introduce,
4C
C' =
5
So, charge is conserved
4C
Q = C 'V ' = V' …(ii)
5
From equation (i) & (ii), we get –
C 4C
× 15 = ×V'
2 5
Capacitance,
15 × 5
V' = = 9.4 V 10 V ε0 A 2ε 0 A
4× 2 C2 = =
d d
+ 1 
429. Dielectric constant for a metal is d  + 1
(a) Zero (b) Infinite
2K 2 K 
(c) One (d) Ten
JCECE-2018 2ε 0 A 4 εA
= × 0 [From equation (i)]
Ans. (b) : The expression for dielectric constant K of 1  3 d
d  + 1
any material is K = ε/ε0 where ε is the permittivity of K 
the material and ε0 is the permittivity of a vacuum. K is 2 4
directly proportional to ε, therefore, dielectric constant =
1 3
of a metal is infinite. +1
K
430. The capacitance of a parallel plate capacitor 1 3
4 +1 =
becomes times its original value. If a K 2
3
1 1
d =
dielectric slab of thickness t = is inserted K 2
2
K=2
between the plates (where, d is the distance of
separation between the plates). What is the 431. A parallel plate air capacitor has a capacitance
dielectric constant of the slab? C. When it is half filled with a dielectric of
dielectric constant 5, the percentage increase in
1
(a) K = 2 (b) K = the capacitance will be
2
(a) 400% (b) 66.6%
(c) K = 1 (d) K = 2 (c) 33.3% (d) 200%
JCECE-2017 JCECE-2009
4 Ans. (b) : Given, Initial Capacitance,
Ans. (a) : Given, New capacitance is of original,
3 ε0 A
C=
4 d
C 2 = C1 .....(i)
3 Now, Half filled with dielectric of constant, K = 5

Objective Physics Volume-III 933 YCT


Then New capacitance, 433. A parallel plate capacitor has a capacity C. The
ε0 A separation between the plates is doubled and a
C2 = dielectric medium is inserted between the
d d/2
 +  plates. If the capacity is 3C, then the dielectric
2 K 
constant of the medium will be
ε0 A 2
C2 = × (a) 1.5 (b) 3
d 1+ 1
(c) 6 (d) 12
K
COMEDK 2015
ε0 A 2
C2 = × Ans. (c) : Given, C1 = C, C2 = 3C
d 1+ 1
5 Let’s d1 = d, so, d2 = 2 d
5 Then, dielectric constant (K) = ?
C2 = C
3 ε0 A ε0 A
C1 = =
5 2 d1 d
∴ % increased in capacitance, = − 1 = × 100
3 3
Kε 0 A
= 66.6% Now, C2 =
d2
So, increase in the capacitance is 66.6%
432. The capacitors A and B have identical Kε 0 A
3C =
geometry. A material with a dielectric constant 2d
3 is present between the plates of B. The
K ε0 A
potential difference across A and B are 3C = ×
respectively: 2 d
K K
3C = C1 ⇒ 3C = C
2 2
K=6
434. A parallel plate capacitor has area 2m2
separated by 3 dielectric slabs. Their relative
(a) 2.5 V, 7.5 V (b) 2 V, 8 V permittivity is 2,3,6 and thickness is 0.4 mm,
(c) 8 V, 2 V (d) 7.5 V, 2.5 V 0.6 mm, 1.2 mm respectively. The capacitance
JCECE-2005 is
Ans. (d) : Let’s potential difference across A and B are (a) 5×10–8 Farad (b) 11×10–8 Farad
VA and VB. –8
(c) 2.95×10 Farad (d) 10×10–8 Farad
We know that, Q = CV
COMEDK 2015
1
Then, C ∝ Ans. (c) : Given, Area (A) = 2 m 2
V
And capacitance, K1 = 2, K2 = 3, K3 = 6
CA = C And d, = 0.4 mm, d2 = 0.6 mm, d3 = 1.2 mm
CB = 3 C (Q K = 3) Then, equivalent capacitance,
VA C B ε0 A
∴ = C=
VB C A d1 d 2 d 3
+ +
VA 3C K1 K 2 K 3
=
VB C 8.85 ×10−12 × 2
C= −3
VA = 3 VB …..(i) 0.4 ×10 0.6 × 10−3 1.2 ×10 −3
+ +
We know that, 2 3 6
VA + VB = 10 8.85 × 10−12 × 2
C=
3VB + VB = 10 0.6 × 10−3
4VB = 10 ⇒ VB = 2.5 V
C = 29.5 × 10–9
∴ VA = 3 × 2.5 = 7.5 V
C = 2.95 × 10–8 F.
Objective Physics Volume-III 934 YCT
435. A capacitor is connected to a battery of voltage 437. A capacitor of capacitance 15 nF have
V. Now a dielectric slab of dielectric constant K dielectric slab of dielectric constant 2.5, electric
is completely inserted between the plates, then field strength 30 MV/m and potential
the final charge on the capacitor will be: difference 30 volt. Calculate the area of plate.
εA kε 0 A (a) 6.7×10–4 m2 (b) 4.2×10–4 m2
(a) 0 V (b) V –4 2
d d (c) 8.0×10 m (d) 9.85×10–4 m2
εA AIIMS-25.05.2019(E) Shift-2
(c) 0 V (d) zero
kd Ans. (a) : Given,
AIIMS-26.05.2019(E) Shift-2 Capacitance, C = 15 nF = 15 × 10−9 F
Ans. (b) : Given that, Dielectric constant, εr = 2.5
Voltage = V Electric field strength, E = 30 MV/m
Dielectric constant = K V = 30 V
We know that,
We know,
K ∈0 A
C= …(i) V
d E=
d
Then, the final charge on the capacitor
30
Q=C×V 30 ×106 =
Putting the value of C from equation (i) d
30
 K ∈0 A  d= = 10−6 m
Q= V 30 × 106
 d 
εεA
436. A capacitor of capacitance 9 nF having Then, C = o r
dielectric slab of εr = 2.4, dielectric strength 20 d
MV/m and P.D. = 20V. Calculate the area of A × 8.85 × 10−12 × 2.5
plates. 15 × 10–9 =
10−6
(a) 2.1×10–4 m2 (b) 4.2×10–4 m2
(c) 1.4×10–4 m2 (d) 2.4×10–4 m2 15 ×10−15
A=
AIIMS-25.05.2019(M) Shift-1 8.85 ×10−12 × 2.5
Ans. (b) : Given that, = 6.7 × 10–4 m2
Capacitance, C = 9 nF = 9 × 10−9 F 438. When capacitor is fully charged, find current
Dielectric constant, εr = 2.4 drawn from the cell.
Potential, V = 20 volt
Dielectric strength = 20 MV/m = 20 × 106 V/m
Let, separation between plates = d
We know,
V
E=
d
20 (a) 2 mA (b) 1 mA
20 × 106 =
d (c) 3 mA (d) 9 mA
20 AIIMS-26.05.2019(M) Shift-1
d= = 10−6 m
20 ×106 Ans. (c) : When capacitor is fully charged then it
εεA becomes open circuit hence no current flow. so the
Then, Capacitance (C) = o r equivalent circuit is-
d
8.85 ×10−12 × 2.4 × A
9 × 10–9 =
10−6
9 ×10−15
A=
8.85 × 2.4 × 10−12
= 0.42 × 10–3
= 4.2 × 10–4 m2 Req = R1 + R2 = 1KΩ + 2kΩ = 3kΩ

Objective Physics Volume-III 935 YCT


Hence, current draw from the cell-
V 9
I= = = 3 × 10−3 = 3mA
R eq 3 × 103
439. Calculate charge on capacitor in steady state.

(a) 3µC (b) 6µC


(c) 12µC (d) 24µC
JCECE-2013
Ans. (b) : At steady state condition there is no current
(a) 50 µC (b) 30 µC flow through capacitor.
(c) 45 µC (d) 60 µC E 5
AIIMS-25.05.2019(E) Shift-2 So, I= = = 1A
R2 + r 4 +1
Ans. (c) : In steady state the capacitor becomes open, so
Potential difference across capacitor
current does not flow through the capacitor.
= I × R2
Then current flow through the resistance in the given
circuit. = 1 × 4 = 4V
C
Charge on each capacitor =   × 4
2
3
= × 4 = 6 µC
2
442. A voltage VPQ = V0 cos ωt (where, V0 is a real
V 9
Current, i = = amplitude) is applied between the points P and
R eq 27 ×103 Q in the network shown in the figure. The
Voltage across the capacitor (Vc) = i × R1 values of capacitance and inductance are
9 1 R 3
= × 15 × 103 = 5 V c= and L =
27 × 103 ωR 3 ω
Charge through the capacitor q = CVe Then, the total impedance between P and Q is
= 9 × 10–6 × 5
= 45 × 10–6 C = 45 µC
440. What is the energy stored in a 50 mH inductor
carrying a current of 4A?
(a) 0.2 J (b) 0.4 J (a) 1.5R (b) 2R
(c) 0.05 J (d) 0.1 J (c) 3 R (d) 4 R
JCECE-2004 (e) 2.5 R
Ans. (b) : Given, Kerala CEE -2018
Inductance, L = 50 mH Ans. (c) : Given that,
Current, I = 4A
1
1 Capacitance, C = ,
Energy stored in an inductor = LI 2 ωR 3
2
R 3
1 Inductance, L =
= × 50 × 10−3 × (4)2 ω
2
= 50 × 10–3 × 8
= 0.4 J
441. In the adjoining figure, E = 5 V, r = 1Ω, R2 =
4Ω, R1 = R3 = 1Ω and C=3 µF. The numerical
value of the charge on each plate of the
capacitor is

Objective Physics Volume-III 936 YCT


R 3 Ans. (d) Given,
Z1 = R + j ωL = R + j ω   = R + jR 3
 ω 
dV
Capacitance, C = 2µF , = 3V / s
dt
1 1
Z2 = R – j =R– j Charge across the plate of capacitor q = CV
ωC  1 
ω  dq dV
 ωR 3  Differentiating with respect to time =C
dt dt
= R − jR 3
dV
Displacement current i = C
Q Impedance Z1 and Z2 are in parallel so, dt
Z1 Z2 = 2µF × 3V/s = 6µA
Zeq =
Z1 + Z2 445. The capacitance of a parallel plate capacitor
with air as medium is 3µF. As a dielectric is
=
( R + jR 3 ).( R − jR 3 ) introduced between the plates, the capacitance
( R + jR 3 ) + ( R − jR 3 ) becomes 15 µF. The permittivity of the
medium in C2N-1m-2 is
(a) 8.15×10–11 (b) 0.44×10–10
( 3)
2
R2 + R2
= (c) 15.2×1012 (d) 1.6×10–14
2R
VITEEE-2015
2
4R Ans. (b) : The capacitance of the capacitor is written as
Zeq = = 2R
2R
εo A
So, total impedance between P and Q is C=
d
ZPQ = R + Zeq = R + 2R = 3R
Capacitance of a same parallel plate capacitor with the
443. A condenser of capacity C1 is charged to introduction of a dielectric medium-
potential V1 and then disconnected. Uncharged Kε A
capacitor of capacity C2 is connected in C' = o
parallel with C1. The resultant potential V2 is d
C1V1 CV Where K is the dielectric constant of a medium
(a) (b) 2 1
C1 + C 2 C1 So,
C'
=K⇒K=
15
=5
C 3
C 2 V1 CV
(c) (d) 1 1 ε
C1 + C 2 C2 Now, K =
εo
MHT-CET 2020
Ans. (a) : Charge on first capacitor (q1) = C1V1 ε = Kεo = 5 × 8.854 × 10–12
Charge on second capacitor (q ) = 0 = 0.44 × 10–10 C2 N–1 m–2
2

When they are connected in parallel then total charge 446. The capacity of a capacitor is 4×10−6 F and its
potential is 100V. The energy released on
q = q1 + q2 (∴ q = C1V1)
discharging it fully will be
And capacitance (C) = C1 + C2 (a) 0.02J (b) 0.04J
Let, V' be the common potential difference across each (c) 0.025J (d) 0.05J
capacitor then q = CV'
VITEEE-2011
q C1V1
∴ V' = = Ans. (a) : Given that,
C C1 + C 2 Capacitance, C = 4 × 10–6 F
444. The capacity of a parallel plate air capacitor is Potential = 100 V
2 µF and voltage between the plates is changing Energy released on discharging the capacitor-
at the rate of 3 V/s. The displacement current
1
in the capacitor is E = CV 2
2
(a) 2 µA (b) 3 µA
1
(c) 5 µA (d) 6 µA = × 4 ×10−6 × (100)2
2
MHT-CET 2015
= 0.02 Jule
Objective Physics Volume-III 937 YCT
447. A capacitor of capacitance C charged by an Ans. (c) : Given that,
amount Q is connected in parallel with an
4
uncharged capacitor of capacitance 2C. The Capacitance, C ' = C
final charges on the capacitors are : 3
Q Q Q 3Q d
(a) , (b) , Thickness (t) =
2 2 4 4 2
Q 2Q Q 4Q We know that,
(c) , (d) ,
3 3 5 5 εA
Capacitance is air (C) = o …..(i)
Karnataka CET-2019 d
Ans. (c) : A capacitor is charged by battery and then Now Let 't' thickness dielectric slab introduced
battery was replaced with another uncharged capacitor
of equal capacitance.
The charge stored in the first capacitor = Q
Charge stored in second capacitor = 0
The two capacitors attain common potential (VC)
The final charges on two capacitors are-
εo A
CQ Q C' = …….(ii)
Q1 = CVC = =  t 
3C 3 d − t + 
 K
2Q
Q 2 = 2CVC = 4
Now, C = C '
3 3
448. A voltmeter reads 4 V when connected to a
4 ε oA ε oA
parallel plate capacitor with air as a dielectric. =
When a dielectric slab is introduced between 3 d d d
d− +
plates for the same configuration, voltmeter 2 2K
reads 2V. What is the dielectric constant of the  1 1 
material ? 4 1 − + =3
(a) 0.5 (b) 2  2 2K 
(c) 8 (d) 10  1 1 3
1 −  + =
Karnataka CET-2014  2  2K 4
Ans. (b) : Let Vo = potential difference plates of air K +1 3
filled parallel plate capacitor =
2K 4
K=2
450. 64 small drops of mercury, each of radius r and
charge q coalesce to form a big drop. The ratio
of the surface density of charge of each small
drop with that of the big drop is :
(a) 64 : 1 (b) 1 : 64
(c) 1 : 4 (d) 4 : 1
Vo Karnataka CET-2002
V=
K Ans. (c) : We know that,
V 4 q
K= o = =2 σ=
V 2 4πr 2
449. Capacitance of a parallel plate capacitor 64r3 = R3
becomes 4/3 times its original value if a R = 4r
d q
dielectric slab of thickness t = is inserted σs = (for small surface)
2 4πr 2
between the plates :
64q
[d is the separation between the plates]. The σl = (for large surface)
4πR 2
dielectric constant of the slab is :
(a) 4 (b) 8 Ratio of small drop to large drop is-
(c) 2 (d) 6 σs q 64q
= ÷
Karnataka CET-2003 σl 4πr 4πR 2
2

Objective Physics Volume-III 938 YCT


(c) decreases by a factor K2
4π ( 4r )
2
q
= × (d) decreases by a factor K
4πr 2
64q
J&K CET- 2002
σs 1
= Ans. (b) : When the empty space between the plates of
σl 4 a capacitor is filled by a liquid of dielectric constant K,
the capacitance of the capacitor increased by the factor
σs : σl = 1 : 4 of K.
451. While a capacitor remains connected to a The capacitance C = KC Where C is the capacitance
0 0
battery, a dielectric slab is slipped between the without the dielectric.
plates. Then
454. A parallel plate capacitor is first charged and
(a) the energy stored in the capacitor decreases
then a dielectric slab is introduced between the
(b) the electric field between the plates increases plates. The quantity that remains unchanged is
(c) charges flow from the battery to the capacitor (a) charge Q (b) potential V
(d) the potential difference between the plates is (c) capacity C (d) energy U
changed
J&K CET- 1999
Karnataka CET-2001
SRMJEE-2012
Ans. (c) : While a capacitor remains connected to a
Ans. (a) : Charge Q remains unchanged when capacitor
battery and dielectric slab is slipped between the plates,
is the charged then dielectric slab is introduced between
the potential difference between the plates remain
the plates.
unchanged. The introduction of dielectric slab increases
the charge of capacitor which flows from the battery. When the capacitor is kept at a voltage, it gains charge.
Now when the system is isolated, the charge present on
452. A dielectric of dielectric constant K is
capacitor cannot change because of law of conservation
introduced such that half of area of capacitor
of charge.
of capacity C is occupied by it. The new
capacity is ∴ Charge always remains constant in isolated systems.
C 455. A capacitor is charged and then made to
(a) 2C (b) discharge through a resistance. The time
2
C constant is τ. In what time will the potential
(c) (1 + K) (d) 2C(1 + K) difference across the capacitor decrease by
2 10%?
J&K CET- 2007 (a) τln0.1 (b) τln0.9
Ans. (c) : The dielectric is introduced such that, half of
10 11
its area is occupied by it. In the given case the two (c) τ ln (d) τ In
capacitors are in parallel. 9 10
JIPMER-2016
Ans. (c) : While discharging, charge on the capacitor
after time t is,
Q = Q 0 e− t / τ ...... (i)
We know that potential difference across C is
proportional to charge (Q) on the capacitor.
Q C ' = C1 + C 2 For the potential difference to fall by 10%, Q also must
Aε 0 KAε 0 fall by 10%
C1 = and C 2 = ∴ Q = 0.9Q0
2d 2d
Aε 0 KAε 0 Using equation (i) we get,
C' = +
2d 2d 0.9Q 0 = Q 0 e − t / τ
C 10
C ' = (1 + K ) = et / τ ......... (ii)
2 9
453. The empty space between the plates of  10  t
capacitor is filled by liquid of dielectric ⇒ ln   =
9 τ
constant K. The capacitance of capacitor
(a) increases by a factor K2  10 
∴ t = τ ln  
(b) increases by a factor K 9

Objective Physics Volume-III 939 YCT


456. A capacitor of capacity 0.1 µF connected in Ans. (b): We know that,
series to a resistor of 10 M Ω is charged to a
A
certain potential and then made to discharge ε 0   K1
ε AK
C1 =   = 0 1
through the resistor. The time in which the 2
potential will take to fall to half its original d d
value is (Given, log10 2 = 0.3010)  
 
2
(a) 2 s (b) 0.693s
A
(c) 0.5 s (d) 1.0 s ε0   K 2
ε AK 2
C2 =  
2
AP EAMCET -2010 = 0
d d
Ans. (b) : Given that,  
2
A capacitance of capacitor = 0.1µF
The capacitors C1 and C2 are in Parallel and their
equivalent capacitance is
ε0 A
C′ = C1 + C 2 = ( K1 + K 2 )
d
This Combination is in series with C3.
Hence, the net capacitance is
Charge on capacitor during discharging is given by,
1 1 1 d d
q = q 0 e− t / RC = + = +
C C C 3 ε 0 A ( K 1 + K 2 ) 2ε 0 A K 3
′′ ′
q0
Now, q= 1 d  1 1 
2 =  + 
q0 C'' ε 0 A  ( K1 + K 2 ) 2K 3 
= q 0 e − t / RC
2 ε 0 AK 1 1 1
C′′ = Where = +
t = RC ln 2 d K ( K1 + K 2 ) 2K 3
= (10 × 106) × (0.1 × 10–6) × 0.693
458. A photographic flash unit consists of a xenon
= 0.693 s
filled tube. It gives a flash of average power
457. A parallel plate air capacitor with plate area 2000 W for 0.04 sec. The flash is due to
‘A’ and plate separation ‘d’ filled with three discharge of a fully charged capacitor of 40µF.
dielectric materials of dielectric constants K1, The voltage to which it is charged before a
K2 and K3 is as shown in the figure. If the three flash is given by the unit is
dielectric materials are replaced by a single (a) 1500 V (b) 2000 V
dielectric material of dielectric constant ‘K’ (c) 2500 V (d) 3000 V
filling the entire capacitor, the capacity
AMU-2016
remains the same, then
Ans. (b) : Given that,
P = 2000 W, t = 0.04 s, C = 40 µF = 40 × 10–6
Total energy stored in capacitor = Power × time
= 2000 × 0.04 J …… (i)
Energy stored in capacitor can also be written as,
CV 2
E= (Q C = 4µF)
2
1
1 1 1 1 = × 40 ×10−6 V 2 ...... (ii)
(a) = + + 2
K K1 K 2 2K 3
Using question (i) and (ii), we get-
1 1 1 1
(b) = + 2000 × 0.04 = × 40 ×10−6 V 2
K K1 + K 2 2K 3 2
1 K1 K 2 ⇒ 2
V = 4 × 10 6
(c) = + 2K 3
K K1 + K 2 V = 2000 V
K1K 2 459. An air capacitor is charged with an amount of
(d) K = charge q and dipped into an oil tank. If the oil
K1 + K 2 is pumped out, the electric field between the
AP EAMCET-28.04.2017, Shift-I plates of capacitor will

Objective Physics Volume-III 940 YCT


(a) increase (b) decrease Ans. (c): Let the charge stored by capacitor with
(c) remain the same (d) become zero dielectric constant 3 be Q.
DCE-2009 Q2
Thus, energy stored, E1=
Ans. (a) : Let, the capacitance of the air capacitor is Co. 2C1
The capacitance becomes C′ = KC0 Since the charge remains the same after charging the
Where dielectric constant of oil medium.
Q2
Now charge on capacitor is q′ = C′V′ dielectric, the new energy stored, E2 =
2C 2
When oil pump out, the charge is q = C0 V
E C K 2
Thus, q′ = q ⇒ 1= 2= 2=
C′V′ = C0V E 2 C 1 K 1 3
KCoV′ = CoV  KAε 0 
C = 
V = KV′  d 
If d be the distance between capacitor plates, the electric 462. In a parallel plate air capacitor of capacitance
field V = Ed 4F if the lower half of air space is filled with a
As V∝ E, the electric field will increase when oil is material of dielectric constant 3, its capacitance
pumped out. changes to
460. Two capacitors of capacitance C are connected 4 8
in series If one of them is filled with dielectric (a) F (b) F
3 3
substance k, what is the effective capacitance ?
(c) 8 F (d) 12 F
kC
(a) (b) C(k +1) EAMCET-1993
(1+ k )
Ans. (c) : Net capacitance, C = C1 + C2 ….. (i)
2kC
(c) (d) none of these εA
1+ k Given, air capacitance C = 0 =4F
d
DCE-2007
Capacitance of Ist half plate,
Ans. (a) : When plates of Capacitor are separated by a
dielectric Medium of dielectric Constant k, its capacity A
ε0
kε 0 A C1 = 2 = 1 ε0 A
Cm = = kC0 = kC [Where, C0 = C] d 2 d
d
1
Now, two capacitor kC and C are in Series, that effect C1 = × 4 = 2F
Capacitance, 2
1 1 1 Capacitance of 2nd half plate,
= +
C kC C A
Kε 0
1 1+ k C2 = 2
⇒ = d
C' kC
kC K A 3
⇒ C' = =  ε 0  = × 4 = 6F
k +1 2 d 2
461. A parallel plate capacitor with a dielectric slab According to question, C1 and C2 are in parallel-
of dielectric constant 3 filling the space between ∴ C = C1 + C2
the plates is charged to a potential V. The =2+6
battery is then disconnected and then replaced
= 8F
by another dielectric slab of dielectric constant
2. If the energies stored in the capacitor before 463. A parallel plate condenser with oil between the
and after the dielectric slab is changed are E1 plates (dielectric constant of oil K = 2) has a
and E2 then E1/E2 is capacitance C. It the oil is removed, the
9 4 capacitance of the capacitor becomes
(a) (b)
5 9 (a) 2 C (b) 2 C
2 3 C C
(c) (d) (c) (d)
3 2 2 2
AP EAMCET(Medical)-2013 EAMCET-1998

Objective Physics Volume-III 941 YCT


Ans. (d): The Capacitance of a parallel plate capacitor Ans. (d): According to question,
with dielectric (oil) between its plates is
Kε 0 A
C= ........ (i)
d
When dielectric (oil) is removed, so capacitance
ε0 A
C0 = ....... (ii)
d
Given, C1 = 1µF, C2 = 1.5µF, C3 = 2.5 µF, C4 = 0.5 µF
Comparing equation (i) & (ii), we get
Equivalent capacitance of C1 and C2, are in series,
C = KC0
1 1 1
C C = +
C0 = = ∴[K = 2] C' C 1 C 2
K 2
1 1 1 2.5 5
464. A parallel plate capacitor filled with a = + = =
C' 1 1.5 1.5 3
dielectric of relative permittivity 5 between its
plates is charged to acquire an energy E and 3
C ' = µF
isolated. If the dielectric is replaced by another 5
of relative permittivity 2, its energy becomes Equivalent capacitance of C3 and C4, are in series,
1 1 1 6 12
(a) E (b) 0.4 E = + = =
C" 2.5 0.5 2.5 5
(c) 2.5 E (d) 6.25 E 5
C" = µF
EAMCET-1999 12
q 2 Total capacitance (Ceq) = C' + C''
Ans. (c) : Energy stored in capacitor (E) = 3 5 61
2C = + = µF
1 5 12 60
As charged is conserved so E ∝ Total charge (Q total ) = C eq × V
C
61 61
Let C0 be the capacitance of air filled capacitor = × 30 = µC ....(i)
When the dielectric material with dielectric constant K 60 2
is filled between the places, the capacitance becomes This charge is distributed in upper and lower of the
circuit,
C' = K C0
Q' Q''
E 2 5C0 = = V (parallel)
Then, = = 2.5 C ' C ''
E1 2C0
Q' C ' 3/ 5 36
= = =
E2 Q'' C '' 5 /12 25
= = 2.5 ⇒ E 2 = 2.5E
E 36Q"
Q' =
465. Four capacitors with capacitance C1=1µF, 25
C2=1.5µF, C3=2.5µF and C4 =0.5µF are From equation (i), we get–
connected as shown in figure to a 30V sources. 61
The potential difference between points a and b Q Q '+ Q'' =
2
is
36Q'' 61
+ Q '' =
25 2
61Q '' 61 25
= ⇒ Q '' = µF
25 2 2
61 25 36
∴ Q' = − = = 18µC
2 2 2
If potential difference ( V ' ) across capacitor C1

(b) −9V Q ' 18


(a) 5V V' = = = 18V
(c) 10V (d) −13V C1 1
CG PET -2016 Q '' 25 / 2
V '' = = = 5V
AP EAMCET-19.08.2021, Shift-II C3 5/ 2

Objective Physics Volume-III 942 YCT


So, VA – Va = 18V Q 2Q
(a) Q, 2Q (b) ,
VA – Vb = 5 V 3 3
(VA – Vb) – (VA – Va) = 5 – 18 3Q 2Q 4Q
(c) ,3Q (d) ,
Va – Vb = – 13V 2 3 3
466. In given circuit when switch S has been closed BITSAT-2005
then charge on capacitor A and B respectively are Ans. (b) : Given circuit,

Charge on C1,
(a) 3q, 6q (b) 6q, 3q
(c) 4.5q, 4.5q (d) 5q,4q  C1  Q
Q1 =  ×Q =
CG PET- 2008 C
 1 + 2C1  3
Ans. (b) : Charge after switch S has been closed. Charge on C2,
 2C1  2
Q1 =  ×Q = Q
C
 1 + 2C1  3
468. n identical drops, each of capacitance C and
charged to a potential V, coalesce to form a
bigger drop. Then the ratio of the energy
stored in the big drop to that in each small
In capacitor A and B are connected in parallel. drop is
So, potential difference across them will be equal (a) n5/3 : 1 (b) n4/3 : 1
VA = VB (c) n : 1 (d) n3 : 1
2/3
(e) n : 1
q1
VA = .....(i) Kerala CEE - 2010
6C
Ans. (a):Volume of big drop= n × volume of small drop
q2
VB = ......(ii) 4 3 4
3C πR = n × πr 3
3 3
Q q1 = 3q and q 2 = 6q R = n1/3r
q2 Capacitance of small drop, C = 4πε0r
So, q1 = ....(iii)
2 Capacitance of big drop C′ = 4πε0R
Charge is conserved, C′ = 4πε0n1/3r
q1 + q2 = 3q + 6q = 9q ...(iv) C′ = n1/3C
On putting the value of q1 in equation (iv), we get - q q
q2/2 + q2 = 9q ⇒ q2 = 6q The potential of small drop, V = =
C 4πε 0 r
From equation (iii), we get -
nq
6q The potential of big drop, V ' =
q1 =
2
= 3q ( 0 ) n1/ 3r
4 πε

467. Two capacitors C1 and C2 = 2C1 are connected in V′ = n2/3V


a circuit with a switch between them as shown in 1
the figure. Initially the switch is open and C1 ∴ Energy of small drop = 2 CV
2

holds charge Q. The switch is closed. At steady


Energy of big drop = C'V '2 = n1/ 3C ( n 2 / 3 V )
1 1 2
state, the charge on each capacitor will be
2 2
5/3 1 2
= n CV
2
Energy( big drop ) n 5/ 3
∴ =
Energy( smalldrop ) 1

Objective Physics Volume-III 943 YCT


469. N identical drops of mercury are charged C
simultaneously to 10 V. When combined to In series combination ( C1 ) = n
form one large drop, the potential is found to In parallel combination, C = nC
2
be 40 V, the value of N is
1
(a) 4 (b) 6 Energy (U) = CV 2

2
(c) 8 (d) 10
U 2 C 2 V22 nC V 2 n 2C 1
(e) 12 ∴ = ⋅ = × = ×
U1 C1 V12 C n2V2 C n2
Kerala CEE 2007 n
Ans. (c) : Given, U2 1
= n2 × 2 = 1
V' = 40V, V = 10V U1 n
Volume of large drop = N × volume of small drop U2 = U1
4 3 4 Hence, the potential difference becomes nV and energy
πR = N × πr 3
3 3 remains the same.
R3 = Nr3 471. Assertion (A): Two capacitor of same capacity
are connected first in parallel and then in
R = N1/ 3 r …..(i) series. The ratio of resultant capacities in the
Also potential on big drop of radius, two cased will be 4:1
 K ( NQ )  Reason (R) : In parallel, capacity increases and
V' =   = 40 in series capacity decreases
 R  (a) Both A and R are true and R is a correct
Potential on 1 small drop of radius, explanation for A
KQ (b) Both A and R are true but R is not a correct
V= = 10 explanation for A
r
(c) A is true, R is false
 KNQ  (d) A is false, R is true
V '  R 
Hence, = AP EAMCET-23.08.2021, Shift-II
V KQ / r
Ans. (b) : Two capacitors of same capacities C.
40 Nr In series,
=
10 R C× C C
CS = =
N C+C 2
4= Q R = N1/ 3 r 
N1/ 3 In parallel,
N2/3 = 4 CP = C + C = 2C
The ratio of CP and CS is-
N = 43/ 2 = 8
CP 2C 4
= =
470. Initially, ‘n’ identical capacitors are joined in CS C 1
parallel and are charged to potential ‘V’. Now 2
they are separated and joined in series. Then, CP : Cs = 4:1
(a) potential difference remains the same and the In parallel, CP > C & In series Cs > C.
energy increases ‘n’ times. So, in parallel, capacity is increased but in series,
capacity is decreased.
(b) potential difference is ‘nV’ and energy
increases ‘n’ times. Hence, Both A and B are true but R is not a correct
explanation for A
(c) potential difference and the total energy of the
472. Assertion : If the distance between parallel plate
combination remain the same.
of a capacitor is halved and dielectric constant is
(d) potential difference becomes ‘nV’ and energy three times, then the capacitance become 6 times.
remains the same. Reason : Capacity of the capacitor does not
MHT-CET 2020 depend upon the nature of the material .
Ans. (d) : Potential difference of each capacitor = V (a) If both Assertion and Reason are correct and
the Reason is a correct explanation of the
Connected in series the potential difference = nV
Assertion

Objective Physics Volume-III 944 YCT


(b) If both Assertion and Reason are correct but 474. If initial capacitance of capacitor is C, then
the Reason is not a correct explanation of the final capacity of capacitor will be
Assertion
(c) If both the Assertion is correct but Reason is
incorrect
(d) If both the Assertion and Reason are
incorrect.
(e) If the Assertion is incorrect but the Reason is
correct.
AIIMS-1997 (a) KC (b) (K+1)C
(c) (K+1) C/2 (d) (K-1)C
Ans. (c) Capacitance of a capacitor,
CG PET- 2007
Kε 0 A
C= Ans. (c) :
d
K
C∝
d
C1 K1 d 2 K d / 2 1
∴ = × = × = A ε0 KA ε 0
C 2 d1 K 2 d 3K 6 C1 = ,C 2 =
2d 2d
C2 = 6C1
In parallel,
Q
Capacity of capacitor (C) = Ceq = C1 + C2
V
Aε KAε 0
As nature of the material (dielectric constant) is a factor Ceq = 0 +
influencing the capacity, therefore, Reason is incorrect. 2d 2d
Aε  K + 1 
473. A parallel plate condenser is filled with two Ceq = 0  
dielectric K1 and K2 as shown in figure. Area of d  2 
each plate is A and separation is d. Its
capacitance will be Ceq = C
( K + 1)
2
475. When two identical capacitors are charged
individually to different potentials and then
connected in parallel, after disconnecting from
the source
(a) [(ε0A)/d] × [K1 + K2] (a) net charge = sum of initial charges
(b) [(ε0A)/d] × [(K1 + K2)/2] (b) net potential difference ≠ sum of individual
(c) [(ε0A)/d] × [2(K1 + K2)] initial potential difference
(d) [(ε0A)/d] × [K1 − K2] (c) net energy stored < sum of individual initial
UP CPMT-2014, 2002 energy
Ans. (b) : Given circuit, (d) All of the above
AP EAMCET (23.09.2020) Shift-I
Ans. (d) :

K1ε 0 A / 2 K1ε 0 A
C1 = =
d 2d
K 2ε0 A After connecting in parallel,
C2 =
2d
Ceq = C1 + C2 (parallel)
K1ε 0 A K 2 ε 0 A
Ceq = +
2d 2d
ε 0 A K1 + K 2
Ceq = ×
d 2
Objective Physics Volume-III 945 YCT
Potential difference, Ans. (d): Potential difference across two capacitors
C V + C 2 V2 CVl + CV2 connected in parallel,
VC = 1 1 =
Cl + C2 C+C C1V1 + C 2 V2
Veq =
C1 + C 2
V1 + V2
VC =
2 Potential of charged capacitor V1 = V & potential of
Which is not equal to the sum of individual initial uncharged capacitor V2 = 0,
potential difference C1V
∴ Veq =
VC ≠ V1 + V2 C1 + C 2
For an isolated system, charge remains always 478. A parallel plate capacitor has a capacity 80 ×
conserved i.e., sum of initial charge = sum of final 10–6 F, when air is present between the plates.
charge The volume between the plates is then
Since, potential difference across each capacitors are completely filled with a dielectric slab of
different and connected in parallel. dielectric constant 20. The capacitor is now
So, charge redistribution will take place from higher connected to a battery of 30 V by wires. The
potential capacitor to lower potential capacitor. dielectric slab is then removed. Then, the
It charge flow, then there is some loss of energy in charge that passes now through the wire is
form of heat. (a) 45.6 × 10–3 C (b) 25.3 × 10–3 C
Ef < Ei (c) 120 × 10–3 C (d) 125 × 10–3 C
Hence, option (d) is correct. AP EAMCET -2012
476. The capacity of a parallel plate capacitor with Ans. (a) : Given,
no dielectric substance but with a separation of
C = 80×10–6 F
0.4 cm is 2 µF. The separation is reduced to
half and it is filled with a dielectric substance of K = 20
value 2.8. The final capacity of the capacitor is V = 30V
(a) 11.2 µF (b) 15.6 µF C'= KC = 20×80×10–6F
(c) 19.2 µF (d) 22.4 µF Q ∆Q = Q' – Q
UPSEE - 2010 ∆Q = VC' – CV
Ans. (a) : Capacitance of parallel plate, ∆Q = V(C' – C)
kε A ∆Q = 30(20×80×10–6 – 80×10–6)
C= 0
d ∆Q = 30×80×10–6 ×19
k ∆Q = 45.6×10–3 C
C∝
d 479. A capacitor is kept connected to a battery and
C1 k1 d 2 a dielectric slab is inserted between the plates.
Q = ×
C2 k 2 d l During the process
2 1 0.4 / 2 (a) Work is done at the cost of the battery
= × (b) Work is done at the cost of the capacitor
C 2 2.8 0.4
(c) Work is done at the cost of the battery and
C2 = 11.2 µF capacitor
477. A capacitor of capacitance ‘C1’ is charged upto (d) No work is done
potential ‘V’ and is then disconnected from the SRMJEEE - 2014
battery. It is then connected to uncharged
capacitor, in parallel, whose capacitance is ‘C2’ Ans. (a) : Charge on capacitor,
. The potential difference across each capacitor Q = CV
is Dielectric constant = k
C + C2 C2 V Capacitance (C') = kC [k > 1]
(a) 1 (b)
C2 V C1 + C 2 Voltage is constant on the battery remains connected.
C1 + C 2 C1V So, the charge will increase.
(c) (d)
C1V C1 + C 2 During the process work is done at the cost of the
MHT-CET 2019 battery.

Objective Physics Volume-III 946 YCT


480. A capacitor of capacitance 5µF is connected as
shown in the figure. The internal resistance of
the cell is 0.5 Ω. The amount of charge on the
capacitor plates is

(a) 2µF (b) 32µF


(c) 6µF (d) 8µF
JEE Main-26.06.2022, Shift-II
(a) 80 µC (b) 40 µC Ans. (c) : As we know that,
(c) 20 µC (d) 10 µC Aε 0
Coriginal =
JIPMER-2010 d
Ans. (d) : In steady state, there will be no current in the
capacitor branch. Net resistance of the circuit.
R = 1 + 1 + 0.5 = 2.5 Ω
Current drawn from the cell,
V 2.5
i= = = 1A
R 2.5
Aε 0 2Aε 0
Potential drop across two parallel branches, C1 = = =C
d d
V = E – ir = 2.5 – 1 × 0.5 = 2.0 V 2
So, charge on the capacitor plates, KAε 0 2KAε 0 6Aε 0
C2 = = = = 3C
d d d
q = CV = 5 × 2 = 10 µC. 2
481. The capacity of an air condenser is 2.0 µF. If So, C1 & C2 are in series
medium is place the capacity becomes 12 µF. C1C 2 C × 3C 3C
Cnew = = =
The dielectric constant of the medium will be C1 + C 2 C + 3C 4
(a) 5 (b) 4 3 2Aε 0 3 Aε 0
= × = ×
(c) 3 (d) 6 4 d 2 d
According to question,
JIPMER-2004
3
Ans. (d) : When dielectric is placed in an electric field, Cnew = Coriginal
2
electric charges do not flow through the material as they 3
do in an electrical conductor but only slightly shift from C new = × 4
2
their average equilibrium positions causing dielectric Cnew = 6 µF
polarization.
483. A parallel plate capacitor has two layers of
Dielectric constant of medium, dielectrics as shown in figure. This capacitor is
connected across a battery. Then the ratio of
Capacity of condenser with medium
K= potential difference across the dielectric layer is
Capacity of condenser without medium
12
K= =6
2
482. A parallel plate capacitor with plate area A and
plate separation d = 2 m has a capacitance of 4 4 1
(a) (b)
µF. The new capacitance of the system if half of 3 2
the space between them is filled with a 1 3
(c) (d)
dielectric material of dielectric constant K = 3 3 2
(as shown in figure) will be: HP CET-2018

Objective Physics Volume-III 947 YCT


Ans. (d) : Given, 485. The capacity of a parallel plate capacitor with
Dielectric constant K1 = 2 and K2 = 6 no dielectric but with a separation 0.4 cm is
2µF. The separation is reduced to half and it is
As we know that,
filled with a dielectric of value 2.8. The final
The capacitance of a parallel- plate capacitor filled with capacity of the capacitor is
a dielectric is- (a) 11.2µF (b) 5.6µF
ε 0 KA (c) 4.0µF (d) 22.4µF
C=
d EAMCET-1997
ε 0 K1A Ans. (a) K = 2.8, d1 = 0.4 cm, d2 = 0.2 cm
C1 =
d C1 = 2µF
ε0 K 2 A C1 d 2 2 0.2
C2 = Q = ⇒ = ⇒ C2 = 4
d C 2 d1 C 2 0.4
q = C1 V1 = C2 V2
Final capacitor,
V1 C 2 K 2 6
= = = C = C2K
V2 C1 2K1 2 × 2 C = 4×2.8 = 11.2 µF
V1 3 486. A parallel plate capacitor has plate area 40 cm2
=
V2 2 and plates separation 2mm. The space between
the plates is filled with a dielectric medium of a
484. A capacitor of capacitance C1 = 1 µF is charged thickness 1mm and dielectric constant 5. The
using a 9V battery. C1 is then removed from capacitance of the system is :
the battery & connected to capacitors C2 and
10 3
C3 of 2µF and 3 µF respectively as shown in the (a) ε0 F (b) ε0 F
figure. Find the charge on C3 after equilibrium 3 10
has reached is (c) 24 ε0F (d) 10ε0F
JEE Main-25.01.2023, Shift-I
Ans. (a) : Here, d1 = d2 = 1 mm = 1 × 10–3 m
Let C1 is capacitance due to dielectric constant k and C2
is capacitance in the air medium.
(a) 4.5×10–6 C (b) 3.5×10–6 C
(c) 2.5×10–6 C (d) 1.5×10–5 C
AP EAMCET-04.07.2022, Shift-I
Ans. (a) : Given circuit,

Then,

C1 = 1µF=1×10–6 F ε 0 kA
C1 =
d1
C2 = 2µF=2×10–6 F
C3 = 3µF=3×10–6 F ε 0 × 5 × 40 × 10−4
C1 = = 20ε 0
When C1 is charge Vvolt battery then total charge on C1 1×10 −3
Q = C1V = 1×10–6×9 = 9×10–6C ε0 A
C2 =
But when battery is removed and C1 is connected to C2 d2
and C3 in parallel then Voltage will same and total
charge are conserved ε 0 × 40 × 10 −4
C2 = = 4ε 0
So, C1V + C2V + C3V = Q {because voltage same} 1× 10−3
1 × 10–6 V + 2 × 10–6 V + 3 × 10–6 V = 9 × 10–6V Here, C1 and C2 are in series.
6×10–6V = 9×10–6 1 1 1 1 1
So, = + = +
V = 1.5 Volt Ceq C1 C 2 20ε 0 4ε 0
So, charge on C3, Q3 = C3V = 3 × 10–6 × 1.5 20 10
Ceq = ε 0 = ε0 F
= 4.5 × 10–6 C 6 3

Objective Physics Volume-III 948 YCT


487. In the given circuit diagram the current 488. Two capacitors C1 and C2 = 2 C1 are connected in
through the battery and the charge on the a circuit with a switch between them as shown in
capacitor respectively in steady state are the figure. Initially the switch is open and C1
holds charge Q. The switch is closed. At steady
state, the charge on each capacitors will be

Q 2Q
(a) Q, 2Q (b) ,
(a) 1 A and 3µC (b) 17 A and 0µC 3 3
6 12 3Q 2Q 4Q
(c) A and µC (d) 6 A and 0µC (c) ,3Q , (d)
7 7 2 3 3
(e) 11 A and 3 µC Manipal UGET-2019
KERALA CEE - 2009 Ans. (b) : From charge distribution law on C1 and C2 at
steady state.
Ans. (e) : Given, V= 6V, C = 0.5µF = 0.5 × 10−6F
R1, R2 and R3 are connected in parallel,

Charge (Q1) = Ceff V


Q
= C1 ×
( C1 + C2 )
Q
= C1 ×
At steady state, current through capacitor is zero ( C1 + 2C1 )
because capacitor get fully charged. Hence current Q
through resistor 4Ω gets zero. Q1 =
3
Then, effective Resistance,
Q
1 1 1 1 Q2 = C2 ×
= + + C1 + C 2
R eff R 1 R 2 R 3
Q
1 1 1 11 = 2C1 ×
= + + = C1 + 2C1
1 2 3 6
2
6 Q2 = Q
R eff = Ω 3
11
489. A thin metal plate P is inserted half way
V 6
Q Current, I = = = 11A between the plates of a parallel plate capacitor
R eff 6 /11 of capacitance C in such a way that it is parallel
∴ Charge on the capacitor (Q) = CV to the two plates. The capacitance now becomes
Q = 0.5 × 10−6 × 6 C
(a) C (b)
2
Q = 3 × 10−6 C
(c) 4C (d) None of these
Q = 3µC
Manipal UGET-2019
Objective Physics Volume-III 949 YCT
Ans. (a): 491. In a capacitor of capacitance 20µF the distance
between the plates is 2mm. If a dielectric slab
of width 1mm and dielectric constant 2 is
inserted between the plates, then the new
capacitance will be–
(a) 22µF (b) 26.6µF
(c) 52.2µF (d) 13µF
BCECE-2007
NEET Oct - 2020
For a parallel plate capacitor,
Ans. (b) : Given that,
A
C = εo Capacitance, C = 20µF,
d
A A Distance, d = 2mm,
C1 = ε o = 2ε o Thickness of slab, t = 1mm,
d/2 d
C1 = 2C ....(i) Dielectric constant, k = 2
So, initially
A 2ε o A
C2 = εo =
d/2 d
C2 = 2C ....(ii)
Since C1 and C2 are in series combination
So, Equivalent Capacitance,
1 1 1 1 1
= + = +
Ceq C1 C 2 2C 2C
A
⇒ C eq = 1C C = εo
d
490. An insulator plate is passed between the plates A
of a capacitor. Then current 20µF = ε o
2
After Dielectric is inserted,
εo A
C' =
( d − t ) + t / k 
Where, t = thickness of slab
K = Dielectric constant
ε o A 2ε o A
(a) first flows from A to B and then from B to A Q C' = =
1 3
(b) first flows from B to A and then from A to B 1+
2
(c) always flows from B to A
2 ε A 
(d) always flows from A to B C' = 2 ×  o 
3 2 
BCECE-2008
UP CPMT-2006 4
= × 20µF
Ans. (b) : 3
C' = 26.6µF

A 492. A parallel plate air capacitor has a capacitance


B C. When it is half filled with a dielectric of
dielectric constant 5, the percentage increase in
the capacitance will be :
As insulator plate is passed between the plates of the (a) 400% (b) 66.6%
capacitor, its capacity increases first and then decreases (c) 33.3% (d) 200%
as the plate slip out. As a result, positive charge on plate Karnataka CET-2006
A increases first and then decreases, hence current in AIIMS-2017
outer circuit flows from B to A and then from A to B. APEAMCET-25.04.2018 (Shift-II)
Objective Physics Volume-III 950 YCT
Ans. (b): Given, dielectric constant, k = 5 The capacitors are connected in series,
1 1 1 1
= + +
Ceq 1 2 5
1 17
=
Ceq 10
10
Ceq = µF
17
Total charge (Q) = VC
10 100
For parallel plate, = 10× = µC
17 17
kε ° A
Capacitance (C) = Potential difference across the 2µF capacitor,
d
kε A 2kε ° A Q 100 /17 50
Q C1 = ° = V= = = V
d d C 2 17
2 494. A 1 µF capacitor C is connected to a battery of
ε ° A 2ε ° A 10 V through a resistance 1 MΩ. The voltage
C2 = = across C after 1 sec is approximately
d d
2 (a) 5.6 V (b) 7.8 V
1 1 1 d 1  d 1  (c) 6.3 V (d) 10 V
∴ = + =  + 1 =  + 1
Ceq C 1 C 2 2ε o A  k  2ε o A  5  WB JEE 2016

1 6 d 10ε° A 5ε° A Ans. (c) : Given,


= × ⇒ Ceq = =
Ceq 10 ε° A 6d 3d C = 1µF = 1 ×10 −6 F

 5 ε° A ε° A  R = 1M Ω = 1× 106 Ω
 − 
% increase in the capacitance =  3 d d 
A
ε°
d
5  2
=  − 1 × 100 = × 100
 3  3
= 66.6 %
Voltage across C after t second,
493. Three capacitors of capacitance 1.0, 2.0 and 5.0
µF are connected in series to a 10 V source. The   
 −b 

V = V ×  1 − e RC  
potential difference across the 2.0 µF capacitor  
 
is
100 20  
−
1
−6

6 

(a) V (b) V V = 10  1 − e 1×10 ×10  
17 17  
 
50 V = 10 × (1–e–1)
(c) V (d) 10V
17
V = 10 × (1– 0.37)
WB JEE 2017
V = 10 × 0.63
Ans. (c) : V = 6.3 volt
495. A capacitor of capacitance C0 is charged to a
potential V0 and is connected with another
capacitor of capacitance C as shown. After
closing the switch S, the common potential
across the two capacitors becomes V. The
capacitance C is given by

Objective Physics Volume-III 951 YCT


Ans. (a) : Charge on the capacitor (Q) = CV
When a capacitor (C) is charged through a resistance
(R) by a battery of emf (e), then the charge on the
capacitor vary with time,
q(t) = Q[1 – e–t/RC]
C0 (V0 − V) C0 (V − V0 ) 497. A 500 µF capacitor is charged at a steady rate
(a) (b)
V0 V0 of 100 µC/sec. The potential across the
capacitor will be 10 V after an interval of
C0 (V + V0 ) C0 (V0 − V)
(c) (d) (a) 5 sec (b) 25 sec
V V (c) 20 sec (d) 50 sec
WB JEE-2013 MP PET-2008
Ans. (d) : Ans. (d) : Given that, C = 500 µF, V = 10 V,
q = 100 µC/sec = 100 × 10−6 C/sec
T=?
We know that,
Q=qt
Given, Q
t=
Capacitance of the capacitor = C0 q
Potential difference = V0 CV 500 µ F ×10 V
t= =
Initial charge (Qi) = C0V0 q 100 × 10 −6 C / sec
When, switch S is closed, the common potential
5 × 10−3 C
difference across the two capacitors of capacitance C0 is t=
V. 100 × 10−6 C / sec
1
Switch is closed, t= × 1000 sec
20
Charge (Qf1 ) = VC0
t = 50 sec
Qf2 = VC 498. A parallel plate condenser is filled with two
dielectrics as shown in figure. Area of each pate
Total charge on capacitors C0 and C when switch S is
is A m2 and the separation is d metre. The
closed. dielectric constants are K1 and K2 respectively.
So, Its capacitance in farad will be
Qi = Qf1 + Qf2
C0V0 = C0V + CV
CV = C0V0 – C0V
CV = C0 (V0 − V)
C0 ( V0 − V )
C=
V
496. When a capacitor of capacity C is charged 2ε 0 A  K 1 + K 2 
through a resistance R by a battery of emf ε0 (a)  
d  K1 K 2 
the charge on the capacitor vary with time
according to the relation, (Here Q is the 2 ε 0 A  K1 K 2 
maximum charge on the capacitor): (b)  
d  K1 + K 2 
(a) q ( t ) = Q 1 − e − t / RC 
ε 0 A  K1 + K 2 
(b) q ( t ) = Qe t / RC (c) 
d  2K1K 2 

(c) q ( t ) = Qe − t / RC ε 0 AK1K 2
(d)
(d) q ( t ) = Q e − t / RC
− 1 2 ( d 2 k1 + d1K 2 )

MP PET -2013 JIPMER-2013

Objective Physics Volume-III 952 YCT


Ans. (b) : Given, C1 = 4 C …..(i)
And the area of each plate = A m2 K ε A/2
Now, C2 = 2 o
K1Aε 0 2K1Aε 0 d
C1 = =
d/2 d 4 εo A
C2 =
2K 2 Aε 0 2 d
C2 =
d C2 = 2 C ....(ii)
Q Capacitors are connected in series From equation (i) and (ii), we get-
C1C 2 Ceq = C1 + C2
∴ C' = Ceq = 4 C + 2 C
C1 + C 2
Ceq = 6 C
 2K1Aε o  2K 2 Aε o 
   500. The time in second required to produce a
C' =   
d d
potential difference of 20 V across a capacitor
 2K1Aε o 2K 2 Aε o 
 +  of 1000 µF when it is charged at the steady rate
 d d  of 200 µC/s is
 A 2εo 2  (a) 50 (b) 100
4  K1K 2 
d2  (c) 150 (d) 200
C' = 
Aε EMCET-2002
( 2K1 + 2K 2 ) o
d Ans. (b) : Given,
4  K K  Aε Rate of charging (∆q/∆t) = 200 µC/s = 200 × 10–6 C/s
C' =  1 2  o
2  K1 + K 2  d Potential difference (V) = 20 V
Capacitor (C) = 1000 µF = 1000 ×10–6 F
2ε A  K K 
C' = 0  1 2  We know,
d  K1 + K 2 
Q = CV
499. A capacitor having capacitance 1µF with air, is ∴ Q = 1000 × 10 −6 × 20
filled with two dielectrics as shown. How many
times capacitance will increase? Q = 2 ×10−2 C
Again we know that,
charge (Q)
(∆q/∆t) =
time (t)
Q
t=
(a) 12 (b) 6 ( / ∆t )
∆ q
(c) 8/3 (d) 3 2 ×10−2
∴ t=
JIPMER-2008 200 × 10−6
Ans. (b) : Given, K1 = 8, K2 = 4 t = 100s
We know that, 501. One end each of a resistance r, capacitor C and
εA resistance 2r are connected together. The other
C= 0 ends are respectively connected to the positive
d
terminals of batteries. P, Q, R having
After filling with dielectrics, we have two capacitors. respectively emf's E, E and 2E. The negative
A terminals of the batteries are then connected
K1ε0   together. In this circuit, with steady current
C1 = 2
the potential drop across the capacitor is
d
E E
8 εoA (a) (b)
C1 = 3 2
2 d
2E
4ε o A (c) (d) E
C1 = 3
d EAMCET-2006

Objective Physics Volume-III 953 YCT


Ans. (a) : 503. Two identical thin metal plates has charge q1
and q2 respectively such that q1 > q2. The plates
were booth close to each other to form a
parallel plate capacitor of capacitance C. The
potential difference between them is:
(q1 + q 2 )
(a)
C
(q1 − q 2 )
(b)
∴ Current through the outer loop, C
2E − E (q1 − q 2 )
i= (c)
2r + r 2C
E 2(q1 − q 2 )
i= (d)
3r 2C
∴ Potential difference across upper branch JEE Main-29.07.2022, Shift-II
E 4E
= E +  r = Ans. (c) :
 3r  3
Q Potential difference across capacitor
4E
= −E
3
E
=
3 Electric field within the plates-
502. A parallel plate air capacitor has a capacitance
E = E q1 + E q 2
of 100 µF. The plates are at a distance d apart.
If a slab of thickness t(t < d) and dielectric E = E1 + E 2
constant 5 is introduced between the parallel
plates, then the capacitance will be q q
E= 1 − 2
(a) 50µF (b) 100µF 2ε0 A 2ε0 A
(c) 200µF (d) 500 µF q − q2
AIIMS-27.05.2018(M) E= 1
2Aε 0
Ans. (d) : Given, Capacitor (C) = 100 µF, dielectric
constant (K) = 5 The potential difference between the plates-
+Q (t) –Q VA − VB = Ed
 q − q2 
= 1 d
 2Aε 0 
K=5

q1 − q 2
=
 Aε 
2 0 
 d 
d
q1 − q 2
t≤d ∆V =
2C
Capacitance of capacitor with dielectric,
504. Two capacitors of capacities 1 µF and C µF are
C' = KCo connected in series and the combination is
Where, Co is the Capacitance of parallel plate air charged to a potential difference of 120 V. If
capacitor the charge on the combination is 80 µC, the
Capacitor, Co = 100 µF energy stored in the capacitor of capacity C in
Also, K=5 µJ is
(a) 1800 (b) 1600
C' = 5 × 100
(c) 14400 (d) 7200
C' = 500 µ F AP EAMCET -2010
Objective Physics Volume-III 954 YCT
Ans. (b): Capacitance 1µF and cµF are connected in 22.365
= = 1.78V
series 12.5
1 1 1 1 Vcommon 1.79 V
= + = +1
Ceq C 1 C
506. Two parallel plate capacitors of capacitances C
1 1+ C and 2C are connected in parallel and charged
=
Ceq C to a potential difference V0. The battery is then
C disconnected and the region between the plates
∴ Ceq = of the capacitor C completely filled with a
1+ C
Given, material of dielectric constant 2. The potential
difference across the capacitors now becomes.
V = 120 V and q = 80 µC
q = Ceq V V0 V0
(a) (b)
C 4 2
80 = ×120
1+ C (c)
3V0
(d) V0
C = 2 µF 4
Energy stored in the capacitor of capacity C AMU-2009
2
1 q Ans. (c) : Initial total charge of the system is,
U=
2 C Qi = Q1+ Q2 = CV0 + 2CV0 = 3CV0

1 ( 80 × 10 )
−6 2 When dielectric is inserted in 'C' so the capacitance
= × become KC.
2 2 × 10−6
Final charge,
1 80 ×10−6 × 80 × 10−6
= × Qf = Q'1+ Q'2 = k CV' + 2 CV'
2 2 × 10−6
U = 1600 µJ = (2 + 2) CV' = 4 CV' [ k = 2]

505. A 3.55 µF capacitor C1 is charged to a potential Where V' = common potential offer disconnected the
difference V0 = 6.30 V using a battery. The battery,
battery is then removed and the capacitor is As battery is disconnected so total charge will remain
connected as shown below to an uncharged 8.95
unchanged
µF capacitor C2, when the switch S is closed
charge flows Thus, Qi = Qf
3CV0 = 4 CV'
3V0
V' =
4
507. A parallel-plate capacitor of area A, plate
separation d and capacitance C is filled with
from C1 to C2 until the capacitors have the
four dielectric materials having dielectric
same potential difference V equal to
constants k1, k2, k3 and k4 as shown in the
(a) 1.50 V (b) 1.79 V figure below. If a single dielectric material is to
(c) 1.93 V (d) 2.15 V be used to have the same capacitance C in this
AMU-2003 capacitor then its dielectric constant k is given
by
Ans. (b) :Given,
C1 = 3.55 µF, C2 = 8.95 µF, V1 = 6.3 V, Vcommos = ?
C V + C2 V2
Vcommon = 1 1 (Q V2 = 0 )
C1 + C2
3.55 × 6.30 + 8.95 × 0
=
3.55 + 8.95
3.55 × 6.30
=
3.55 + 8.95

Objective Physics Volume-III 955 YCT


(a) k = k1 + k2 + k3 + 3k4 Ans. (a) : We know that,
2
(b) k = (k1 + k 2 + k 3 ) + 2k 4
3
2 3 1
(c) = +
k k1 + k 2 + k 3 k 4
1 1 1 1 3
(d) = + + +
k k1 k 2 k 3 2k 4
NEET-2016
Ans. (c) : We know that, Initial capacitance,
kεo A Aεo
Capacitance, C = C= ....(i)
d d
Ak1εo 2k ε A New capacitance,
C1 = = 1 o
3d / 2 3d Aεo Aεo
Similarly, C' = =
d' ( 3d / 5 )
2k 2 ε0 A
C2 =
3d 5 Aεo
C' =
2k ε A 3 d
C3 = 3 0
3d 5
C' = C
Calculation of dielectric constant K since all these 3
capacitor are in parallel.
509. Find the capacitance in shown figure
(zone A) CA = C 1 + C 2 + C 3
2ε o A
= ( k1 + k 2 + k 3 )
3d
k εA 2k ε A
Now taking (zone B), C4 = 4 o = 4 o
d/2 d
Since zone A and zone B are in series.
So, 2KAε0 2KAε0
(a) (b)
C zoneA × C 4 (K + 1)d d
Ceq =
C zoneA + C 4 (K + 1)Aε0 2KAε 0
(c) (d)
2d (K 2 + 1)d
2ε o A 2k ε A
kε o A ( k1 + k 2 + k 3 ) × 4 o JIPMER-2018
= 3d d
d 2εo A 2k 4 εo A
( k1 + k 2 + k 3 ) + Ans. (a) :
3d d
2 3 1
= +
k k1 + k 2 + k 3 k 4
508. Between the plates of parallel plate capacitor of
capacity C, two parallel plate of the same
material and area same as the plate of the
original capacitor, are placed. If the thickness
of these plates is equal to 1/5 of the distance
between the plates of the original capacitor,
then the capacity of the new capacitor is: εA
Q C= 0
d
5 3
(a) C (b) C This is the combination of series.
3 5
3C 10C ∴ Capacitance of dielectric portion,
(c) (d)
10 3 ε KA
C1 = 0
AP EAMCET(Medical)-2003 d/2

Objective Physics Volume-III 956 YCT


Capacitance in air portion, Combination in series
ε0 A C1C 2
C2 = ∴ Ceq = ….(i)
d/2 C1 + C 2
C1 C2 Putting the value of C1 & C2 in equation (i), we get-
∴ Cequivalent =
C1 + C 2 ε0 A ε 0 AK
×
ε0 KA ε0 A Ceq =
(d – t ) t
× ε0 A ε AK
d/2 d/2 + 0
Kε0 A ε0 A
+
(d – t ) t
d/2 d/2 ε0 A ⋅ ε0 AK
4Kε02 A 2 t (d – t )
Ceq =
= d2 ε0 At + ε0 AK ( d – t )
2Kε 0 A + 2ε 0 A
t (d – t )
d
ε0 AK ε0 A t (d – t )
4Kε 02 A 2 d = ×
= × t ( d – t ) ε0 At + ε 0 AK ( d – t ) 
d2 ( 2Kε0 A + 2ε0 A )
4ε 20 A 2 K 2ε 0 AK  
= = ε0 AK  ε0 A 
2ε 0 A (1 + K ) d d (1 + K ) =  
ε0 AK  d – t + t 
2ε 0 AK
( )
 K
Ceq =
d (1 + K ) ε0 A ε0 A
Ceq = =
510. The separation between the plates of a parallel  t  1 
 d – t +  d +  – 1 t
plate capacitor is d and area of each plate is A.  K K 
When a slab of material of dielectric constant ε0 A
K and thickness t is introduced between the Ceq =
 1
plates, the capacitance becomes d – t 1 – 
 K
εo A εo A
(a) (b) 511. A parallel plate air capacitor has capacity C,
 1  1
d + t 1 –  d + t 1+  distance of separation between plates is d and
 K  K
potential difference V is applied between the
εo A εo A plates. Force of attraction between the plates of
(c) (d)
 1  1 the parallel plate air capacitor is
d – t 1 –  d – t 1+ 
 K  K C2 V 2 CV 2
(a) (b)
AMU-2002 2d 2d
Ans. (c) : CV 2 C2 V 2
(c) (d)
d 2d 2
AIPMT-2015
Ans. (b) : Given, Capacity of capacitor = C
Separation between plates = d
Potential difference = V
ε0 A Force of attraction between plates,
Q C=
d Q2
F=
From figure, 2Aε0
Capacitance in air portions & dielectric portion is C1 &
C2, Q Q = CV
( CV )
2
ε0 A Kε 0 A
C1 = , C2 = ∴ F=
(d – t ) t 2Aε0

Objective Physics Volume-III 957 YCT


C2 V 2 513. As show in the figure, in steady state, the
F= ….(i) charge stored in the capacitor is ..........× 10-6C.
2Aε0
ε0 A
Q C=
d
∴ ε0A = Cd
Putting the value in equation (i)
C2 V 2
F=
2Cd
CV 2
F= JEE Main-27.07.2022, Shift-II
2d
512. The charge on the capacitor of capacitance C Ans. (10) Given, diagram,
shown in the figure below will be

CER1
(a) CE (b)
R1 + r
In steady state,
CER 2 CER1
(c) (d) Capacitor behaves like open circuit, then modified
R2 + r R2 + r circuit.
WB JEE-2011
Ans. (c) : In steady state capacitor behave as open
circuit so modified circuit-

Apply KVL on the circuit,


10 – I×100 –10×I = 0
10 = 110×I
10
I = A
110
Apply voltage division rule ∴Voltage across 100 Ω
Voltage across R2
10 100
E R2 V100Ω = Ι R = ×100 = V
VR 2 = 110 11
r + R2 This voltage is in parallel with capacitor
VR 2 is equal to voltage across capacitor. q=CV
Q 100
Q V= q = 1.1×10–6 ×
C 11
q = 10×10–6C
∴ Q = CV
Q = charge across capacitor 514. A parallel plate air capacitor of capacitance C
is connected to a cell of emf V and then
∴ Q = C × VR 2
disconnected from it. A dielectric slab of
C × ER 2 CER 2 dielectric constant K, which can just fill the air
Q= =
(r + R2 ) R2 + r gap of the capacitor, is now inserted in it.
Which of the following is incorrect?

Objective Physics Volume-III 958 YCT


(a) The potential difference between the plates (a) 4 (b) 2
decreases K times (c) 6 (d) 1
(b) The energy stored in the capacitor decreases AP EAMCET -2013
K times
εA
(c) The change in energy stored is Ans. (b) : We know, C = 0
d
1 1 
CV 2  − 1 Let the gap for capacitor (C1) first be p and that for
2 K 
capacitor (C2) second be q.
(d) The charge on the capacitor is not conserved
εA εA d
AIPMT-2015 C1 = 0 ,C 2 = 0 and p + q =
p q 2
Ans. (d) : From question,
Since, both capacitor are in series -
initial charge Q = CV
New capacitance (C) = K.C 1 1 1
So, = +
Thus, new potential difference. C ' C1 C 2
Q V C1C2
( V ') = = C' =
C1 + C 2
KC K
Q2 ε0 A ε0 A
Initial energy (Ui) = .
2C p q
C' =
ε0 A ε0 A
Ui Q2 +
Final energy (Uf) = = p q
K 2KC
Change in energy (∆U) = Uf – Ui ε0 A 2ε A  d
C' = = 0 Q p + q = 2 
Q2 Q2 (p + q) d
= −
2KC 2C C' = 2C
2
Q 1  C'
=2
=  − 1 Hence,
C
2C  K 
1 1  516. Two dielectrics of dielectric constants K1 and
∆U = CV 2  − 1 K2 are filled in gap of parallel plate capacitor
2 K 
as shown in figure
Hence, statement in option (a), (b), (c) are correct and
(d) is incorrect.
515. Two metal plates each of area 'A' form a
parallel plate capacitor with air in between the
plates. The distance between the plates is 'd'. A
d
metal plate of thickness and of same area A The capacitance of capacitor will be :
2
is inserted between the plates to form two ε0 A(K1 + K 2 ) ε 0 A  K1 + K 2 
capacitors of capacitances C1 and C2 as shown
(a) (b)  
2d 2d  K1K 2 
in the figure. If the effective capacitance of the
two capacitors is C' and the capacitance of the ε0  K1K 2  ε 0 A  K1 + K 2 
(c)   (d)  
capacitor initially is C, then
C'
is d  K1 + K 2  d  K1K 2 
C
BCECE-2003
Ans. (a) :

K lεo ( A )
C=
d
Objective Physics Volume-III 959 YCT
For first plate: – Now, the overall charge –
Q = Ce × V
K1εo ( A / 2 ) K1εo A
C1 = = = 2 × 10–6 × 1800 C
d 2d
= 3.6 × 10–3 C
For second plate : –
Q 3.6 ×10 –3
K 2εo ( A / 2 ) K 2εo A VQR = = = 600V
C2 = = Cp 6 × 10 –6
d 2d
Q2 = C2 × VQR = 4 × 10–6 × 600 = 2.4 × 10–3 C
C1 and C2 are in parallel connection Q3 = C3 × VQR = 2 × 10–6 × 600 = 1.2 × 10–3 C
CR = C1 + C2 518. A parallel plate capacitor of area 'A' plate
KεA K ε A separation 'd' is filled with two dielectrics as
= 1 o + 2 o shown. What is the capacitance of the
2d 2d
arrangement?
εo A
CR = ( K1 + K 2 )
2d
517. In the circuit shown in figure, if the point R is
earthed and point P is given a potential of
+1800 V, then charges on C2 and C3 are 3Kε0 A
(a)
respectively 4d
4Kε0 A
(b)
3d
(K + 1)ε0 A
(c)
2d
K(K + 3)ε0 A
(d)
2(K + 1)d
BITSAT-2016
Ans. (d) : Given figure,
(a) 2.4 ×10–3C; 1.2 ×10–3C
(b) 1.6 ×10–3C; 0.8 ×10–3C
(c) 32 ×10–3C; 1.6 ×10–3C
(d) 4.8 ×10–3C; 2.4 ×10–3C
AP EAMCET (23.04.2018) Shift-2
Ans. (a) Given, potential at point P = 1800 V
(A / 2)ε0 Aε0
C1 = =
d/2 d
Aε0
C2 = K
d
Aε0
C3 = K
2d
C1 × C2
∴ Ceq = + C3
C1 + C 2
The potential difference across Q & R Aε0 Aε
×K 0
VPQ + VQR = 1800, Ceq = d d + K Aε0
Aε0 Aε0 2d
CP = 4 + 2 = 6 µF +K
d d
C1C p 3 × 6 18 K(K + 3)ε0 A
∴ Ceffective = = = = 2µF Ceq =
C1 + C p 3+ 6 9 2(K + 1)d

Objective Physics Volume-III 960 YCT

You might also like